Joy Board Exam 28.08.08

  • April 2020
  • PDF

This document was uploaded by user and they confirmed that they have the permission to share it. If you are author or own the copyright of this book, please report to us by using this DMCA report form. Report DMCA


Overview

Download & View Joy Board Exam 28.08.08 as PDF for free.

More details

  • Words: 57,022
  • Pages: 241
Saturday, November 10, 2007 StuffedNurse : diannemaydee's 300 items NLE reviewer

Situation – Richard has a nursing diagnosis of ineffective airway clearance related to excessive secretions and is at risk for infection because of retained secretions. Part of Nurse Mario’s nursing care plan is to loosen and remove excessive secretions in the airway. Mario listens to Richard’s bilateral sounds and finds that congestion is in the upper lobes of the lungs. The appropriate position to drain the anterior and posterior apical segments of the lungs when Mario does percussion would be: A. Client lying on his back then flat on his abdomen on Trendelenburg position B. Client seated upright in bed or on a chair then leaning forward in sitting position then flat on his back and on his abdomen C. Client lying flat on his back and then flat on his abdomen D. Client lying on his right then left side on Trendelenburg position When documenting outcome of Richard’s treatment Mario should include the following in his recording EXCEPT: A. Color, amount and consistency of sputum B. Character of breath sounds and respiratory rate before and after procedure C. Amount of fluid intake of client before and after the procedure D. Significant changes in vital signs When assessing Richard for chest percussion or chest vibration and postural drainage, Mario would focus on the following EXCEPT: A. Amount of food and fluid taken during the last meal before treatment B. Respiratory rate, breath sounds and location of congestion C. Teaching the client’s relatives to perform the procedure D. Doctor’s order regarding position restrictions and client’s tolerance for lying flat Mario prepares Richard for postural drainage and percussion. Which of the following is a special consideration when doing the procedure? A. Respiratory rate of 16 to 20 per minute B. Client can tolerate sitting and lying positions C. Client has no signs of infection D. Time of last food and fluid intake of the client

The purpose of chest percussion and vibration is to loosen secretions in the lungs. The difference between the procedures is: A. Percussion uses only one hand while vibration uses both hands B. Percussion delivers cushioned blows to the chest with cupped palms while vibration gently shakes secretion loose on the exhalation cycle C. In both percussion and vibration the hands are on top of each other and hand action is in tune with client’s breath rhythm D. Percussion slaps the chest to loosen secretions while vibration shakes the secretions along with the inhalation of air Situation – A 61 year old man, Mr. Regalado, is admitted to the private ward for observation after complaints of severe chest pain. You are assigned to take care of the client. When doing an initial assessment, the best way for you to identify the client’s priority problem is to: A. Interview the client for chief complaints and other symptoms B. Talk to the relatives to gather data about history of illness C. Do auscultation to check for chest congestion D. Do a physical examination while asking the client relevant questions Nancy blames God for her situation. She is easily provoked to tears and wants to be left alone, refusing to eat or talk to her family. A religious person before, she now refuses to pray or go to church stating that God has abandoned her. The nurse understands that Nancy is grieving for her self and is in the stage of: A. bargaining B. denial C. anger D. acceptance

Which of the following ethical principles refers to the duty to do good? A. Beneficence B. Fidelity C. Veracity D. Nonmaleficence

During which step of the nursing process does the nurse analyze data related to the patient's health status? A. Assessment B. Implementation C. Diagnosis D. Evaluation The basic difference between nursing diagnoses and collaborative problems is that A. nurses manage collaborative problems using physician-prescribed interventions. B. collaborative problems can be managed by independent nursing interventions. C. nursing diagnoses incorporate physician-prescribed interventions. D. nursing diagnoses incorporate physiologic complications that nurses monitor to detect change in status. Situation – Mrs. Seva, 52 years old, asks you about possible problems regarding her elimination now that she is in the menopausal stage. Instruction on health promotion regarding urinary elimination is important. Which would you include? A. Hold urine as long as she can before emptying the bladder to strengthen her sphincter muscles B. If burning sensation is experienced while voiding, drink pineapple juice C. After urination, wipe from anal area up towards the pubis D. Tell client to empty the bladder at each voiding Mrs. Seva also tells the nurse that she is often constipated. Because she is aging, what physical changes predispose her to constipation? A. inhibition of the parasympathetic reflex B. weakness of sphincter muscles of anus C. loss of tone of the smooth muscles of the colon D. decreased ability to absorb fluids in the lower intestines The nurse understands that one of these factors contributes to constipation: A. excessive exercise B. high fiber diet C. no regular time for defecation daily D. prolonged use of laxative You will do nasopharyngeal suctioning on Mr. Abad. Your guide for the length of insertion of the tubing for an adult would be: A. tip of the nose to the base of the neck

B. the distance from the tip of the nose to the middle of the neck C. the distance from the tip of the nose to the tip of the ear lobe D. eight to ten inches Situation– Mr. Dizon, 84 years old, brought to the Emergency Room for complaint of hypertension, flushed face, severe headache, and nausea. You are doing the initial assessment of vital signs. You are to measure the client’s initial blood pressure reading by doing all of the following EXCEPT: A. Take the blood pressure reading on both arms for comparison B. Listen to and identify the phases of Korotkoff’s sound C. Pump the cuff to around 50 mmHg above the point where the pulse is obliterated D. Observe procedures for infection control A pulse oximeter is attached to Mr. Dizon’s finger to: A. Determine if the client’s hemoglobin level is low and if he needs blood transfusion B. Check level of client’s tissue perfusion C. Measure the efficacy of the client’s anti-hypertensive medications D. Detect oxygen saturation of arterial blood before symptoms of hypoxemia develops In which type of shock does the patient experiences a mismatch of blood flow to the cells? A. Distributive B. Cardiogenic C. Hypovolemic D. Septic The preferred route of administration of medication in the most acute care situations is which of the following routes? A. Intravenous B. Epidural C. Subcutaneous D. Intramuscular After a few hours in the Emergency Room, Mr. Dizon is admitted to the ward with an order of hourly monitoring of blood pressure. The nurse finds that the cuff is too narrow and this will cause the blood pressure reading to be: A. inconsistent B. low systolic and high diastolic C. higher than what the reading should be D. lower than what the reading should be

Through the client’s health history, you gather that Mr. Dizon smokes and drinks coffee. When taking the blood pressure of a client who recently smoked or drank coffee, how long should the nurse wait before taking the client’s blood pressure for accurate reading? A. 15 minutes B. 30 minutes C. 1 hour D. 5 minutes While the client has pulse oximeter on his fingertip, you notice that the sunlight is shining on the area where the oximeter is. Your action will be to: A. Set and turn on the alarm of the oximeter B. Do nothing since there is no identified problem C. Cover the fingertip sensor with a towel or bedsheet D. Change the location of the sensor every four hours

When taking blood pressure reading the cuff should be: A. deflated fully then immediately start second reading for same client B. deflated quickly after inflating up to 180 mmHg C. large enough to wrap around upper arm of the adult client 1 cm above brachial artery D. inflated to 30 mmHg above the estimated systolic BP based on palpation of radial or bronchial artery To ensure client safety before starting blood transfusions the following are needed before the procedure can be done EXCEPT: A. take baseline vital signs B. blood should be warmed to room temperature for 30 minutes before blood transfusions is administered C. have two nurses verify client identification, blood type, unit number and expiration date of blood D. get consent signed for blood transfusion Mr. Bruno asks what the “normal” allowable salt intake is. Your best response to Mr. Bruno is: A. 1 tsp of salt/day with iodine and sprinkle of MSG B. 5 gms per day or 1 tsp of table salt/day C. 1 tbsp of salt/day with some patis and toyo D. 1 tsp of salt/day but no patis and toyo Which of the following methods is the best method for determining nasogastric tube placement in the stomach?

A. X-ray B. Observation of gastric aspirate C. Testing of pH of gastric aspirate D. Placement of external end of tube under water Which of the following is the most important risk factor for development of Chronic Obstructive Pulmonary Disease? A. Cigarette smoking B. Occupational exposure C. Air pollution D. Genetic abnormalities When performing endotracheal suctioning, the nurse applies suctioning while withdrawing and gently rotating the catheter 360 degrees for which of the following time periods? A. 10-15 seconds B. 30-35 seconds C. 20-25 seconds D. 0-5 seconds The nurse auscultates the apex beat at which of the following anatomical locations? A. Fifth intercostal space, midclavicular line B. Mid-sternum C. 2” to the left of the lower end of the sternum D. 1” to the left of the xiphoid process Which of the following terms describes the amount of blood ejected per heartbeat? A. Stroke volume B. Cardiac output C. Ejection fraction D. Afterload You are to apply a transdermal patch of nitoglycerin to your client. The following are important guidelines to observe EXCEPT: A. Apply to hairless clean area of the skin not subject to much wrinkling B. Patches may be applied to distal part of the extremities like forearm C. Change application and site regularly to prevent irritation of the skin D. Wear gloves to avoid any medication on your hand The GAUGE size in ET tubes determines:

A. The external circumference of the tube B. The internal diameter of the tube C. The length of the tube D. The tube’s volumetric capacity The nurse is correct in performing suctioning when she applies the suction intermittently during: A. Insertion of the suction catheter B. Withdrawing of the suction catheter C. both insertion and withdrawing of the suction catheter D. When the suction catheter tip reaches the bifurcation of the trachea The purpose of the cuff in Tracheostomy tube is to: A. Separate the upper and lower airway B. Separate trachea from the esophagus C. Separate the larynx from the nasopharynx D. Secure the placement of the tube Which priority nursing diagnosis is applicable for a patient with indwelling urinary catheter? A. Self esteem disturbance B. Impaired urinary elimination C. Impaired skin integrity D. Risk for infection An incontinent elderly client frequently wets his bed and eventually develop redness and skin excoriation at the perianal area. The best nursing goal for this client is to: A. Make sure that the bed linen is always dry B. Frequently check the bed for wetness and always keep it dry C. Place a rubber sheet under the client’s buttocks D. Keep the patient clean and dry As a Nurse Manager, DMLM enjoys her staff of talented and self motivated individuals. She knew that the leadership style to suit the needs of this kind of people is called: A. Autocratic B. Participative C. Democratic D. Laissez Faire A fire has broken in the unit of DMLM R.N. The best leadership style suited in cases of emergencies like this is:

A. Autocratic B. Participative C. Democratic E. Laissez Faire Which step of the management process is concerned with Policy making and Stating the goals and objective of the institution? A. Planning B. Organizing C. Directing D. Controlling In the management process, the periodic checking of the results of action to make sure that it coincides with the goal of the institution is termed as: A. Planning B. Evaluating C. Directing D. Organizing The Vision of a certain agency is usually based on their beliefs, Ideals and Values that directs the organization. It gives the organization a sense of purpose. The belief, Ideals and Values of this Agency is called: A. Philosophy B. Mission C. Vision D. Goals and Objectives Mr. CKK is unconscious and was brought to the E.R. Who among the following can give consent for CKK’s Operation? A. Doctor B. Nurse C. Next of Kin D. The Patient Mang Carlos has been terminally ill for 5 years. He asked his wife to decide for him when he is no longer capable to do so. As a Nurse, You know that this is called: A. Last will and testament B. DNR C. Living will D. Durable Power of Attorney

Mang Carlos has a standing DNR order. He then suddenly stopped breathing and you are at his bedside. You would: A. Give extraordinary measures to save Mang Carlos B. Stay with Mang Carlos and Do nothing C. Call the physician D. Activate Code Blue It is not a legally binding document but nevertheless, Very important in caring for the patients. A. BON Resolution No. 220 Series of 2002 B. Patient’s Bill of Rights C. Nurse’s Code of Ethics D. Philippine Nursing Act of 2002 In monitoring the patient in PACU, the nurse correctly identify that checking the patient’s vital signs is done every: A. 1 hour B. 5 minutes C. 15 minutes D. 30 minutes diannemayde R.N is conducting a research on her unit about the effects of effective nursepatient communication in decreasing anxiety of post operative patients. Which of the following step in nursing research should she do next? A. Review of related literature B. Ask permission from the hospital administrator C. Determine the research problem D. Formulate ways on collecting the data Before diannemaydee perform the formal research study, what do you call the pre testing, small scale trial run to determine the effectiveness of data collection and methodological problem that might be encountered? A. Sampling B. Pre testing C. Pre Study E. Pilot Study On the study “effects of effective nurse-patient communication in decreasing anxiety of post operative patients” What is the Independent variable?

A. Effective Nurse-patient communication B. Communication C. Decreasing Anxiety D. Post operative patient On the study “effects of effective nurse-patient communication in decreasing anxiety of post operative patients” What is the Dependent variable? A. Effective Nurse-patient communication B. Communication C. Anxiety level D. Post operative patient In the recent technological innovations, which of the following describe researches that are made to improve and make human life easier? A. Pure research B. Basic research C. Applied research D. Experimental research Which of the following is not true about a Pure Experimental research? A. There is a control group B. There is an experimental group C. Selection of subjects in the control group is randomized D. There is a careful selection of subjects in the experimental group When Mrs. Guevarra, a nurse, delegates aspects of the clients care to the nurse-aide who is an unlicensed staff, Mrs. Guevarra A. makes the assignment to teach the staff member B. is assigning the responsibility to the aide but not the accountability for those tasks C. does not have to supervise or evaluate the aide D. most know how to perform task delegated Process of formal negotiations of working conditions between a group of registered nurses and employer is A. grievance B. arbitration C. collective bargaining D. strike You are attending a certification on cardiopulmonary resuscitation (CPR) offered and required by the hospital employing you. This is

A. professional course towards credits B. inservice education C. advance training D. continuing education The law which regulated the practice of nursing profession in the Philippines is: A. R.A 9173 B. LOI 949 C. Patient’s Bill of Rights E. Code of Ethics for Nurses This quality is being demonstrated by a Nurse who raise the side rails of a confuse and disoriented patient? A. Autonomy B. Responsibility C. Prudence D. Resourcefulness Nurse Joel and Ana is helping a 16 year old Nursing Student in a case filed against the student. The case was frustrated homicide. Nurse Joel and Ana are aware of the different circumstances of crimes. They are correct in identifying which of the following Circumstances that will be best applied in this case? A. Justifying B. Aggravating C. Mitigating D. Exempting In signing the consent form, the nurse is aware that what is being observed as an ethical consideration is the patient’s A. Autonomy B. Justice C. Accountability D. Beneficence Why is there an ethical dilemma? A. Because the law do not clearly state what is right from what is wrong B. Because morality is subjective and it differs from each individual C. Because the patient’s right coincide with the nurse’s responsibility D. Because the nurse lacks ethical knowledge to determine what action is correct and what action is unethical

Who among the following can work as a practicing nurse in the Philippines without taking the Licensure examination? A. Internationally well known experts which services are for a fee B. Those that are hired by local hospitals in the country C. Expert nurse clinicians hired by prestigious hospitals D. Those involved in medical mission who’s services are for free In signing the consent form, the nurse is aware that what is being observed as an ethical consideration is the patient’s A. Autonomy B. Justice C. Accountability D. Beneficence Nurse Buddy gave Inapsine instead of Insulin to a patient in severe hyperglycemia. He reported the incident as soon as he knew there was an error. A nurse that is always ready to answer for all his actions and decision is said to be: A. Accountable B. Responsible C. Critical thinker D. Assertive Which of the following best describes Primary Nursing? A. Is a form of assigning a nurse to lead a team of registered nurses in care of patient from admission to discharge B. A nurse is responsible in doing certain tasks for the patient C. A registered nurse is responsible for a group of patients from admission to discharge D. A registered nurse provides care for the patient with the assistant of nursing aides The best and most effective method in times of staff and financial shortage is: A. Functional Method B. Primary Nursing C. Team Nursing E. Modular Method You are doing bed bath to the client when suddenly, The nursing assistant rushed to the room and tell you that the client from the other room was in Pain. The best intervention in such case is: A. Raise the side rails, cover the client and put the call bell within reach and then attend to the

client in pain to give the PRN medication B. Tell the nursing assistant to give the pain medication to the client complaining of pain C. Tell the nursing assistant to go the client’s room and tell the client to wait D. Finish the bed bath quickly then rush to the client in Pain Angie is a disoriented client who frequently falls from the bed. As her nurse, which of the following is the best nursing intervention to prevent future falls? A. Tell Angie not to get up from bed unassisted B. Put the call bell within her reach C. Put bedside commode at the bedside to prevent Angie from getting up D. Put the bed in the lowest position ever When injecting subcutaneous injection in an obese patient, It should be angled at around: A. 45 ° B. 90 ° C. 180 ° D. Parallel to the skin The following statements are all true about Z-Track technique except: A. Z track injection prevent irritation of the subcutaneous tissues B. The technique involve creating a Zig Zag like pattern of medication C. It forces the medication to be contained at the subcutaneous tissues D. It is used when administering Parenteral Iron Communication is best undertaken if barriers are first removed. Considering this statement, which of the following is considered as deterrent factor in communication? A. Not universally accepted abbreviations B. Wrong Grammar C. Poor Penmanship D. Old age of the client Nurse DMLM is correct in identifying the correct sequence of events during abdominal assessment if she identifies which of the following? A. Inspection, Auscultation, Percussion, Palpation B. Inspection, Percussion, Palpation, Auscultation C. Inspection, Palpation, Percussion, Auscultation D. Inspection, Auscultation, Palpation, Percussion . To prevent injury and strain on the muscles, the nurse should observe proper body mechanics. Among the following, which is a principle of proper body mechanics?

A. Broaden the space between the feet B. Push instead of pull C. Move the object away from the body when lifting D. Bend at the waist, not on the knees In taking the client’s blood pressure, the nurse should position the client’s arm: A. At the level of the heart B. Slightly above the level of the heart C. At the 5th intercostals space midclavicular line D. Below the level of the heart What principle is used when the client with fever loses heat through giving cooling bed bath to lower body temperature? A. Radiation B. Convection C. Evaporation D. Conduction The most effective way in limiting the number of microorganism in the hospital is: A. Using strict aseptic technique in all procedures B. Wearing mask and gown in care of all patients with communicable diseases C. Sterilization of all instruments D. Handwashing The immunoglobulin of the mother that crosses the placenta to protect the child is an example of: A. Natural active immunity B. Natural passive immunity C. Artificial active immunity D. Artificial passive immunity Richard is a subject of a research lead by his doctor. The nurse knows that all of the following is a correct understanding as his right as a research subject except: A. I can withdraw with this research even after the research has been started B. My confidentiality will not be compromised in this research C. I must choose another doctor if I withdrew from this research D. I can withdraw with this research before the research has been started Which of the following is a normal finding during assessment of a Chest tube in a 3 way bottle system?

A. There is a continuous bubbling in the drainage bottle B. There is an intermittent bubbling in the suction control bottle C. The water fluctuates during inhalation of the patient D. There is 3 cm of water left in the water seal bottle In obtaining a urine specimen for culture and sensitivity on a catheterized patient, the nurse is correct if: A. Clamp the catheter for 30 minutes, Alcoholize the tube above the clamp site, Obtain a syringe and draw the specimen on the tube above the clamp

sterile

B. Alcoholize the self sealing port, obtain a sterile syringe and draw the specimen on the self sealing port C. Disconnect the drainage bag, obtain a sterile syringe and draw the specimen from the drainage bag D. Disconnect the tube, obtain a sterile syringe and draw the specimen from the tube Which of the following is an example of secondary prevention? A. Teaching the diabetic client on obtaining his blood sugar level using a glucometer B. Screening patients for hypertension C. Immunizing infants with BCG D. Providing PPD on a construction site Which of the following is a form of primary prevention? A. Regular Check ups B. Regular Screening C. Self Medication D. Immunization An abnormal condition in which a person must sit, stand or use multiple pillows when lying down is: A. Orthopnea B. Dyspnea C. Eupnea D. Apnea As a nurse assigned for care for geriatric patients, you need to frequently assess your patient using the nursing process. Which of the following needs be considered with the highest priority?

A. Patients own feeling about his illness B. Safety of the client especially those elderly clients who frequently falls C. Nutritional status of the elderly client D. Physiologic needs that are life threatening The component that should receive the highest priority before physical examination is the: A. Psychological preparation of the client B. Physical Preparation of the client C. Preparation of the Environment D. Preparation of the Equipments Legally, Patients chart are: A. Owned by the government since it is a legal document B. Owned by the doctor in charge and should be kept from the administrator for whatever reason C. Owned by the hospital and should not be given to anyone who request it other than the doctor in charge D. Owned by the patient and should be given by the nurse to the client as requested Which of the following categories identifies the focus of community/public health nursing practice? A. Promoting and maintaining the health of populations and preventing and minimizing the progress of disease B. Rehabilitation and restorative services C. Adaptation of hospital care to the home environment D. Hospice care delivery A major goal for home care nurses is A. restoring maximum health function. B. promoting the health of populations. C. minimizing the progress of disease. D. maintaining the health of populations. A written nursing care plan is a tool that: A. Check whether nursing care goals were achieved B. Gives quality nursing care C. Select the appropriate nursing intervention D. Make a nursing diagnosis Gina, A client in prolong labor said she cannot go on anymore. The health care team decided that both the child and the mother cannot anymore endure the process. The baby is premature

and has a little chance of surviving. Caesarian section is not possible because Gina already lost enough blood during labor and additional losses would tend to be fatal. The husband decided that Gina should survive and gave his consent to terminate the fetus. The principle that will be used by the health care team is: A. Beneficence B. Non malfeasance C. Justice D. Double effect Situation – There are various developments in health education that the nurse should know about: The provision of health information in the rural areas nationwide through television and radio programs and video conferencing is referred to as: A. Community health program B. Telehealth program C. Wellness program D. Red Cross program In teaching the sister of a diabetic client about the proper use of a glucometer in determining the blood sugar level of the client, The nurse is focusing in which domain of learning according to bloom? A. Cognitive B. Affective C. Psychomotor D. Affiliative A nearby community provides blood pressure screening, height and weight measurement, smoking cessation classes and aerobics class services. This type of program is referred to as A. outreach program B. hospital extension program C. barangay health program D. wellness program After cleaning the abrasions and applying antiseptic, the nurse applies cold compress to the swollen ankle as ordered by the physician. This statement shows that the nurse has correct understanding of the use of cold compress: A. Cold compress reduces blood viscosity in the affected area B. It is safer to apply than hot compress C. Cold compress prevents edema and reduces pain D. It eliminates toxic waste products due to vasodilation

After receiving prescription for pain medication, Ronnie is instructed to continue applying 30 minute cold at home and start 30 minute hot compress the next day. You explain that the use of hot compress: A. Produces anesthetic effect B. Increases nutrition in the blood to promote wound healing C. Increase oxygenation to the injured tissues for better healing D. Induces vasoconstriction to prevent infection Situation – A nursing professor assigns a group of students to do data gathering by interviewing their classmates as subjects. She instructed the interviewees not to tell the interviewees that the data gathered are for her own research project for publication. This teacher has violated the student’s right to: A. Not be harmed B. Disclosure C. Privacy D. Self-determination Before the nurse researcher starts her study, she analyzes how much time, money, materials and people she will need to complete the research project. This analysis prior to beginning the study is called: A. Validity B. Feasibility C. Reliability D. Researchability Data analysis is to be done and the nurse researcher wants to include variability. These include the following EXCEPT: A. Variance B. Range C. Standards of Deviation D. Mean Nurse Minette needs to schedule a first home visit to OB client Leah. When is a first homecare visit typically made? A. Within 4 days after discharge B. Within 24 hours after discharge C. Within 1 hour after discharge D. Within 1 week of discharge

By force of law, therefore, the PRC-Board of Nursing released Resolution No. 14 Series of 1999 entitled: “Adoption of a Nursing Specialty Certification Program and Creation of Nursing Specialty Certification Council.” This rule-making power is called: A. Quasi-Judicial Power B. Regulatory Power C. Quasi-Legislative Power D. Executive/Promulgating Power Anita is performing BSE and she stands in front of the Mirror. The rationale for standing in front of the mirror is to check for: A. Unusual discharges coming out from the breast B. Any obvious malignancy C. The Size and Contour of the breast D. Thickness and lumps in the breast An emerging technique in screening for Breast Cancer in developing countries like the Philippines is: A. Mammography once a year starting at the age of 50 B. Clinical BSE Once a year C. BSE Once a month D. Pap smear starting at the age of 18 or earlier if sexually active Transmission of HIV from an infected individual to another person occurs: A. Most frequently in nurses with needlesticks B. Only if there is a large viral load in the blood C. Most commonly as a result of sexual contact D. In all infants born to women with HIV infection After a vaginal examination, the nurse determines that the client’s fetus is in an occiput posterior position. The nurse would anticipate that the client will have: A. A precipitous birth B. Intense back pain C. Frequent leg cramps D. Nausea and vomiting The rationales for using a prostaglandin gel for a client prior to the induction of labor is to: A. Soften and efface the cervix B. Numb cervical pain receptors C. Prevent cervical lacerations D. Stimulate uterine contractions

Nurse Lorena is a Family Planning and Infertility Nurse Specialist and currently attends to FAMILY PLANNING CLIENTS AND INFERTILE COUPLES. The following conditions pertain to meeting the nursing needs of this particular population group. Dina, 17 years old, asks you how a tubal ligation prevents pregnancy. Which would be the best answer? A. Prostaglandins released from the cut fallopian tubes can kill sperm B. Sperm can not enter the uterus because the cervical entrance is blocked. C. Sperm can no longer reach the ova, because the fallopian tubes are blocked D. The ovary no longer releases ova as there is no where for them to go. The Dators are a couple undergoing testing for infertility. Infertility is said to exist when: A. a woman has no uterus B. a woman has no children C. a couple has been trying to conceive for 1 year D. a couple has wanted a child for 6 months The correct temperature to store vaccines in a refrigerator is: A. between -4 deg C and +8 deg C B. between 2 deg C and +8 deg C C. between -8 deg C and 0 deg C D. between -8 deg C and +4 deg C Which of the following vaccines is not done by intramuscular (IM) injection? A. Measles vaccine B. DPT C. Hepa-B vaccine D. Tetanus toxoids This vaccine content is derived from RNA recombinants. A. Measles B. Tetanus toxoids C. Hepatitis B vaccines D. DPT This special form is used when the patient is admitted to the unit. The nurse completes the information in this record particularly his/her basic personal data, current illness, previous health history, health history of the family, emotional profile, environmental history as well as physical assessment together with nursing diagnosis on admission. What do you call this record?

A. Nursing Kardex B. Nursing Health History and Assessment Worksheet C. Medicine and Treatment Record D. Discharge Summary These are sheets/forms which provide an efficient and time saving way to record information that must be obtained repeatedly at regular and/or short intervals of time. This does not replace the progress notes; instead this record of information on vital signs, intake and output, treatment, postoperative care, post partum care, and diabetic regimen, etc. This is used whenever specific measurements or observations are needed to be documented repeatedly. What is this? A. Nursing Kardex B. Graphic Flow Sheets C. Discharge Summary D. Medicine and Treatment Record These records show all medications and treatment provided on a repeated basis. What do you call this record? A. Nursing Health History and Assessment Worksheet B. Discharge Summary C. Nursing Kardex D. Medicine and Treatment Record This flip-over card is usually kept in a portable file at the Nurse’s Station. It has 2-parts: the activity and treatment section and a nursing care plan section. This carries information about basic demographic data, primary medical diagnosis, current orders of the physician to be carried out by the nurse, written nursing care plan, nursing orders, scheduled tests and procedures, safety precautions in patient care and factors related to daily living activities. This record is used in the charge-of-shift reports or during the bedside rounds or walking rounds. What record is this? A. Discharge Summary B. Medicine and Treatment Record C. Nursing Health History and Assessment Worksheet D. Nursing Kardex Most nurses regard this conventional recording of the date, time, and mode by which the patient leaves a healthcare unit but this record includes importantly, directs of planning for discharge that starts soon after the person is admitted to a healthcare institution. It is accepted that collaboration or multidisciplinary involvement (of all members of the health team) in discharge results in comprehensive care. What do you call this? A. Discharge Summary

B. Nursing Kardex C. Medicine and Treatment Record D. Nursing Health History and Assessment Worksheet Based on the Code of Ethics for Filipino Nurses, what is regarded as the hallmark of nursing responsibility and accountability? A. Human rights of clients, regardless of creed and gender B. The privilege of being a registered professional nurses C. Health, being a fundamental right of every individual D. Accurate documentation of actions and outcomes A nurse should be cognizant that professional programs for specialty certification by the Board of Nursing accredited through the: A. Professional Regulation Commission B. Nursing Specialty Certification Council C. Association of Deans of Philippine Colleges of Nursing D. Philippine Nurse Association Integrated management for childhood illness is the universal protocol of care endorsed by WHO and is use by different countries of the world including the Philippines. In any case that the nurse classifies the child and categorized the signs and symptoms in PINK category, You know that this means: A. Urgent referral B. Antibiotic Management C. Home treatment D. Out patient treatment facility is needed You know that fast breathing of a child age 13 months is observed if the RR is more than: A. 40 B. 50 C. 60 D. 30 Angelo, An 8 month old child is brought to the health care facility with sunken eyes. You pinch his skin and it goes back very slowly. In what classification of dehydration will you categorize Angelo? A. No Dehydration B. Some Dehydration C. Severe Dehydration D. Diarrhea

In responding to the care concerns of children with severe disease, referral to the hospital is of the essence especially if the child manifests which of the following? A. Wheezing B. Stop feeding well C. Fast breathing D. Difficulty to awaken A child with ear problem should be assessed for the following, EXCEPT: A. is there any fever? B. Ear discharge C. If discharge is present for how long? D. Ear pain If the child does not have ear problem, using IMCI, what should you as the nurse do? A. Check for ear discharge B. Check for tender swellings behind the ear C. Check for ear pain D. Go to the next question, check for malnutrition All of the following are treatment for a child classified with no dehydration except: A. 1,000 ml to 1,400 ml be given within 4 hours B. Continue feeding C. Have the child takes as much fluid as he wants D. Return the child to the doctor if condition worsens An ear infection that persists but still less than 14 days is classified as: A. Mastoiditis B. Chronic Ear Infection C. Acute Ear Infection D. Otitis Media If a child has two or more pink signs, you would classify the child as having: A. No disease B. Mild form of disease C. Urgent Referral D. Very severe disease The nurse knows that the most common complication of Measles is: A Pneumonia and larynigotracheitis

B. Encephalitis C. Otitis Media D. Bronchiectasis A client scheduled for hysterosalpingography needs health teaching before the procedure. The nurse is correct in telling the patient that: A. She needs to void prior to the procedure B. A full bladder is needed prior to the procedure C. Painful sensation is felt as the needle is inserted D. Flushing sensation is felt as the dye in injected In a population of 9,500. What is your estimate of the population of pregnant woman needing tetanus toxoid vaccination? A. 632.5 B. 512.5 C. 450.5 D. 332.5 All of the following are seen in a child with measles. Which one is not? A. Reddened eyes B. Coryza C. Pustule D. Cough Mobilizing the people to become aware of their own problem and to do actions to solve it is called: A. Community Organizing B. Family Nursing Care Plan C. Nursing Intervention D. Nursing Process Prevention of work related accidents in factories and industries are responsibilities of which field of nursing? A. School health nursing B. Private duty nursing C. Occupational health nursing D. Institutional nursing In one of your home visit to Mr. JUN, you found out that his son is sick with cholera. There is a great possibility that other member of the family will also get cholera. This possibility is a/an:

A. Foreseeable crisis B. Health threat C. Health deficit D. Crisis Why is bleeding in the leg of a pregnant woman considered as an emergency? A. Blood volume is greater in pregnant woman; therefore, blood loss is increased B. There is an increase blood pressure during pregnancy increasing the likelihood of hemorrhage C. Pregnant woman are anemic, all forms of blood loss should be considered as an emergency especially if it is in the lower extremity D. The pressure of the gravid uterus will exert additional force thus, increasing the blood loss in the lower extremities Aling Maria is nearing menopause. She is habitually taking cola and coffee for the past 20 years. You should tell Aling Maria to avoid taking caffeinated beverages because: A. It is stimulating B. It will cause nervousness and insomnia C. It will contribute to additional bone demineralization D. It will cause tachycardia and arrhythmias All of the following are contraindication when giving Immunization except: A. BCG Vaccines can be given to a child with AIDS B. BCG Vaccine can be given to a child with Hepatitis B C. DPT Can be given to a child that had convulsion 3 days after being given the first DPT Dose D. DPT Can be given to a child with active convulsion or other neurological disease Theresa, a mother with a 2 year old daughter asks, “at what age can I be able to take the blood pressure of my daughter as a routine procedure since hypertension is common in the family?” Your answer to this is: A. At 2 years you may B. As early as 1 year old C. When she’s 3 years old D. When she’s 6 years old Baby John develops hyperbilirubinemia. What is a method used to treat hyperbilirubinemia in a newborn? A. Keeping infants in a warm and dark environment B. Administration of cardiovascular stimulant C. Gentle exercise to stop muscle breakdown D. Early feeding to speed passage of meconium

The community/Public Health Bag is: A. a requirement for home visits B. an essential and indispensable equipment of the community health nurse C. contains basic medications and articles used by the community health nurse D. a tool used by the Community health nurse is rendering effective nursing procedures during a home visit What is the rationale in the use of bag technique during home visits? A. It helps render effective nursing care to clients or other members of the family B. It saves time and effort of the nurse in the performance of nursing procedures C. It should minimize or prevent the spread of infection from individuals to families D. It should not overshadow concerns for the patient In consideration of the steps in applying the bag technique, which side of the paper lining of the CHN bag is considered clean to make a non-contaminated work area? A. The lower lip B. The outer surface C. The upper tip D. The inside surface How many words does a typical 12-month-old infant use? A. About 12 words B. Twenty or more words C. About 50 words D. Two, plus “mama” and “papa” . During operation, The OR suite’s lighting, noise, temperature and other factors that affects the operation are managed by whom? A. Nurse Supervisor B. Surgeon C. Circulating nurse D. Scrub nurse Before and after the operation, the operating suite is managed by the: A. Surgeon B. Nurse Supervisor C. Nurse Manager D. Chief Nurse

The counting of sponges is done by the Surgeon together with the: A. Circulating nurse B. Scrub nurse C. Assistant surgeon D. Nurse supervisor The OR team performs distinct roles for one surgical procedure to be accomplished within a prescribed time frame and deliver a standard patient outcome. While the surgeon performs the surgical procedure, who monitors the status of the client like urine output, blood loss? A. Scrub Nurse B. Surgeon C. Anaesthesiologist D. Circulating Nurse Surgery schedules are communicated to the OR usually a day prior to the procedure by the nurse of the floor or ward where the patient is confined. For orthopedic cases, what department is usually informed to be present in the OR? A. Rehabilitation department B. Laboratory department C. Maintenance department D. Radiology department In some hip surgeries, an epidural catheter for Fentanyl epidural analgesia is given. What is your nursing priority care in such a case? A. Instruct client to observe strict bed rest B. Check for epidural catheter drainage C. Administer analgesia through epidural catheter as prescribed D. Assess respiratory rate carefully The patient’s medical record can work as a double edged sword. When can the medical record become the doctor’s/nurse’s worst enemy? A. When the record is voluminous B. When a medical record is subpoenaed in court C. When it is missing D. When the medical record is inaccurate, incomplete, and inadequate Disposal of medical records in government hospitals/institutions must be done in close coordination with what agency? A. Department of Interior and Local Government (DILG) B. Metro Manila Development Authority (MMDA)

C. Records Management Archives Office (RMAO) D. Department of Health (DOH) In the hospital, when you need the medical record of a discharged patient for research you will request permission through: A. Doctor in charge B. The hospital director C. The nursing service D. Medical records section You will give health instructions to Carlo, a case of bronchial asthma. The health instruction will include the following, EXCEPT: A. Avoid emotional stress and extreme temperature B. Avoid pollution like smoking C. Avoid pollens, dust, seafood D. Practice respiratory isolation As the head nurse in the OR, how can you improve the effectiveness of clinical alarm systems? A. Limit suppliers to a few so that quality is maintained B. Implement a regular inventory of supplies and equipment C. Adherence to manufacturer’s recommendation D. Implement a regular maintenance and testing of alarm systems

Overdosage of medication or anesthetic can happen even with the aid of technology like infusion pumps, sphygmomanometer and similar devices/machines. As a staff, how can you improve the safety of using infusion pumps? A. Check the functionality of the pump before use B. Select your brand of infusion pump like you do with your cellphone C. Allow the technician to set the infusion pump before use D. Verify the flow rate against your computation While team effort is needed in the OR for efficient and quality patient care delivery, we should limit the number of people in the room for infection control. Who comprise this team? A. Surgeon, anesthesiologist, scrub nurse, radiologist, orderly B. Surgeon, assistants, scrub nurse, circulating nurse, anesthesiologist C. Surgeon, assistant surgeon, anesthesiologist, scrub nurse, pathologist D. Surgeon, assistant surgeon, anesthesiologist, intern, scrub nurse When surgery is on-going, who coordinates the activities outside, including the family?

A. Orderly/clerk B. Nurse Supervisor C. Circulating Nurse D. Anesthesiologist The breakdown in teamwork is often times a failure in: A. Electricity B. Inadequate supply C. Leg work D. Communication To prevent recurrent attacks on client with glomerulonephritis, the nurse instructs the client to: A. Take a shower instead of tub baths B. Avoid situations that involve physical activity C. Continue the same restriction on fluid intake D. Seek early treatment for respiratory infection When administering Tapazole, The nurse should monitor the client for which of the following adverse effect? A. Hyperthyroidism B. Hypothyroidism C. Drowsiness D. Seizure Post bronchoscopy, the nurse priority is to check which of the following before feeding? A. Gag reflex B. Wearing off of anesthesia C. Swallowing reflex D. Peristalsis Changes normally occur in the elderly. Among the following, which is a normal change in an elderly client? A. Increased sense of taste B. Increased appetite C. Urinary frequency D. Thinning of the lens

Colostomy is a surgically created anus. It can be temporary or permanent, depending on the disease condition.

Skin care around the stoma is critical. Which of the following is not indicated as a skin care barriers? A. Apply liberal amount of mineral oil to the area B. Use karaya paste and rings around the stoma C. Clean the area daily with soap and water before applying bag D. Apply talcum powder twice a day What health instruction will enhance regulation of a colostomy (defecation) of clients? A. Irrigate after lunch everyday B. Eat fruits and vegetables in all three meals C. Eat balanced meals at regular intervals D. Restrict exercise to walking only After ileostomy, which of the following condition is NOT expected? A. Increased weight B. Irritation of skin around the stoma C. Liquid stool D. Establishment of regular bowel movement The following are appropriate nursing interventions during colostomy irrigation, EXCEPT: A. Increase the irrigating solution flow rate when abdominal cramps is felt B. Insert 2-4 inches of an adequately lubricated catheter to the stoma C. Position client in semi-Fowler D. Hang the solution 18 inches above the stoma What sensation is used as a gauge so that patients with ileostomy can determine how often their pouch should be drained? A. Sensation of taste B. Sensation of pressure C. Sensation of smell D. Urge to defecate In performing a cleansing enema, the nurse performs the procedure by positioning the client in: A. Right lateral position B. Left lateral position C. Right sim’s position D. Left sim’s position Mang Caloy is scheduled to have a hemorrhoidectomy, after the operation, you would expect that

the client’s position post operatively will be: A. Knee chest position B. Side lying position C. Sims position D. Genopectoral position You would expect that after an abdominal perineal resection, the type of colostomy that will be use is? A. Double barrel colostomy B. Temporary colostomy C. Permanent colostomy D. An Ileostomy You are an ostomy nurse and you know that colostomy is defined as: A. It is an incision into the colon to create an artificial opening to the exterior of the abdomen B. It is end to end anastomosis of the gastric stump to the duodenum C. It is end to end anastomosis of the gastric stump to the jejunum D. It is an incision into the ileum to create an artificial opening to the exterior of the abdomen

Larry, 55 years old, who is suspected of having colorectal cancer, is admitted to the CI. After taking the history and vital signs the physician does which test as a screening test for colorectal cancer. A. Barium enema B. Carcinoembryonic antigen C. Annual digital rectal examination D. Proctosigmoidoscopy Symptoms associated with cancer of the colon include: A. constipation, ascites and mucus in the stool B. diarrhea, heart burn and eructation C. blood in the stools, anemia, and “pencil shaped” stools D. anorexia, hematemesis, and increased peristalasis

24 Hours after creation of colostomy, Nurse Violy is correct if she identify that the normal appearance of the stoma is : A. Pink, moist and slightly protruding from the abdomen B. Gray, moist and slightly protruding from the the abdomen C. Pink, dry and slightly protruding from the abdomen

D. Red, moist and slightly protruding from the abdomen

In cleaning the stoma, the nurse would use which of the following cleaning mediums? A. Hydrogen Peroxide, water and mild soap B. Providone Iodine, water and mild soap C. Alcohol, water and mild soap D. Mild soap and water When observing a return demonstration of a colostomy irrigation, you know that more teaching is required if pt: A. Lubricates the tip of the catheter prior to inserting into the stoma B. Hangs the irrigating bag on the bathroom door cloth hook during fluid insertion C. Discontinues the insertion of fluid after only 500 ml of fluid has been instilled D. Clamps of the flow of fluid when felling uncomfortable What does a sample group represent? A. Control group B. Study subjects C. General population D. Universe As a nurse, you can help improve the effectiveness of communication among healthcare givers by: A. Use of reminders of ‘what to do’ B. Using standardized list of abbreviations, acronyms, and symbols C. One-on-one oral endorsement D. Text messaging and e-mail Myxedema coma is a life threatening complication of long standing and untreated hypothyroidism with one of the following characteristics. A. Hyperglycemia B. Hypothermia C. Hyperthermia D. Hypoglycemia Mang Edgardo has a chest tube inserted in place after a Lobectomy. The nurse knows that that Chest tube after this procedure will: A. Prevents mediastinal shift B. Promote chest expansion of the remaining lung C. Drain fluids and blood accumulated post operatively

D. Remove the air in the lungs to promote lung expansion

Mrs. Pichay who is for thoracentesis is assigned by the nurse to any of the following positions, EXCEPT: A. straddling a chair with arms and head resting on the back of the chair B. lying on the unaffected side with the bed elevated 30-40 degrees C. lying prone with the head of the bed lowered 15-30 degrees D. sitting on the edge of the bed with her feet supported and arms and head on a padded overhead table Chest x-ray was ordered after thoracentesis. When your client asks what is the reason for another chest x-ray, you will explain: A. to rule out pneumothorax B. to rule out any possible perforation C. to decongest D. to rule out any foreign body The RR nurse should monitor for the most common postoperative complication of: A. hemorrhage B. endotracheal tube perforation C. osopharyngeal edema D. epiglottis The PACU nurse will maintain postoperative T and A client in what position? A. Supine with neck hyperextended and supported with pillow B. Prone with the head on pillow and turned to the side C. Semi-fowler’s with neck flexed D. Reverse trendelenburg with extended neck Tony is to be discharged in the afternoon of the same day after tonsillectomy and adenoidectomy. You as the RN will make sure that the family knows to: A. offer osterized feeding B. offer soft foods for a week to minimize discomfort while swallowing C. supplement his diet with Vitamin C rich juices to enhance healing D. offer clear liquid for 3 days to prevent irritation Situation – Rudy was diagnosed to have chronic renal failure. Hemodialysis is ordered so that an A-V shunt was surgically created. Which of the following action would be of highest priority with regards to the external shunt?

A. Avoid taking BP or blood sample from the arm with the shunt B. Instruct the client not to exercise the arm with the shunt C. Heparinize the shunt daily D. Change dressing of the shunt daily Diet therapy for Rudy, who has acute renal failure is low-protein, low potassium and low sodium. The nutrition instructions should include: A. Recommend protein of high biologic value like eggs, poultry and lean meats B. Encourage client to include raw cucumbers, carrot, cabbage, and tomatoes C. Allowing the client cheese, canned foods and other processed food D. Bananas, cantaloupe, orange and other fresh fruits can be included in the diet

The most common causative agent of Pyelonephritis in hospitalized patient attributed to prolonged catheterization is said to be: A. E. Coli B. Klebsiella C. Pseudomonas D. Staphylococcus The IVP reveals that Fe has small renal calculus that can be passed out spontaneously. To increase the chance of passing the stones, you instructed her to force fluids and do which of the following? A. Balanced diet B. Ambulate more C. Strain all urine D. Bed rest Sergio is brought to Emergency Room after the barbecue grill accident. Based on the assessment of the physician, Sergio sustained superficial partial thickness burns on his trunk, right upper extremities and right lower extremities. His wife asks what that means? Your most accurate response would be: A. Structures beneath the skin are damage B. Dermis is partially damaged C. Epidermis and dermis are both damaged D. Epidermis is damaged During the first 24 hours after the thermal injury, you should asses Sergio for: A. hypokalemia and hypernatremia B. hypokalemia and hyponatremia C. hyperkalemia and hyponatremia

D. hyperkalemia and hypernatremia

All of the following are instruction for proper foot care to be given to a client with peripheral vascular disease caused by Diabetes. Which is not? A. Trim nail using nail clipper B. Apply cornstarch to the foot C. Always check for the temperature of the water before bathing D. Use Canvas shoes You are on morning duty in the medical ward. You have 10 patients assigned to you. During your endorsement rounds, you found out that one of your patients was not in bed. The patient next to him informed you that he went home without notifying the nurses. Which among the following will you do first? A. Make an incident report B. Call security to report the incident C. Wait for 2 hours before reporting D. Report the incident to your supervisor You are on duty in the medical ward. You were asked to check the narcotics cabinet. You found out that what is on record does not tally with the drugs used. Which among the following will you do first? A. Write an incident report and refer the matter to the nursing director B. Keep your findings to yourself C. Report the matter to your supervisor D. Find out from the endorsement any patient who might have been given narcotics You are on duty in the medical ward. The mother of your patient who is also a nurse, came running to the nurses station and informed you that Fiolo went into cardiopulmonary arrest. A. Start basic life support measures B. Call for the Code C. Bring the crash cart to the room D. Go to see Fiolo and assess for airway patency and breathing problems

When observing a return demonstration of a colostomy irrigation, you know that more teaching is required if pt: E. Lubricates the tip of the catheter prior to inserting into the stoma F. Hangs the irrigating bag on the bathroom door cloth hook during fluid insertion G. Discontinues the insertion of fluid after only 500 ml of fluid has been instilled

H. Clamps of the flow of fluid when felling uncomfortable

Which of the four phases of emergency management is defined as “sustained action that reduces or eliminates long-term risk to people and property from natural hazards and their effects.”? A. Recovery B. Mitigation C. Response D. Preparedness Which of the following terms refer to a process by which the individual receives education about recognition of stress reaction and management strategies for handling stress which may be instituted after a disaster? A. Clinical incident stress management B. Follow-up C. Defriefing D. Defusion

Fires are approached using the mnemonic RACE, in which, R stands for: A. Run B. Race C. Rescue D. Remove

You are caring for Conrad who has a brained tumor and increased Intracranial Pressure (ICP). Which intervention should you include in your plan to reduce ICP? A. Administer bowel softener B. Position Conrad with his head turned toward the side of the tumor C. Provide sensory stimulation D. Encourage coughing and deep breathing Keeping Conrad’s head and neck alignment results in: A. increased inthrathoracic pressure B. increased venous outflow C. decreased venous outflow

D. increased intrabdominal pressure

Earliest sign of skin reaction to radiation therapy is: A. desquamation B. erythema C. atrophy D. pigmentation A guideline that is utilized in determining priorities is to asses the status of the following, EXCEPT: A. perfusion C. respiration B. locomotion D. mentation

Miss Kate is a bread vendor and you are buying a bread from her. You noticed that she receives and changes money and then hold the bread without washing her hand. As a nurse, What will you say to Miss Kate? A. Miss, Don’t touch the bread I’ll be the one to pick it up B. Miss, Please wash your hands before you pick up those breads C. Miss, Use a pick up forceps when picking up those breads D. Miss, Your hands are dirty I guess I’ll try another bread shop In administering blood transfusion, what needle gauge is used? A. 18 B. 22 C. 23 D. 24 Before administration of blood and blood products, the nurse should first: A. Check with another R.N the client’s name, Identification number, ABO and RH type. B. Explain the procedure to the client C. Assess baseline vital signs of the client D. Check for the BT order The only IV fluid compatible with blood products is: A. D5LR B. D5NSS C. NSS

D. Plain LR In any event of an adverse hemolytic reaction during blood transfusion, Nursing intervention should focus on: A. Slow the infusion, Call the physician and assess the patient B. Stop the infusion, Assess the client, Send the remaining blood to the laboratory and call the physician C. Stop the infusion, Call the physician and assess the client D. Slow the confusion and keep a patent IV line open for administration of medication The nurse knows that after receiving the blood from the blood bank, it should be administered within: A. 1 hour B. 2 hours C. 4 hours D. 6 hours During blood administration, the nurse should carefully monitor adverse reaction. To monitor this, it is essential for the nurse to: A. Stay with the client for the first 15 minutes of blood administration B. Stay with the client for the entire period of blood administration C. Run the infusion at a faster rate during the first 15 minutes D. Tell the client to notify the staff immediately for any adverse reaction

As Leda’s nurse, you plan to set up an emergency equipment at her beside following thyroidectomy. You should include: A. An airway and rebreathing tube B. A tracheostomy set and oxygen C. A crush cart with bed board D. Two ampules of sodium bicarbonate Which of the following nursing interventions is appropriate after a total thyroidectomy? A. Place pillows under your patient’s shoulders. B. Raise the knee-gatch to 30 degrees C. Keep you patient in a high-fowler’s position. D. Support the patient’s head and neck with pillows and sandbags. If there is an accidental injury to the parathyroid gland during a thyroidectomy which of the following might Leda develop postoperatively?

A. Cardiac arrest C. Respiratory failure B. Dyspnea D. Tetany

After surgery Leda develops peripheral numbness, tingling and muscle twitching and spasm. What would you anticipate to administer? A. Magnesium sulfate C. Potassium iodide B. Calcium gluconate D. Potassium chloride NURSES are involved in maintaining a safe and healthy environment. This is part of quality care management. The first step in decontamination is: A. to immediately apply a chemical decontamination foam to the area of contamination B. a through soap and water wash and rinse of the patient C. to immediately apply personal protective equipment D. removal of the patients clothing and jewelry and then rinsing the patient with water E. For a patient experiencing pruritus, you recommend which type of bath. A. water C. saline B. colloidal (oatmeal) D. sodium bicarbonate Induction of vomiting is indicated for the accidental poisoning patient who has ingested. A. rust remover C. toilet bowl cleaner B. gasoline D. aspirin

Because severe burn can affect the person’s totality it is important that-you apply interventions focusing on the various dimensions of man. You also have to understand the rationale of the treatment.

A client was rushed in the E.R showing a whitish, leathery and painless burned area on his skin. The nurse is correct in classifying this burn as: A. First degree burn B. Second degree burn C. Third degree burn E. Partial thickness burn

During the first 24 hours of burn, nursing measures should focus on which of the following? A. I and O hourly

B. Strict aseptic technique C. Forced oral fluids D. Isolate the patient

During the Acute phase of burn, the priority nursing intervention in caring for this client is: A. Prevention of infection B. Pain management C. Prevention of Bleeding D. Fluid Resuscitation The nurse knows that the most fatal electrolyte imbalance in burned client during the Emergent phase of burn is: A. Hypokalemia B. Hyperkalemia C. Hypernatremia D. Hyponatremia Hypokalemia is reflected in the ECG by which of the following? A. Tall T waves B. Widening QRS Complex C. Pathologic Q wave D. U wave Pain medications given to the burn clients are best given via what route? A. IV B. IM C. Oral D. SQ

What type of debridement involves proteolytic enzymes? A. Interventional C. Surgical B. Mechanical D. Chemical Which topical antimicrobial is most frequently used in burn wound care? A. Neosporin B. Silver nitrate C. Silver sulfadiazine D. Sulfamylon

Hypertrophic burn scars are caused by: A. exaggerated contraction B. random layering of collagen C. wound ischemia D. delayed epithelialization This study which is an in depth study of one boy is a: A. case study B. longitudinal study C. cross-sectional study D. evaluative study The process recording was the principal tool for data collection. Which of the following is NOT a part of a process recording? A. Non verbal narrative account B. Analysis and interpretation C. Audio-visual recording D. Verbal narrative account the most significant factor that might affect the nurse’s care for the psychiatric patient is: A. Nurse’s own beliefs and attitude about the mentally ill B. Amount of experience he has with psychiatric clients C. Her abilities and skill to care for the psychiatric clients D. Her knowledge in dealing with the psychiatric clients In order to establish a therapeutic relationship with the client, the nurse must first have: A. Self awareness B. Self understanding C. Self acceptance D. Self motivation Nurse Edna thinks that the patient is somewhat like his father. She then identifies positive feeling for the patient that affects the objectivity of her nursing care. This emotional reaction is called: A. Transference B. Counter Transference C. Reaction formation D. Sympathy the most important quality of a nurse during a Nurse-Patient interaction is:

A. Understanding B. Acceptance C. Listening D. Teaching Selective inattention is seen in what level of anxiety? A. Mild B. Moderate C. Severe D. Panic Obsessive compulsive disorder is characterized by: A. Uncontrollable impulse to perform an act or ritual repeatedly B. Persistent thoughts and behavior C. Recurring unwanted and disturbing thoughts D. Pathological persistence of unwilled thoughts Ms. Maria Salvacion says that she is the incarnation of the holy Virgin Mary. She said that she is the child of the covenant that would save this world from the evil forces of Satan. One morning, while caring for her, she stood in front of you and said “Bow down before me! I am the holy mother of Christ! I am the blessed Virgin Mary!” The best response by the Nurse is: A. Tell me more about being the Virgin Mary B. So, You are the Virgin Mary? C. Excuse me but, you are not anymore a Virgin so you cannot be the Blessed Virgin Mary. D. You are Maria Salvacion Maria’s statement “Bow down before me! I am the holy mother of Christ! I am the blessed Virgin Mary!” is an example of: A. Delusion of grandeur B. Visual Hallucination C. Religious delusion D. Auditory Hallcucination The nurse interprets the statement “Bow down before me! I am the holy mother of Christ! I am the blessed Virgin Mary!” as important in documenting in which of the following areas of mental status examination? A. Thought content B. Mood C. Affect D. Attitude

Mang David, A 27 year old psychiatric client was admitted with a diagnosis of schizophrenia. During the morning assessment, Mang David shouted “Did you know that I am the top salesman in the world? Different companies want me!” As a nurse, you know that this is an example of: A. Hallucination B. Delusion C. Confabulation D. Flight of Ideas The recommended treatment modality in clients with obsessive compulsive disorder is: A. Psychotherapy B. Behavior therapy C. Aversion therapy D. Psychoanalysis A state of disequilibrium wherein a person cannot readily solve a problem or situation even by using his usual coping mechanisms is called: A. Mental illness B. Mental health C. Crisis D. Stress Obsessive compulsive disorder is classified under: A. Psychotic disorders B. Neurotic disorders C. Major depressive disorder D. Bipolar disorder Which nursing diagnosis is a priority for clients with Borderline personality disorder? A. Risk for injury B. Ineffective individual coping C. Altered thought process D. Sensory perceptual alteration An appropriate nursing diagnosis for clients in the acute manic phase of bipolar disorder is: A. Risk for injury directed to self B. Risk for injury directed to others C. Impaired nutrition less than body requirements D. Ineffective individual coping

A paranoid client refuses to eat telling you that you poisoned his food. The best intervention to this client is: A. Taste the food in front of him and tell him that the food is not poisoned B. Offer other types of food until the client eats C. Simply state that the food is not poisoned D. Offer sealed foods Toilet training occurs in the anal stage of Freud’s psychosexual task. This is equivalent to Erikson’s: A. Trust vs. Mistrust B. Autonomy vs. Shame and Doubt C. Initiative vs. Guilt D. Industry vs. Inferiority During the phallic stage, the child must identify with the parent of: A. The same sex B. The opposite sex C. The mother or the primary caregiver D. Both sexes Ms. ANA had a car accident where he lost her boyfriend. As a result, she became passive and submissive. The nurse knows that the type of crisis Ms. ANA is experiencing is: A. Developmental crisis B. Maturational crisis C. Situational crisis D. Social Crisis Persons experiencing crisis becomes passive and submissive. As a nurse, you know that the best approach in crisis intervention is to be: A. Active and Directive B. Passive friendliness C. Active friendliness D. Firm kindness The psychosocial task of a 55 year old adult client is: A. Industry vs. Inferiority B. Intimacy vs. Isolation C. Integrity vs. Despair D. Generativity vs. Stagnation

The stages of grieving identified by Elizabeth Kubler-Ross are: A. Numbness, anger, resolution and reorganization B. Denial, anger, identification, depression and acceptance C. Anger, loneliness, depression and resolution D. Denial, anger, bargaining, depression and acceptance Which physiologic effect should the nurse expect in a client addicted to hallucinogens? A. Dilated pupils B. Constricted pupils C. Bradycardia D. Bradypnea Miss CEE is admitted for treatment of major depression. She is withdrawn, disheveled and states “Nobody wants me” The nurse most likely expects that Miss CEE is to be placed on: A. Neuroleptics medication B. Special diet C. Suicide precaution D. Anxiolytics medication In alcoholic patient, the nurse knows that the vitamin deficient to these types of clients that leads to psychoses is: A. Thiamine B. Vitamin C C. Niacin D. Vitamin A

Which of the following terms refers to weakness of both legs and the lower part of the trunk? A. Paraparesis B. Hemiplegia C. Quadriparesis D. Paraplegia

Of the following neurotransmitters, which demonstrates inhibitory action, helps control mood and sleep, and inhibits pain pathways? A. Serotonin B. Enkephalin C. Norepinephrine D. Acetylcholine

The lobe of the brain that contains the auditory receptive areas is the ____________ lobe. A. temporal B. frontal C. parietal D. occipital In preparation for ECT, the nurse knows that it is almost similar to that of: A. ECG B. General Anesthesia C. EEG D. MRI The expected side effect after ECT is commonly associated with: A. Transient loss of memory, confusion and disorientation B. Nausea and vomiting C. Fractures D. Hypertension and increased in cardiac rate The purpose of ECT in clients with depression is to: A. Stimulation in the brain to increase brain conduction and counteract depression B. Mainly Biologic, increasing the norepinephrine and serotonin level C. Creates a temporary brain damage that will increase blood flow to the brain D. Involves the conduction of electrical current to the brain to charge the neurons and combat depression

The priority nursing diagnosis for a client with major depression is: A. Altered nutrition B. Altered thought process C. Self care deficit D. Risk for injury A patient tells the nurse “I am depressed to talk to you, leave me alone” Which of the following response by the nurse is most therapeutic? A. I’ll be back in an hour B. Why are you so depressed? C. I’ll seat with you for a moment D. Call me when you feel like talking to me

One of the following statements is true with regards to the care of clients with depression: A. Only mentally ill persons commit suicide B. All depressed clients are considered potentially suicidal C. Most suicidal person gives no warning D. The chance of suicide lessens as depression lessens An adolescent client has bloodshot eyes, a voracious appetite and dry mouth. Which drug abuse would the nurse most likely suspect? A. Marijuana B. Amphetamines C. Barbiturates D. Anxiolytics

During which phase of therapeutic relationship should the nurse inform the patient for termination of therapy? A. Pre orientation B. Orientation C. Working D. Termination A client says to the nurse “I am worthless person, I should be dead” The nurse best replies: A. “Don’t say you are worthless, you are not a worthless person” B. “We are going to help you with your feelings” C. “What makes you feel you’re worthless?” D. “What you say is not true”

The nurse’s most unique tool in working with the emotionally ill client is his/her A. theoretical knowledge B. personality make up C. emotional reactions D. communication skills The mentally ill person responds positively to the nurse who is warm and caring. This is a demonstration of the nurse’s role as: A. counselor B. mother surrogate C. therapist D. socializing agent

The past history of Camila would most probably reveal that her premorbid personality is: A. schizoid B. extrovert C. ambivert D. cycloid In an extreme situation and when no other resident or intern is available, should a nurse receive telephone orders, the order has to be correctly written and signed by the physician within: A. 24 hours B. 36 hours C. 48 hours D. 12 hours . If it is established that the child is physically abused by a parent, the most important goal the nurse could formulate with the family is that: A. Child and any siblings will live in a safe environment B. Family will feel comfortable in their relationship with the counselor C. Family will gain understanding of their abusive behavior patterns D. Mother will be able to use verbal discipline with her children Cocaine is derived from the leaves of coca plant; the nurse knows that cocaine is classified as: A. Narcotic B. Stimulant C. Barbiturate D. Hallucinogen To successfully complete the tasks of older adulthood, an 85 year old who has been a widow for 25 years should be encouraged to: A. Invest her creative energies in promoting social welfare B. Redefine her role in the society and offer something and offer something of value C. Feel a sense of satisfaction in reflecting on her productive life D. Look to recapture the opportunities that were never started or completed In a therapeutic relationship, the nurse must understand own values, beliefs, feelings, prejudices & how these affect others. This is called: A. Therapeutic use of self B. Psychotherapy C. Therapeutic communication

D. Self awareness

While on Bryant’s traction, which of these observations of Graciela and her traction apparatus would indicate a decrease in the effectiveness of her traction? A. Graciela’s buttocks are resting on the bed. B. The traction weights are hanging 10 inches above the floor. C. Graciela’s legs are suspended at a 90 degree angle to her trunk. D. The traction ropes move freely through the pulley. The nurse notes that the fall might also cause a possible head injury. She will be observed for signs of increased intracranial pressure which include: A. Narrowing of the pulse pressure B. Vomiting C. Periorbital edema D. A positive Kernig’s sign

This is a tricyclic antidepressant drug: A. Venlafaxine (Effexor) B. Flouxetine (Prozac) C. Sertraline (Zoloft) D. Imipramine (Tofranil) The working phase in a therapy group is usually characterized by which of the following? A. Caution B. Cohesiveness C. Confusion D. Competition

Substance abuse is different from substance dependence in that, substance dependence: A. includes characteristics of adverse consequences and repeated use B. requires long term treatment in a hospital based program C. produces less severe symptoms than that of abuse D. includes characteristics of tolerance and withdrawal

Ricky’s IQ falls within the range of 50-55. he can be expected to: A. Profit from vocational training with moderate supervision

B. Live successfully in the community C. Perform simple tasks in closely supervised settings D. Acquire academic skills of 6th grade level The mother of a drug dependent would never consider referring her son to a drug rehabilitation agency because she fears her son might just become worse while relating with other drug users. The mother’s behavior can be described as: A. Unhelpful B. Codependent C. Caretaking D. Supportive You teach your clients the difference between, Type I (IDDM) and Type II (NDDM) diabetes. Which of the following is true? A. both types diabetes mellitus clients are all prone to developing ketosis B. Type II (NIDDM) is more common and is also preventable compared to Type I (IDDM) diabetes which is genetic in etiology C. Type I (IIDM) is characterized by fasting hyperglycemia D. Type II (NIDDM) is characterized by abnormal immune response Lifestyle-related diseases in general share areas common risk factors. These are the following except: A. physical activity B. smoking C. genetics D. nutrition The following mechanisms can be utilized as part of the quality assurance program of your hospital EXCEPT: A. Patient satisfaction surveys B. Peer review to assess care provided C. Review of clinical records of care of client D. Use of Nursing Interventions Classification The use of the Standards of Nursing Practice is important in the hospital. Which of the following statements best describes what it is? A. These are statements that describe the maximum or highest level of acceptable performance in nursing practice B. It refers to the scope of nursing practice as defined in Republic Act 9173 C. It is a license issued by the Professional Regulation Commission to protect the public from substandard nursing practice

D. The Standards of Care includes the various steps of the nursing process and the standards of professional performance you are taking care of critically ill client and the doctor in charge calls to order a DNR (do not resuscitate) for the client. Which of the following is the appropriate action when getting DNR order over the phone? A. Have the registered nurse, family spokesperson, nurse supervisor and doctor sign B. Have 2 nurse validate the phone order, both nurses sign the order and the doctor should sign his order within 24 hours C. Have the registered nurse, family and doctor sign the order D. Have 1 nurse take the order and sign it and have the doctor sign it within 24 hours Under the PRC-Board of Nursing Resolution promulgating the adoption of a Nursing Specialty Certification Program and Council, which two (2) of the following serves as the strongest for its enforcement? (a) Advances made in Science and Technology have provided the climate for specialization in almost all aspects of human endeavor; and (b) As necessary consequence, there has emerged a new concept known as globalization which seeks to remove barriers in trade, industry and services imposed by the national laws of countries all over the world; and (c) Awareness of this development should impel the nursing sector to prepare our people in the services sector to meet the above challenge; and (d) Current trends of specialization in nursing practice recognized by the International Council of Nurses (ICN) of which the Philippines is a member for the benefit of the Filipino in terms of deepening and refining nursing practice and enhancing the quality of nursing care. A. b & c are strong justifications B. a & b are strong justifications C. a & c are strong justifications D. a & d are strong justifications Knowing that for a comatose patient hearing is the last sense to be lost, as Judy’s nurse, what should you do? A. Tell her family that probably she can’t hear them B. Talk loudly so that Wendy can hear you C. Tell her family who are in the room not to talk D. Speak softly then hold her hands gently Which among the following interventions should you consider as the highest priority when caring for June who has hemiparesis secondary to stroke? A. Place June on an upright lateral position B. Perform range of motion exercises

C. Apply antiembolic stockings D. Use hand rolls or pillows for support Salome was fitted a hearing aid. She understood the proper use and wear of this device when she says that the battery should be functional, the device is turned on and adjusted to a: A. therapeutic level B. comfortable level C. prescribed level D. audible level Membership dropout generally occurs in group therapy after a member: A. Accomplishes his goal in joining the group B. Discovers that his feelings are shared by the group members C. Experiences feelings of frustration in the group D. Discusses personal concerns with group members Which of the following questions illustrates the group role of encourager? A. What were you saying? B. Who wants to respond next? C. Where do you go from here? D. Why haven’t we heard from you? The goal of remotivation therapy is to facilitate: A. Insight B. Productivity C. Socialization D. Intimacy

Being in contact with reality and the environment is a function of the: A. conscience B. ego C. id D. super ego Substance abuse is different from substance dependence in that, substance dependence: E. includes characteristics of adverse consequences and repeated use F. requires long term treatment in a hospital based program G. produces less severe symptoms than that of abuse H. includes characteristics of tolerance and withdrawal

During the detoxification stage, it is a priority for the nurse to: A. teach skills to recognize and respond to health threatening situations B. increase the client’s awareness of unsatisfactory protective behaviors C. implement behavior modification D. promote homeostasis and minimize the client’s withdrawal symptoms Commonly known as “shabu” is: A. Cannabis Sativa B. Lysergic acid diethylamide C. Methylenedioxy methamphetamine D. Methampetamine hydrochloride

Wednesday, November 21, 2007 diannemaydee : ANSWER KEY 300 items reviewer

Situation – Richard has a nursing diagnosis of ineffective airway clearance related to excessive secretions and is at risk for infection because of retained secretions. Part of Nurse Mario’s nursing care plan is to loosen and remove excessive secretions in the airway. Mario listens to Richard’s bilateral sounds and finds that congestion is in the upper lobes of the

lungs. The appropriate position to drain the anterior and posterior apical segments of the lungs when Mario does percussion would be: B. Client seated upright in bed or on a chair then leaning forward in sitting position then flat on his back and on his abdomen When documenting outcome of Richard’s treatment Mario should include the following in his recording EXCEPT: C. Amount of fluid intake of client before and after the procedure When assessing Richard for chest percussion or chest vibration and postural drainage, Mario would focus on the following EXCEPT:

C. Teaching the client’s relatives to perform the procedure Mario prepares Richard for postural drainage and percussion. Which of the following is a special consideration when doing the procedure? B. Client can tolerate sitting and lying positions The purpose of chest percussion and vibration is to loosen secretions in the lungs. The difference between the procedures is:

B. Percussion delivers cushioned blows to the chest with cupped palms while vibration gently shakes secretion loose on the exhalation cycle Situation – A 61 year old man, Mr. Regalado, is admitted to the private ward for observation after complaints of severe chest pain. You are assigned to take care of the client. When doing an initial assessment, the best way for you to identify the client’s priority problem is to: D. Do a physical examination while asking the client relevant questions Nancy blames God for her situation. She is easily provoked to tears and wants to be left alone, refusing to eat or talk to her family. A religious person before, she now refuses to pray or go to church stating that God has abandoned her. The nurse understands that Nancy is grieving for her self and is in the stage of: C. anger Which of the following ethical principles refers to the duty to do good?

A. Beneficence During which step of the nursing process does the nurse analyze data related to the patient's health status? A. Assessment The basic difference between nursing diagnoses and collaborative problems is that A. nurses manage collaborative problems using physician-prescribed interventions. Situation – Mrs. Seva, 52 years old, asks you about possible problems regarding her elimination now that she is in the menopausal stage. Instruction on health promotion regarding urinary elimination is important. Which would you include? A. Hold urine as long as she can before emptying the bladder to strengthen her sphincter muscles Mrs. Seva also tells the nurse that she is often constipated. Because she is aging, what physical changes predispose her to constipation? C. loss of tone of the smooth muscles of the colon The nurse understands that one of these factors contributes to constipation: D. prolonged use of laxatives

You will do nasopharyngeal suctioning on Mr. Abad. Your guide for the length of insertion of the tubing for an adult would be: C. the distance from the tip of the nose to the tip of the ear lobe situation– Mr. Dizon, 84 years old, brought to the Emergency Room for complaint of hypertension, flushed face, severe headache, and nausea. You are doing the initial assessment of vital signs. You are to measure the client’s initial blood pressure reading by doing all of the following EXCEPT: C. Pump the cuff to around 50 mmHg above the point where the pulse is obliterated A pulse oximeter is attached to Mr. Dizon’s finger to: D. Detect oxygen saturation of arterial blood before symptoms of hypoxemia develops

In which type of shock does the patient experiences a mismatch of blood flow to the cells? A. Distributive

The preferred route of administration of medication in the most acute care situations is which of the following routes? A. Intravenous

After a few hours in the Emergency Room, Mr. Dizon is admitted to the ward with an order of hourly monitoring of blood pressure. The nurse finds that the cuff is too narrow and this will cause the blood pressure reading to be: C. higher than what the reading should be Through the client’s health history, you gather that Mr. Dizon smokes and drinks coffee. When taking the blood pressure of a client who recently smoked or drank coffee, how long should the nurse wait before taking the client’s blood pressure for accurate reading? B. 30 minutes While the client has pulse oximeter on his fingertip, you notice that the sunlight is shining on the area where the oximeter is. Your action will be to: C. Cover the fingertip sensor with a towel or bedsheet When taking blood pressure reading the cuff should be: D. inflated to 30 mmHg above the estimated systolic BP based on palpation of radial or bronchial artery -this is a typo error of brachial artery

To ensure client safety before starting blood transfusions the following are needed before the procedure can be done EXCEPT: B. blood should be warmed to room temperature for 30 minutes before blood transfusions is administered Mr. Bruno asks what the “normal” allowable salt intake is. Your best response to Mr. Bruno is:

D. 1 tsp of salt/day but no patis and toyo Which of the following methods is the best method for determining nasogastric tube placement in the stomach? A. X-ray Which of the following is the most important risk factor for development of Chronic Obstructive Pulmonary Disease? A. Cigarette smoking

When performing endotracheal suctioning, the nurse applies suctioning while withdrawing and gently rotating the catheter 360 degrees for which of the following time periods? A. 10-15 seconds The nurse auscultates the apex beat at which of the following anatomical locations? A. Fifth intercostal space, midclavicular line Which of the following terms describes the amount of blood ejected per heartbeat? A. Stroke volume

You are to apply a transdermal patch of nitoglycerin to your client. The following are important guidelines to observe EXCEPT:

B. Patches may be applied to distal part of the extremities like forearm The GAUGE size in ET tubes determines:

B. The internal diameter of the tube The nurse is correct in performing suctioning when she applies the suction intermittently during: B. Withdrawing of the suction catheter The purpose of the cuff in Tracheostomy tube is to: A. Separate the upper and lower airway the placement of the tube

Which priority nursing diagnosis is applicable for a patient with indwelling urinary catheter? D. Risk for infection An incontinent elderly client frequently wets his bed and eventually develop redness and skin excoriation at the perianal area. The best nursing goal for this client is to: D. Keep the patient clean and dry

As a Nurse Manager, DMLM enjoys her staff of talented and self motivated individuals. She knew that the leadership style to suit the needs of this kind of people is called:

D. Laissez Faire A fire has broken in the unit of DMLM R.N. The best leadership style suited in cases of emergencies like this is: A. Autocratic

Which step of the management process is concerned with Policy making and Stating the goals and objective of the institution? A. Planning In the management process, the periodic checking of the results of action to make sure that it coincides with the goal of the institution is termed as:

B. Evaluating The Vision of a certain agency is usually based on their beliefs, Ideals and Values that directs the organization. It gives the organization a sense of purpose. The belief, Ideals and Values of this Agency is called: A. Philosophy Mr. CKK is unconscious and was brought to the E.R. Who among the following can give consent for CKK’s Operation? C. Next of Kin Mang Carlos has been terminally ill for 5 years. He asked his wife to decide for him when he is

no longer capable to do so. As a Nurse, You know that this is called: D. Durable Power of Attorney Mang Carlos has a standing DNR order. He then suddenly stopped breathing and you are at his bedside. You would: B. Stay with Mang Carlos and Do nothing It is not a legally binding document but nevertheless, Very important in caring for the patients. B. Patient’s Bill of Rights

In monitoring the patient in PACU, the nurse correctly identify that checking the patient’s vital signs is done every: C. 15 minutes

diannemayde R.N is conducting a research on her unit about the effects of effective nurse-patient communication in decreasing anxiety of post operative patients. Which of the following step in nursing research should she do next? A. Review of related literature Before diannemaydee perform the formal research study, what do you call the pre testing, small scale trial run to determine the effectiveness of data collection and methodological problem that might be encountered? E. Pilot Study On the study “effects of effective nurse-patient communication in decreasing anxiety of post operative patients” What is the Independent variable? A. Effective Nurse-patient communication On the study “effects of effective nurse-patient communication in decreasing anxiety of post operative patients” What is the Dependent variable? C. Anxiety level

In the recent technological innovations, which of the following describe researches that are made to improve and make human life easier?

C. Applied research Which of the following is not true about a Pure Experimental research? D. There is a careful selection of subjects in the experimental group When Mrs. Guevarra, a nurse, delegates aspects of the clients care to the nurse-aide who is an unlicensed staff, Mrs. Guevarra B. is assigning the responsibility to the aide but not the accountability for those tasks

Process of formal negotiations of working conditions between a group of registered nurses and employer is C. collective bargaining You are attending a certification on cardiopulmonary resuscitation (CPR) offered and required by the hospital employing you. This is B. inservice education The law which regulated the practice of nursing profession in the Philippines is: A. R.A 9173 This quality is being demonstrated by a Nurse who raise the side rails of a confuse and disoriented patient? C. Prudence

Nurse Joel and Ana is helping a 16 year old Nursing Student in a case filed against the student. The case was frustrated homicide. Nurse Joel and Ana are aware of the different circumstances of crimes. They are correct in identifying which of the following Circumstances that will be best applied in this case? C. Mitigating In signing the consent form, the nurse is aware that what is being observed as an ethical consideration is the patient’s A. Autonomy Why is there an ethical dilemma?

C. Because the patient’s right coincide with the nurse’s responsibility Who among the following can work as a practicing nurse in the Philippines without taking the Licensure examination? D. Those involved in medical mission who’s services are for free

In signing the consent form, the nurse is aware that what is being observed as an ethical consideration is the patient’s A. Autonomy Nurse Buddy gave Inapsine instead of Insulin to a patient in severe hyperglycemia. He reported the incident as soon as he knew there was an error. A nurse that is always ready to answer for all his actions and decision is said to be: A. Accountable

Which of the following best describes Primary Nursing? C. A registered nurse is responsible for a group of patients from admission to discharge

The best and most effective method in times of staff and financial shortage is: A. Functional Method You are doing bed bath to the client when suddenly, The nursing assistant rushed to the room and tell you that the client from the other room was in Pain. The best intervention in such case is: A. Raise the side rails, cover the client and put the call bell within reach and then attend to the client in pain to give the PRN medication Angie is a disoriented client who frequently falls from the bed. As her nurse, which of the following is the best nursing intervention to prevent future falls? D. Put the bed in the lowest position ever When injecting subcutaneous injection in an obese patient, It should be angled at around: B. 90 ° The following statements are all true about Z-Track technique except:

C. It forces the medication to be contained at the subcutaneous tissues Communication is best undertaken if barriers are first removed. Considering this statement, which of the following is considered as deterrent factor in communication? A. Not universally accepted abbreviations Nurse DMLM is correct in identifying the correct sequence of events during abdominal assessment if she identifies which of the following? A. Inspection, Auscultation, Percussion, Palpation To prevent injury and strain on the muscles, the nurse should observe proper body mechanics. Among the following, which is a principle of proper body mechanics? A. Broaden the space between the feet In taking the client’s blood pressure, the nurse should position the client’s arm: A. At the level of the heart What principle is used when the client with fever loses heat through giving cooling bed bath to lower body temperature? D. Conduction The most effective way in limiting the number of microorganism in the hospital is: D. Handwashing The immunoglobulin of the mother that crosses the placenta to protect the child is an example of:

B. Natural passive immunity Richard is a subject of a research lead by his doctor. The nurse knows that all of the following is a correct understanding as his right as a research subject except: C. I must choose another doctor if I withdrew from this research

Which of the following is a normal finding during assessment of a Chest tube in a 3 way bottle system? C. The water fluctuates during inhalation of the patient

In obtaining a urine specimen for culture and sensitivity on a catheterized patient, the nurse is correct if: B. Alcoholize the self sealing port, obtain a sterile syringe and draw the specimen on the self sealing port Which of the following is an example of secondary prevention? B. Screening patients for hypertension

Which of the following is a form of primary prevention? D. Immunization An abnormal condition in which a person must sit, stand or use multiple pillows when lying down is: A. Orthopnea As a nurse assigned for care for geriatric patients, you need to frequently assess your patient using the nursing process. Which of the following needs be considered with the highest priority? D. Physiologic needs that are life threatening The component that should receive the highest priority before physical examination is the: A. Psychological preparation of the client Legally, Patients chart are: C. Owned by the hospital and should not be given to anyone who request it other than the doctor in charge

Which of the following categories identifies the focus of community/public health nursing practice? A. Promoting and maintaining the health of populations and preventing and minimizing the progress of disease A major goal for home care nurses is A. restoring maximum health function.

A written nursing care plan is a tool that: B. Gives quality nursing care Gina, A client in prolong labor said she cannot go on anymore. The health care team decided that both the child and the mother cannot anymore endure the process. The baby is premature and has a little chance of surviving. Caesarian section is not possible because Gina already lost enough blood during labor and additional losses would tend to be fatal. The husband decided that Gina should survive and gave his consent to terminate the fetus. The principle that will be used by the health care team is: D. Double effect Situation – There are various developments in health education that the nurse should know about: The provision of health information in the rural areas nationwide through television and radio programs and video conferencing is referred to as: B. Telehealth program In teaching the sister of a diabetic client about the proper use of a glucometer in determining the blood sugar level of the client, The nurse is focusing in which domain of learning according to bloom? C. Psychomotor A nearby community provides blood pressure screening, height and weight measurement, smoking cessation classes and aerobics class services. This type of program is referred to as D. wellness program After cleaning the abrasions and applying antiseptic, the nurse applies cold compress to the swollen ankle as ordered by the physician. This statement shows that the nurse has correct understanding of the use of cold compress: C. Cold compress prevents edema and reduces pain After receiving prescription for pain medication, Ronnie is instructed to continue applying 30 minute cold at home and start 30 minute hot compress the next day. You explain that the use of hot compress: C. Increase oxygenation to the injured tissues for better healing Situation – A nursing professor assigns a group of students to do data gathering by interviewing their classmates as subjects.

She instructed the interviewees not to tell the interviewees that the data gathered are for her own research project for publication. This teacher has violated the student’s right to:

B. Disclosure

Before the nurse researcher starts her study, she analyzes how much time, money, materials and people she will need to complete the research project. This analysis prior to beginning the study is called: B. Feasibility Data analysis is to be done and the nurse researcher wants to include variability. These include the following EXCEPT: D. Mean Nurse Minette needs to schedule a first home visit to OB client Leah. When is a first home-care visit typically made? B. Within 24 hours after discharge By force of law, therefore, the PRC-Board of Nursing released Resolution No. 14 Series of 1999 entitled: “Adoption of a Nursing Specialty Certification Program and Creation of Nursing Specialty Certification Council.” This rule-making power is called: C. Quasi-Legislative Power Anita is performing BSE and she stands in front of the Mirror. The rationale for standing in front of the mirror is to check for: C. The Size and Contour of the breast An emerging technique in screening for Breast Cancer in developing countries like the Philippines is: C. BSE Once a month Transmission of HIV from an infected individual to another person occurs: A. Most frequently in nurses with needlesticks After a vaginal examination, the nurse determines that the client’s fetus is in an occiput posterior position. The nurse would anticipate that the client will have:

B. Intense back pain

The rationales for using a prostaglandin gel for a client prior to the induction of labor is to: A. Soften and efface the cervix

Nurse Lorena is a Family Planning and Infertility Nurse Specialist and currently attends to FAMILY PLANNING CLIENTS AND INFERTILE COUPLES. The following conditions pertain to meeting the nursing needs of this particular population group. Dina, 17 years old, asks you how a tubal ligation prevents pregnancy. Which would be the best answer? C. Sperm can no longer reach the ova, because the fallopian tubes are blocked The Dators are a couple undergoing testing for infertility. Infertility is said to exist when:

C. a couple has been trying to conceive for 1 year

The correct temperature to store vaccines in a refrigerator is: B. between 2 deg C and +8 deg C Which of the following vaccines is not done by intramuscular (IM) injection? A. Measles vaccine This vaccine content is derived from RNA recombinants. C. Hepatitis B vaccines

This special form is used when the patient is admitted to the unit. The nurse completes the information in this record particularly his/her basic personal data, current illness, previous health history, health history of the family, emotional profile, environmental history as well as physical assessment together with nursing diagnosis on admission. What do you call this record? B. Nursing Health History and Assessment Worksheet These are sheets/forms which provide an efficient and time saving way to record information that must be obtained repeatedly at regular and/or short intervals of time. This does not replace the progress notes; instead this record of information on vital signs, intake and output, treatment,

postoperative care, post partum care, and diabetic regimen, etc. This is used whenever specific measurements or observations are needed to be documented repeatedly. What is this? B. Graphic Flow Sheets These records show all medications and treatment provided on a repeated basis. What do you call this record? D. Medicine and Treatment Record This flip-over card is usually kept in a portable file at the Nurse’s Station. It has 2-parts: the activity and treatment section and a nursing care plan section. This carries information about basic demographic data, primary medical diagnosis, current orders of the physician to be carried out by the nurse, written nursing care plan, nursing orders, scheduled tests and procedures, safety precautions in patient care and factors related to daily living activities. This record is used in the charge-of-shift reports or during the bedside rounds or walking rounds. What record is this? D. Nursing Kardex Most nurses regard this conventional recording of the date, time, and mode by which the patient leaves a healthcare unit but this record includes importantly, directs of planning for discharge that starts soon after the person is admitted to a healthcare institution. It is accepted that collaboration or multidisciplinary involvement (of all members of the health team) in discharge results in comprehensive care. What do you call this? A. Discharge Summary Based on the Code of Ethics for Filipino Nurses, what is regarded as the hallmark of nursing responsibility and accountability? D. Accurate documentation of actions and outcomes

A nurse should be cognizant that professional programs for specialty certification by the Board of Nursing accredited through the: B. Nursing Specialty Certification Council Integrated management for childhood illness is the universal protocol of care endorsed by WHO and is use by different countries of the world including the Philippines. In any case that the nurse classifies the child and categorized the signs and symptoms in PINK category, You know that this means: A. Urgent referral You know that fast breathing of a child age 13 months is observed if the RR is more than:

A. 40 Angelo, An 8 month old child is brought to the health care facility with sunken eyes. You pinch his skin and it goes back very slowly. In what classification of dehydration will you categorize Angelo? C. Severe Dehydration

In responding to the care concerns of children with severe disease, referral to the hospital is of the essence especially if the child manifests which of the following? D. Difficulty to awaken A child with ear problem should be assessed for the following, EXCEPT: A. is there any fever? If the child does not have ear problem, using IMCI, what should you as the nurse do? D. Go to the next question, check for malnutrition

All of the following are treatment for a child classified with no dehydration except: A. 1,000 ml to 1,400 ml be given within 4 hours An ear infection that persists but still less than 14 days is classified as: C. Acute Ear Infection

If a child has two or more pink signs, you would classify the child as having: very severe disease The nurse knows that the most common complication of Measles is: A Pneumonia and larynigotracheitis

A client scheduled for hysterosalpingography needs health teaching before the procedure. The nurse is correct in telling the patient that: D. Flushing sensation is felt as the dye in injected

In a population of 9,500. What is your estimate of the population of pregnant woman needing tetanus toxoid vaccination?

D. 332.5

All of the following are seen in a child with measles. Which one is not? C. Pustule Mobilizing the people to become aware of their own problem and to do actions to solve it is called: A. Community Organizing

Prevention of work related accidents in factories and industries are responsibilities of which field of nursing? C. Occupational health nursing

In one of your home visit to Mr. JUN, you found out that his son is sick with cholera. There is a great possibility that other member of the family will also get cholera. This possibility is a/an: B. Health threat

Why is bleeding in the leg of a pregnant woman considered as an emergency? D. The pressure of the gravid uterus will exert additional force thus, increasing the blood loss in the lower extremities Aling Maria is nearing menopause. She is habitually taking cola and coffee for the past 20 years. You should tell Aling Maria to avoid taking caffeinated beverages because: C. It will contribute to additional bone demineralization All of the following are contraindication when giving Immunization except: B. BCG Vaccine can be given to a child with Hepatitis B Theresa, a mother with a 2 year old daughter asks, “at what age can I be able to take the blood

pressure of my daughter as a routine procedure since hypertension is common in the family?” Your answer to this is: C. When she’s 3 years old Baby John develops hyperbilirubinemia. What is a method used to treat hyperbilirubinemia in a newborn? D. Early feeding to speed passage of meconium The community/Public Health Bag is: B. an essential and indispensable equipment of the community health nurse What is the rationale in the use of bag technique during home visits? B. It saves time and effort of the nurse in the performance of nursing procedures In consideration of the steps in applying the bag technique, which side of the paper lining of the CHN bag is considered clean to make a non-contaminated work area? D. The inside surface

How many words does a typical 12-month-old infant use? D. Two, plus “mama” and “papa”

During operation, The OR suite’s lighting, noise, temperature and other factors that affects the operation are managed by whom? C. Circulating nurse Before and after the operation, the operating suite is managed by the: C. Nurse Manager

The counting of sponges is done by the Surgeon together with the: B. Scrub nurse

The OR team performs distinct roles for one surgical procedure to be accomplished within a prescribed time frame and deliver a standard patient outcome. While the surgeon performs the

surgical procedure, who monitors the status of the client like urine output, blood loss? C. Anaesthesiologist Surgery schedules are communicated to the OR usually a day prior to the procedure by the nurse of the floor or ward where the patient is confined. For orthopedic cases, what department is usually informed to be present in the OR? D. Radiology department In some hip surgeries, an epidural catheter for Fentanyl epidural analgesia is given. What is your nursing priority care in such a case? D. Assess respiratory rate carefully The patient’s medical record can work as a double edged sword. When can the medical record become the doctor’s/nurse’s worst enemy? D. When the medical record is inaccurate, incomplete, and inadequate Disposal of medical records in government hospitals/institutions must be done in close coordination with what agency? D. Department of Health (DOH) In the hospital, when you need the medical record of a discharged patient for research you will request permission through: D. Medical records section

You will give health instructions to Carlo, a case of bronchial asthma. The health instruction will include the following, EXCEPT: D. Practice respiratory isolation As the head nurse in the OR, how can you improve the effectiveness of clinical alarm systems? D. Implement a regular maintenance and testing of alarm systems

Overdosage of medication or anesthetic can happen even with the aid of technology like infusion pumps, sphygmomanometer and similar devices/machines. As a staff, how can you improve the safety of using infusion pumps? A. Check the functionality of the pump before use

While team effort is needed in the OR for efficient and quality patient care delivery, we should limit the number of people in the room for infection control. Who comprise this team? B. Surgeon, assistants, scrub nurse, circulating nurse, anesthesiologist When surgery is on-going, who coordinates the activities outside, including the family? B. Nurse Supervisor The breakdown in teamwork is often times a failure in: D. Communication To prevent recurrent attacks on client with glomerulonephritis, the nurse instructs the client to: D. Seek early treatment for respiratory infection When administering Tapazole, The nurse should monitor the client for which of the following adverse effect? B. Hypothyroidism Post bronchoscopy, the nurse priority is to check which of the following before feeding? A. Gag reflex

Changes normally occur in the elderly. Among the following, which is a normal change in an elderly client? C. Urinary frequency

Colostomy is a surgically created anus. It can be temporary or permanent, depending on the disease condition.

Skin care around the stoma is critical. Which of the following is not indicated as a skin care barriers? A. Apply liberal amount of mineral oil to the area What health instruction will enhance regulation of a colostomy (defecation) of clients? C. Eat balanced meals at regular intervals

After ileostomy, which of the following condition is NOT expected? A. Increased weight The following are appropriate nursing interventions during colostomy irrigation, EXCEPT: A. Increase the irrigating solution flow rate when abdominal cramps is felt What sensation is used as a gauge so that patients with ileostomy can determine how often their pouch should be drained?

B. Sensation of pressure

In performing a cleansing enema, the nurse performs the procedure by positioning the client in: D. Left sim’s position Mang Caloy is scheduled to have a hemorrhoidectomy, after the operation, you would expect that the client’s position post operatively will be: B. Side lying position

You would expect that after an abdominal perineal resection, the type of colostomy that will be use is? C. Permanent colostomy

You are an ostomy nurse and you know that colostomy is defined as: A. It is an incision into the colon to create an artificial opening to the exterior of the abdomen

Larry, 55 years old, who is suspected of having colorectal cancer, is admitted to the CI. After taking the history and vital signs the physician does which test as a screening test for colorectal cancer. D. Proctosigmoidoscopy Symptoms associated with cancer of the colon include:

C. blood in the stools, anemia, and “pencil shaped” stools

24 Hours after creation of colostomy, Nurse Violy is correct if she identify that the normal appearance of the stoma is : D. Red, moist and slightly protruding from the abdomen

In cleaning the stoma, the nurse would use which of the following cleaning mediums? D. Mild soap and water When observing a return demonstration of a colostomy irrigation, you know that more teaching is required if pt:

B. Hangs the irrigating bag on the bathroom door cloth hook during fluid insertion What does a sample group represent? C. General population As a nurse, you can help improve the effectiveness of communication among healthcare givers by: B. Using standardized list of abbreviations, acronyms, and symbols

Myxedema coma is a life threatening complication of long standing and untreated hypothyroidism with one of the following characteristics. B. Hypothermia

Mang Edgardo has a chest tube inserted in place after a Lobectomy. The nurse knows that that Chest tube after this procedure will: C. Drain fluids and blood accumulated post operatively

Mrs. Pichay who is for thoracentesis is assigned by the nurse to any of the following positions, EXCEPT: C. lying prone with the head of the bed lowered 15-30 degrees

Chest x-ray was ordered after thoracentesis. When your client asks what is the reason for another chest x-ray, you will explain: A. to rule out pneumothorax The RR nurse should monitor for the most common postoperative complication of: A. hemorrhage

The PACU nurse will maintain postoperative T and A client in what position?

B. Prone with the head on pillow and turned to the side

Tony is to be discharged in the afternoon of the same day after tonsillectomy and adenoidectomy. You as the RN will make sure that the family knows to: B. offer soft foods for a week to minimize discomfort while swallowing Situation – Rudy was diagnosed to have chronic renal failure. Hemodialysis is ordered so that an A-V shunt was surgically created. Which of the following action would be of highest priority with regards to the external shunt? A. Avoid taking BP or blood sample from the arm with the shunt Diet therapy for Rudy, who has acute renal failure is low-protein, low potassium and low sodium. The nutrition instructions should include: A. Recommend protein of high biologic value like eggs, poultry and lean meats The most common causative agent of Pyelonephritis in hospitalized patient attributed to prolonged catheterization is said to be: A. E. Coli

The IVP reveals that Fe has small renal calculus that can be passed out spontaneously. To increase the chance of passing the stones, you instructed her to force fluids and do which of the following? B. ambulate more Sergio is brought to Emergency Room after the barbecue grill accident. Based on the assessment of the physician, Sergio sustained superficial partial thickness burns on his trunk, right upper

extremities and right lower extremities. His wife asks what that means? Your most accurate response would be: C. Epidermis and dermis are both damaged

During the first 24 hours after the thermal injury, you should asses Sergio for: C. hyperkalemia and hyponatremia

All of the following are instruction for proper foot care to be given to a client with peripheral vascular disease caused by Diabetes. Which is not? D. Use Canvas shoes You are on morning duty in the medical ward. You have 10 patients assigned to you. During your endorsement rounds, you found out that one of your patients was not in bed. The patient next to him informed you that he went home without notifying the nurses. Which among the following will you do first? D. Report the incident to your supervisor You are on duty in the medical ward. You were asked to check the narcotics cabinet. You found out that what is on record does not tally with the drugs used. Which among the following will you do first? C. Report the matter to your supervisor

You are on duty in the medical ward. The mother of your patient who is also a nurse, came running to the nurses station and informed you that Fiolo went into cardiopulmonary arrest. C. Bring the crash cart to the room

When observing a return demonstration of a colostomy irrigation, you know that more teaching is required if pt: F. Hangs the irrigating bag on the bathroom door cloth hook during fluid insertion Which of the four phases of emergency management is defined as “sustained action that reduces or eliminates long-term risk to people and property from natural hazards and their effects.”?

B. Mitigation

Which of the following terms refer to a process by which the individual receives education about recognition of stress reaction and management strategies for handling stress which may be instituted after a disaster? D. Defusion Fires are approached using the mnemonic RACE, in which, R stands for: C. Rescue You are caring for Conrad who has a brained tumor and increased Intracranial Pressure (ICP). Which intervention should you include in your plan to reduce ICP? A. Administer bowel softener Keeping Conrad’s head and neck alignment results in: B. increased venous outflow Earliest sign of skin reaction to radiation therapy is: B. erythema A guideline that is utilized in determining priorities is to asses the status of the following, EXCEPT: D. mentation Miss Kate is a bread vendor and you are buying a bread from her. You noticed that she receives and changes money and then hold the bread without washing her hand. As a nurse, What will you say to Miss Kate? C. Miss, Use a pick up forceps when picking up those breads In administering blood transfusion, what needle gauge is used? A. 18 Before administration of blood and blood products, the nurse should first: A. Check with another R.N the client’s name, Identification number, ABO and RH type. The only IV fluid compatible with blood products is: C. NSS

In any event of an adverse hemolytic reaction during blood transfusion, Nursing intervention should focus on: B. Stop the infusion, Assess the client, Send the remaining blood to the laboratory and call the physician

The nurse knows that after receiving the blood from the blood bank, it should be administered within: C. 4 hours

During blood administration, the nurse should carefully monitor adverse reaction. To monitor this, it is essential for the nurse to: A. Stay with the client for the first 15 minutes of blood administration As Leda’s nurse, you plan to set up an emergency equipment at her beside following thyroidectomy. You should include: B. A tracheostomy set and oxygen Which of the following nursing interventions is appropriate after a total thyroidectomy? D. Support the patient’s head and neck with pillows and sandbags. If there is an accidental injury to the parathyroid gland during a thyroidectomy which of the following might Leda develop postoperatively? D. Tetany

After surgery Leda develops peripheral numbness, tingling and muscle twitching and spasm. What would you anticipate to administer?

B. Calcium gluconate NURSES are involved in maintaining a safe and healthy environment. This is part of quality care management. The first step in decontamination is:

D. removal of the patients clothing and jewelry and then rinsing the patient with water For a patient experiencing pruritus, you recommend which type of bath. B. colloidal (oatmeal) Induction of vomiting is indicated for the accidental poisoning patient who has ingested.

D. aspirin

Because severe burn can affect the person’s totality it is important that-you apply interventions focusing on the various dimensions of man. You also have to understand the rationale of the treatment.

A client was rushed in the E.R showing a whitish, leathery and painless burned area on his skin. The nurse is correct in classifying this burn as: C. Third degree burn

During the first 24 hours of burn, nursing measures should focus on which of the following? A. I and O hourly During the Acute phase of burn, the priority nursing intervention in caring for this client is: D. Fluid Resuscitation The nurse knows that the most fatal electrolyte imbalance in burned client during the Emergent phase of burn is:

B. Hyperkalemia

Hypokalemia is reflected in the ECG by which of the following? D. U wave Pain medications given to the burn clients are best given via what route? A. IV

What type of debridement involves proteolytic enzymes? D. Chemical Which topical antimicrobial is most frequently used in burn wound care? D. Sulfamylon Hypertrophic burn scars are caused by:

B. random layering of collagen This study which is an in depth study of one boy is a: A. case study The process recording was the principal tool for data collection. Which of the following is NOT a part of a process recording? C. Audio-visual recording

the most significant factor that might affect the nurse’s care for the psychiatric patient is: A. Nurse’s own beliefs and attitude about the mentally ill In order to establish a therapeutic relationship with the client, the nurse must first have: A. Self awareness

Nurse Edna thinks that the patient is somewhat like his father. She then identifies positive feeling for the patient that affects the objectivity of her nursing care. This emotional reaction is called: B. Counter Transference the most important quality of a nurse during a Nurse-Patient interaction is: C. Listening Selective inattention is seen in what level of anxiety? B. Moderate

Obsessive compulsive disorder is characterized by: B. Persistent thoughts and behavior

Ms. Maria Salvacion says that she is the incarnation of the holy Virgin Mary. She said that she is the child of the covenant that would save this world from the evil forces of Satan. One morning, while caring for her, she stood in front of you and said “Bow down before me! I am the holy mother of Christ! I am the blessed Virgin Mary!” The best response by the Nurse is: D. You are Maria Salvacion Maria’s statement “Bow down before me! I am the holy mother of Christ! I am the blessed Virgin Mary!” is an example of: C. Religious delusion

The nurse interprets the statement “Bow down before me! I am the holy mother of Christ! I am the blessed Virgin Mary!” as important in documenting in which of the following areas of mental status examination? A. Thought content

Mang David, A 27 year old psychiatric client was admitted with a diagnosis of schizophrenia. During the morning assessment, Mang David shouted “Did you know that I am the top salesman in the world? Different companies want me!” As a nurse, you know that this is an example of: B. Delusion The recommended treatment modality in clients with obsessive compulsive disorder is: A. Psychotherapy A state of disequilibrium wherein a person cannot readily solve a problem or situation even by using his usual coping mechanisms is called: C. Crisis Obsessive compulsive disorder is classified under: B. Neurotic disorders

Which nursing diagnosis is a priority for clients with Borderline personality disorder? A. Risk for injury An appropriate nursing diagnosis for clients in the acute manic phase of bipolar disorder is: B. Risk for injury directed to others A paranoid client refuses to eat telling you that you poisoned his food. The best intervention to this client is: D. Offer sealed foods Toilet training occurs in the anal stage of Freud’s psychosexual task. This is equivalent to Erikson’s: B. Autonomy vs. Shame and Doubt

During the phallic stage, the child must identify with the parent of: A. The same sex Ms. ANA had a car accident where he lost her boyfriend. As a result, she became passive and submissive. The nurse knows that the type of crisis Ms. ANA is experiencing is: C. Situational crisis

Persons experiencing crisis becomes passive and submissive. As a nurse, you know that the best approach in crisis intervention is to be: A. Active and Directive

The psychosocial task of a 55 year old adult client is: D. Generativity vs. Stagnation The stages of grieving identified by Elizabeth Kubler-Ross are: A. Numbness, anger, resolution and reorganization

Which physiologic effect should the nurse expect in a client addicted to hallucinogens?

A. Dilated pupils cardia D. Bradypnea Miss CEE is admitted for treatment of major depression. She is withdrawn, disheveled and states “Nobody wants me” The nurse most likely expects that Miss CEE is to be placed on: C. Suicide precaution

In alcoholic patient, the nurse knows that the vitamin deficient to these types of clients that leads to psychoses is: A. Thiamine Which of the following terms refers to weakness of both legs and the lower part of the trunk? A. Paraparesis Of the following neurotransmitters, which demonstrates inhibitory action, helps control mood and sleep, and inhibits pain pathways? A. Serotonin The lobe of the brain that contains the auditory receptive areas is the ____________ lobe. A. temporal In preparation for ECT, the nurse knows that it is almost similar to that of: B. General Anesthesia

The expected side effect after ECT is commonly associated with: A. Transient loss of memory, confusion and disorientation The purpose of ECT in clients with depression is to: B. Mainly Biologic, increasing the norepinephrine and serotonin level The priority nursing diagnosis for a client with major depression is: D. Risk for injury

A patient tells the nurse “I am depressed to talk to you, leave me alone” Which of the following response by the nurse is most therapeutic? A. I’ll be back in an hour One of the following statements is true with regards to the care of clients with depression: B. All depressed clients are considered potentially suicidal An adolescent client has bloodshot eyes, a voracious appetite and dry mouth. Which drug abuse would the nurse most likely suspect? A. Marijuana During which phase of therapeutic relationship should the nurse inform the patient for termination of therapy? B. Orientation A client says to the nurse “I am worthless person, I should be dead” The nurse best replies: C. “What makes you feel you’re worthless?”

The nurse’s most unique tool in working with the emotionally ill client is his/her D. communication skills The mentally ill person responds positively to the nurse who is warm and caring. This is a demonstration of the nurse’s role as: B. mother surrogate The past history of Camila would most probably reveal that her premorbid personality is: A. schizoid In an extreme situation and when no other resident or intern is available, should a nurse receive telephone orders, the order has to be correctly written and signed by the physician within: A. 24 hours If it is established that the child is physically abused by a parent, the most important goal the nurse could formulate with the family is that: A. Child and any siblings will live in a safe environment

Cocaine is derived from the leaves of coca plant; the nurse knows that cocaine is classified as: A. Narcotic

To successfully complete the tasks of older adulthood, an 85 year old who has been a widow for 25 years should be encouraged to: C. Feel a sense of satisfaction in reflecting on her productive life

In a therapeutic relationship, the nurse must understand own values, beliefs, feelings, prejudices & how these affect others. This is called: D. Self awareness

While on Bryant’s traction, which of these observations of Graciela and her traction apparatus would indicate a decrease in the effectiveness of her traction? A. Graciela’s buttocks are resting on the bed. The nurse notes that the fall might also cause a possible head injury. She will be observed for signs of increased intracranial pressure which include: B. Vomiting This is a tricyclic antidepressant drug: D. Imipramine (Tofranil) The working phase in a therapy group is usually characterized by which of the following? B. Cohesiveness

Substance abuse is different from substance dependence in that, substance dependence: D. includes characteristics of tolerance and withdrawal

Ricky’s IQ falls within the range of 50-55. he can be expected to: D. Acquire academic skills of 6th grade level

The mother of a drug dependent would never consider referring her son to a drug rehabilitation agency because she fears her son might just become worse while relating with other drug users. The mother’s behavior can be described as: B. Codependent You teach your clients the difference between, Type I (IDDM) and Type II (NDDM) diabetes. Which of the following is true? B. Type II (NIDDM) is more common and is also preventable compared to Type I (IDDM) diabetes which is genetic in etiology Lifestyle-related diseases in general share areas common risk factors. These are the following except: C. genetics

The following mechanisms can be utilized as part of the quality assurance program of your hospital EXCEPT: D. Use of Nursing Interventions Classification The use of the Standards of Nursing Practice is important in the hospital. Which of the following statements best describes what it is? D. The Standards of Care includes the various steps of the nursing process and the standards of professional performance you are taking care of critically ill client and the doctor in charge calls to order a DNR (do not resuscitate) for the client. Which of the following is the appropriate action when getting DNR order over the phone? B. Have 2 nurse validate the phone order, both nurses sign the order and the doctor should sign his order within 24 hours Under the PRC-Board of Nursing Resolution promulgating the adoption of a Nursing Specialty Certification Program and Council, which two (2) of the following serves as the strongest for its enforcement? (a) Advances made in Science and Technology have provided the climate for specialization in almost all aspects of human endeavor; and (b) As necessary consequence, there has emerged a new concept known as globalization which seeks to remove barriers in trade, industry and services imposed by the national laws of countries all over the world; and

(c) Awareness of this development should impel the nursing sector to prepare our people in the services sector to meet the above challenge; and (d) Current trends of specialization in nursing practice recognized by the International Council of Nurses (ICN) of which the Philippines is a member for the benefit of the Filipino in terms of deepening and refining nursing practice and enhancing the quality of nursing care.

B. a & b are strong justifications

Knowing that for a comatose patient hearing is the last sense to be lost, as Judy’s nurse, what should you do? D. Speak softly then hold her hands gently Which among the following interventions should you consider as the highest priority when caring for June who has hemiparesis secondary to stroke? B. Perform range of motion exercises Salome was fitted a hearing aid. She understood the proper use and wear of this device when she says that the battery should be functional, the device is turned on and adjusted to a: D. audible level Membership dropout generally occurs in group therapy after a member: C. Experiences feelings of frustration in the group Which of the following questions illustrates the group role of encourager?

B. Who wants to respond next? The goal of remotivation therapy is to facilitate: A. Insight Being in contact with reality and the environment is a function of the: B. ego Substance abuse is different from substance dependence in that, substance dependence: H. includes characteristics of tolerance and withdrawal

During the detoxification stage, it is a priority for the nurse to: D. promote homeostasis and minimize the client’s withdrawal symptoms Commonly known as “shabu” is: D. Methampetamine hydrochloride Posted by diannemaydee at 8:13 PM 4 comments

read read read and study the following!!! * all that has been discussed and stressed by sir Jungco, ung PA-lmr lec nya, no need to sweat it out reading Venzon * my 300 items reviewer, lalo na ung side lectures and notes q during our rationalization * the MS reviewer posted here, don't you ever skip reading the rationales! lalo na if you don't have the CDs * the fatima exclusive material and the last ten tips for last june posted at http://pinoybsn.blogspot.com/ * all medication administration procedure na naka-table sa Fundamentals of Nursing by Kozier * Appendix C of the PHN book * Sir V's OR nursing and "special lec" * all that i stressed and wrote in bold letters during our rationalization * love ma'am Mallari promise! * all notes na makukuha nyo during the final coaching! don't be absent, wag nyo q gayahin pls lang! * percussion and vibration Kozier * Udan MS all practice test and end of the chapter questions * Makasaysayang RESEARCH METHODOLOGY MANUAL of olfu * Pilliteri pages 570-590 * mga appendix at glossary of Venzon PA * new immunization table * page 215-216 Venzon Management 3rd ed * Ung likod ng Souvenir chuva namin * Tables of child development sa pilliteri, tonsillectomy and contraindications to immunization, ung 2 pages na fine print * Operating Room technique Instructional Manual 1st ed pages 41-55 * My JCAHO, pain and nle CBQs handout

Thursday, October 18, 2007 StuffedNurse: MEDICAL SURGICAL BOARD EXAM REVIEWER guys these are almost the same as what was in the june board and same concepts as the june 06 and dec 06..... answers are the first ones after the questions Source : Smeltzer and Bare/ Brunner and Suddarth CD 1. According to Maslow, which of the following categories of needs represents the most basic? Physiologic needs Physiologic needs must be met before an individual is able to move toward psychological health and well-being.

Self-actualization Self-actualization is the highest level of need Safety and security needs Safety and security needs, while lower level, are not essential to physiologic survival. Belongingness Belongingness and affection needs are not essential to physiologic survival.

2. Which of the following statements reflects the World Health Organization’s definition of health? A state of complete physical, mental, and social well-being and not merely the absence of disease and infirmity. Such a definition, however, does not allow for any variations in the degrees of wellness or illness. A condition of homeostatis and adaptation. The WHO definition addresses physical, mental, and social dimensions of being. An individual’s location along a wellness--illness continuum. The concept of a health--illness continuum allows for a greater range in describing a person’s health than the definition provided by the WHO. A fluid, ever-changing balance reflected through physical, mental, and social behavior. The WHO definition does not allow for any variations in the degrees of wellness and illness.

3. Which of the following statements defines culture? The learned patterns of behavior, beliefs, and values that can be attributed to a particular group of people. Included among characteristics that distinguish cultural groups are manner of dress, values, artifacts, and health beliefs and practices. A group of people distinguished by genetically transmitted material. A group of people distinguished by genetically transmitted material describes the term race. The status of belonging to a particular region by origin, birth, or naturalization. The status of belonging to a particular region by origin, birth, or naturalization describes the term nationality. The classification of a group based upon certain distinctive characteristics. The classification of a group based upon certain distinctive characteristics describes the term

ethnicity.

4. The reason that case management has gained such prominence in health care can be traced to decreased cost of care associated with inpatient stay. The reasons case management has gained such prominence can be traced to the decreased cost of care associated with decreased length of hospital stay, coupled with rapid and frequent inter-unit transfers from specialty to standard care units. increased length of hospital stay. In general, length of hospital stay has decreased over the past 5 years. discharge from specialty care units to home. In general, patients are transferred from specialty care units to standard care units at least 24 hours prior to discharge. limited availability for inter-unit hospital transfers. In general, patients in acute care hospitals undergo frequent inter-unit transfers from specialty to standard care units.

5. A preferred provider organization is described as a business arrangement between hospitals and physicians. PPO’s usually contract to provide health care to subscribers, usually businesses, for a negotiated fee that often is discounted. prepaid group health practice system. A prepaid group health practice system is termed a health maintenance organization. limited insurance program. Insurance is a cost payment system of shared risk, not a health care delivery system. health care savings account program. A health care savings account program is an incentive program to consumers, not a health care delivery system.

6. Which of the following categories identifies the focus of community/public health nursing practice? Promoting and maintaining the health of populations and preventing and minimizing the progress

of disease Although nursing interventions used by public health nurses might involve individuals, families, or small groups, the central focus remains promoting health and preventing disease in the entire community. Rehabilitation and restorative services Rehabilitation and restorative services are the focus of extended care facilities and home care nursing. Adaptation of hospital care to the home environment Adaptation of hospital care to the home environment is the focus of home nursing. Hospice care delivery Hospice care delivery refers to the delivery of services to the terminally ill.

7. A major goal for home care nurses is restoring maximum health function. Tertiary preventive nursing care, focusing on rehabilitation and restoring maximum health function, is a goal for home care nurses. promoting the health of populations. Promoting the health of populations is a focus of community/public health nursing. minimizing the progress of disease. Minimizing the progress of disease is a focus of community/public health nursing. maintaining the health of populations. Maintaining the health of populations is a focus of community/public health nursing.

8. In the United States, nurses performing invasive procedures need to be up-to-date with their immunizations, particularly hepatitis B. Hepatitis B is transmitted through contact with infected blood or plasma. hepatitis E. Hepatitis E is found mainly in underdeveloped countries with substandard sanitation and water quality. hepatitis A. hepatitis A is transmitted through the oral route from the feces and saliva of an infected person.

hepatitis C. At present, immunization against hepatitis C is not available.

9. At what time during a patient’s hospital stay does discharge planning begin? Admission To prepare for early discharge and the possible need for follow-up in the home, discharge planning begins with the patient’s admission. Twenty-four hours prior to discharge Discharge planning requires identification of patient needs and anticipatory guidance and is not relegated to a specific time for beginning. The shift prior to discharge Discharge planning requires communication with and cooperation of the patient, family, and health care team and is not relegated to a specific time for beginning. By the third hospital day Discharge planning may require involvement of personnel and agencies in the planning process and is not relegated to a specific day of hospital stay.

10. The leading health problems of elementary school children include cancer. The leading health problems of elementary school children are injuries, infections, malnutrition, dental disease, and cancer. alcohol and drug abuse. Alcohol and drug abuse are leading health problems for high school students. mental and emotional problems. Mental and emotional problems are leading health problems for high school students. homicide. Homicide is a leading health problem for high school children.

11. Which skill needed by the nurse to think critically involves identification of patient problems indicated by data?

Analysis Analysis is used to identify patient problems indicated by data. Interpretation Interpretation is used to determine the significance of data that is gathered. Inferencing Inferences are used by the nurse to draw conclusions. Explanation Explanation is the justification of actions or interventions used to address patient problems and to help a patient move toward desired outcomes.

12. The ethics theory that focuses on ends or consequences of actions is the utilitarian theory. Utilitarian theory is based on the concept of the greatest good for the greatest number. formalist theory. Formalist theory argues that moral standards exist independently of the ends or consequences. deontological theory. Deontological theory argues that moral standards exist independently of the ends or consequences. adaptation theory. Adaptation theory is not an ethics theory.

13. Which of the following ethical principles refers to the duty to do good? Beneficence Beneficence is the duty to do good and the active promotion of benevolent acts. Fidelity Fidelity refers to the duty to be faithful to one's commitments. Veracity Veracity is the obligation to tell the truth. Nonmaleficence Nonmaleficence is the duty not to inflict, as well as to prevent and remove, harm; it is more binding than beneficence.

14. During which step of the nursing process does the nurse analyze data related to the patient's health status? Assessment Analysis of data is included as part of the assessment. Implementation Implementation is the actualization of the plan of care through nursing interventions. Diagnosis Diagnosis is the identification of patient problems. Evaluation Evaluation is the determination of the patient's responses to the nursing interventions and the extent to which the outcomes have been achieved.

15. The basic difference between nursing diagnoses and collaborative problems is that nurses manage collaborative problems using physician-prescribed interventions. Collaborative problems are physiologic complications that nurses monitor to detect onset or changes and manage through the use of physician-prescribed and nursing-prescribed interventions to minimize the complications of events. collaborative problems can be managed by independent nursing interventions. Collaborative problems require both nursing and physician-prescribed interventions. nursing diagnoses incorporate physician-prescribed interventions. Nursing diagnoses can be managed by independent nursing interventions. nursing diagnoses incorporate physiologic complications that nurses monitor to detect change in status. Nursing diagnoses refer to actual or potential health problems that can be managed by independent nursing interventions.

16. Health education of the patient by the nurse is an independent function of nursing practice. Health education is an independent function of nursing practice and is included in all state nurse practice acts.

requires a physician's order. Teaching, as a function of nursing, is included in all state nurse practice acts. must be approved by the physician. Health education is a primary responsibility of the nursing profession. must focus on wellness issues. Health education by the nurse focuses on promoting, maintaining, and restoring health; preventing illness; and assisting people to adapt to the residual effects of illness.

17. Nonadherence to therapeutic regimens is a significant problem for which of the following age groups? Adults 65 and over Elderly people frequently have one or more chronic illnesses that are managed with numerous medications and complicated by periodic acute episodes, making adherence difficult. Teenagers Problems of teenagers, generally, are time limited and specific, and require promoting adherence to treatment to return to health. Children In general, the compliance of children depends on the compliance of their parents. Middle-aged adults Middle-aged adults, in general, have fewer health problems, thus promoting adherence.

18. Experiential readiness to learn refers to the patient's past history with education and life experience. Experiential readiness refers to past experiences that influence a person's ability to learn. emotional status. Emotional readiness refers to the patient's acceptance of an existing illness or the threat of an illness and its influence on the ability to learn. acceptance of an existing illness. Emotional readiness refers to the patient's acceptance of an existing illness or the threat of an illness and its influence on the ability to learn. ability to focus attention.

Physical readiness refers to the patient's ability to cope with physical problems and focus attention upon learning.

19. Asking the patient questions to determine if the person understands the health teaching provided would be included during which step of the nursing process? Evaluation Evaluation includes observing the person, asking questions, and comparing the patient's behavioral responses with the expected outcomes. Assessment Assessment includes determining the patient's readiness regarding learning. Planning and goals Planning includes identification of teaching strategies and writing the teaching plan. Implementation Implementation is the step during which the teaching plan is put into action.

20. Which of the following items is considered the single most important factor in assisting the health professional in arriving at a diagnosis or determining the person's needs? History of present illness The history of the present illness is the single most important factor in assisting the health professional in arriving at a diagnosis or determining the person's needs. Physical examination The physical examination is helpful but often only validates the information obtained from the history. Diagnostic test results Diagnostic test results can be helpful, but they often only verify rather than establish the diagnosis. Biographical data Biographical information puts the health history in context but does not focus the diagnosis.

21. Of the following areas for assessing the patient profile, which should be addressed after the others?

Body image The patient is often less anxious when the interview progresses from information that is less personal to information that is more personal. Education Educational level is relatively impersonal and readily revealed by the patient. Occupation Occupation is relatively impersonal and readily revealed by the patient. Environment Housing, religion, and language are relatively impersonal and readily revealed by the patient.

22. Which of the following methods of physical examination refers to the translation of physical force into sound? Percussion Percussion translates the application of physical force into sound. Palpation Palpation refers to examination by non-forceful touching. Auscultation Auscultation refers to the skill of listening to sounds produced within the body created by movement of air or fluid. Manipulation Manipulation refers to the use of the hands to determine motion of a body part.

23. In which range of body mass index (BMI) are patients considered to have increased risk for problems associated with poor nutritional status? Below 24 Additionally, higher mortality rates in hospitalized patients and community-dwelling elderly are associated with individuals who have low BMI. 25-29 Those who have a BMI of 25 to 29 are considered overweight. 30 to 39 Those who have BMI of 30-39 are considered obese.

Over 40 Those who have BMI over 40 are considered extremely obese.

24. To calculate the ideal body weight for a woman, the nurse allows 100 pounds for 5 feet of height. To calculate the ideal body weight of a woman, the nurse allows 100 pounds for 5 feet of height and adds 5 pounds for each additional inch over 5 feet 106 pounds for 5 feet of height. The nurse allows 106 pounds for 5 feet of height in calculating the ideal body weight for a man. 6 pounds for each additional inch over 5 feet. The nurse adds 6 pounds for each additional inch over 5 feet in calculating the ideal body weight for a man. 80 pounds for 5 feet of height. Eighty pounds for 5 feet of height is too little.

25. A steady state within the body is termed homeostasis. When a change occurs that causes a body function to deviate from its stable range, processes are initiated to restore and maintain the steady state or homeostasis. constancy. Constancy refers to the balanced internal state of the human body maintained by physiologic and biochemical processes. adaptation. Adaptation refers to a constant, ongoing process that requires change in structure, function, or behavior so that the person is better suited to the environment. stress. Stress refers to a state produced by a change in the environment that is perceived as challenging, threatening, or damaging to the person's dynamic balance or equilibrium.

26. Which of the following terms, according to Lazarus, refers to the process through which an event is evaluated with respect to what is at stake and what might and can be done?

Cognitive appraisal The outcome of cognitive appraisal is identification of the situation as either stressful or nonstressful. Coping Coping consists of both cognitive and behavioral efforts made to manage the specific external or internal demand that taxes a person's resources. Hardiness Hardiness is a personality characteristic that is composed of control, commitment, and challenge. Adaptation Lazarus believed adaptation was affected by emotion that subsumed stress and coping.

27. An increase in the number of new cells in an organ or tissue that is reversible when the stimulus for production of new cells is removed is termed hyperplasia. Hyperplasia occurs as cells multiply and are subjected to increased stimulation resulting in tissue mass enlargement. hypertrophy. Hypertrophy is an increase in size and bulk of tissue that does not result from an increased number of cells. atrophy. Atrophy refers to reduction in size of a structure after having come to full maturity. neoplasia. With neoplasia, the increase in the number of new cells in an organ or tissue continues after the stimulus is removed.

28. Which of the following types of cells have a latent ability to regenerate? Stable Stable cells have a latent ability to regenerate if they are damaged or destroyed and are found in the kidney, liver, and pancreas, among other body organs. Labile Labile cells multiply constantly to replace cells worn out by normal physiologic processes. Permanent

Permanent cells include neurons --- the nerve cell bodies, not their axons. Destruction of a neuron causes permanent loss, but axons may regenerate. Epithelial Epithelial cells are a type of labile cell that multiply constantly to replace cells worn out by normal physiologic processes.

29. The relaxation techniques of progressive muscle relaxation, relaxation with guided imagery, and the Benson Relaxation Response share which of the following elements? A mental device (something on which to focus the attention) Similar elements also include a quiet environment, a comfortable position, and a passive attitude. Nutritional foundation Relaxation techniques do not encompass specific nutritional guidelines. Analgesic preparation Relaxation techniques are used to reduce one's response to stress and do not require analgesia prior to practicing the techniques. Physician's order A physician's order is not required to assist an individual to learn techniques to reduce one's response to stress.

30. Which of the following terms has been defined by the American Psychiatric Association as a group of behavioral or psychological symptoms or a pattern that manifests itself in significant distress, impaired functioning, or accentuated risk of enduring severe suffering or possible death? Mental disorder The definition was adopted by the American Psychiatric Association in 1994. Emotional disorder There is no universally accepted definition of what constitutes an emotional disorder. Anxiety Anxiety is defined as fear of the unknown. Schizophrenia Schizophrenia is a specific disorder characterized by psychosis.

31.

Establishing financial security has been identified as a developmental task of which of the following groups? Middle adult The middle adult's tasks also include launching children, and refocusing on one's marital relationship. Older adult The older adult's tasks include adapting to retirement and declining physical stamina. Young adult The young adult's tasks include establishing a lifestyle and independence. Teenager The teenager's primary developmental tasks include developing an identity and intimacy.

32. When up to a 6-month period elapses between the experience of trauma and the onset of symptoms of posttraumatic stress disorder (PTSD), the episode is termed delayed. In the case of delayed PTSD, there may be up to a 6-month period of time that elapses between the trauma and the manifestation of symptoms. acute. Acute PTSD is defined as the experience of symptoms for less than a 3-month period. chronic. Chronic PTSD is defined as the experience of symptoms lasting longer than 3 months. primary. The concept of primary disease is not used in relation to PTSD.

33. Which of the following statements accurately describes a risk factor for depression? History of physical or sexual abuse History of physical or sexual abuse and current substance abuse are risk factors for depression. Male gender A risk factor for depression is female gender. Age over 50 years A risk factor for depression is onset before 40 years.

Negative family history of depression Family history of depression is a risk factor.

34. Of the following stages of grieving as described by Kubler-Ross, which is the initial? Denial The stages include: denial, anger, bargaining, depression, and acceptance. Anger Anger is the second stage of the process. Bargaining Bargaining is the third stage of the process. Depression Depression is the fourth stage of the process.

35. Which of the following terms refers to Leininger's description of the learned and transmitted knowledge about values, beliefs, rules of behavior, and lifestyle practices that guide a designated group in their thinking and actions in patterned ways? Culture Leininger was the founder of the specialty called transcultural nursing and advocated culturally competent nursing care. Minority Minority refers to a group of people whose physical or cultural characteristics differ from the majority of people in a society. Race Race refers to a group of people distinguished by genetically transmitted characteristics. Subculture Subculture refers to a group that functions within a culture.

36. The inability of a person to recognize his or her own values, beliefs, and practices and those of others because of strong ethnocentric tendencies is termed cultural blindness.

Cultural blindness results in bias and stereotyping. acculturation. Acculturation is the process by which members of a culture adapt or learn how to take on the behaviors of another group. cultural imposition. Cultural imposition is the tendency to impose one's cultural beliefs, values, and patterns of behavior on a person from a different culture. cultural taboo. Cultural taboos are those activities governed by rules of behavior that are avoided, forbidden, or prohibited by a particular cultural group.

37. Which of the following groups of individuals may stare at the floor during conversations as a sign of respect? Native Americans Some Native Americans stare at the floor during conversations, conveying respect and indicating that the listener is paying close attention to the speaker. Indo-Chinese The Indo-Chinese may consider direct eye contact impolite or aggressive. Arabs Arabs may consider direct eye contact impolite or aggressive. Asians Asians may consider direct eye contact impolite or aggressive.

38. For which of the following religious groups is all meat prohibited? Hinduism Hinduism prohibits consumption of all meats and animal shortening. Seventh-Day Adventism Seventh-Day Adventism prohibits consumption of pork. Judaism Judaism prohibits consumption of pork. Islam

Islam prohibits the consumption of pork and animal shortening.

39. The paradigm that explains the cause of illness as an imbalance in the forces of nature is the holistic perspective. The naturalist or holistic perspective believes that health exists when all aspects of a persona are in perfect balance or harmony. magico-religious view. The magico-religious view holds that illness is caused by forces of evil. biomedical view. The biomedical view holds life events as cause and effect and incorporates the bacterial or viral explanation of communicable disease. scientific view. The scientific view holds life events as cause and effect and incorporates the bacterial or viral explanation of communicable disease.

40. The aim of genomic medicine is improving predictions about individuals’ susceptibility to diseases Predictions regarding the time of their onset, their extent and eventual severity as well as which treatments or medications are likely to be most effective or harmful are the focus of genomic medicine. reproduction The focus of genomic medicine is broader than the reproduction of cells. cure of disease The focus of genomic medicine is broader than the cure of disease. cloning Genomic medicine is gene-based health care. 41. Nondisjunction of a chromosome results in which of the following diagnoses? Down Syndrome When a pair of chromosomes fails to separate completely and creates a sperm or oocyte that contains two copies of a particular chromosome (nondisjunction) Down syndrome results from three number 21 chromosomes.

Huntingon Disease Huntington disease is one example of a germ-line mutation. Duchenne Muscular Dystrophy Duchenne muscular dystrophy, an inherited form of muscular dystrophy, is an example of a genetic caused by structural gene mutations. Marphan Syndrome Marphan Syndrome is a genetic condition that may occur in a single family member as a result of spontaneous mutation.

42. Which type of Mendelian inherited condition results in both genders being affected equally in a vertical pattern? Automosomal dominant inheritance An individual who has an autosomal dominant inherited condition carries a gene mutation for that condition on one chromosome of a pair. Automosomal recessive inheritance The pattern of inheritance in autosomal recessive inherited conditions is different from that of autosomal dominant inherited conditions in that it is more horizontal than vertical, with relatives of a single generation tending to have the condition. X-linked inheritance X-linked conditions may be inherited in families in recessive or dominant patterns. In both, the gene mutation is located on the X-chromosome. All males inherit an X chromosome from their mother with no counterpart; hence, all males express the gene mutation. Multifactorial genetic inheritance Neural tube defects, such as spina bifida and anencephaly, are examples of multifactorial genetic conditions. The majority of neural tube defects are caused by both genetic and environmental influences that combine during early embryonic development leading to incomplete closure of the neural tube.

43. A specific BRCA1 cancer-predisposing gene mutation seems to occur more frequently among women of which descent? Ashkanazi Jewish Expression of the BRCA1 gene is an example of inheritance in the development of breast cancer. Mediterranean

Glucose-6-phosphate dehydrogenase deficiency (G6PD) is a common enzyme abnormality that affects millions of people throughout the world, especially those of Mediterranean, South East Asian, African, Middle Eastern, and Near Eastern origin. African American Sickle cell anemia is associated with the African-American population. Chinese and Japanese Individuals of Chinese and Japanese descent who are rapid metabolizers of the enzyme Nacetyltransferase and who are prescribed the drug isoniazid (as part of treatment for tuberculosis) are at significantly increased risk for developing isoniazid-induced hepatitis.

44. Which of the following statements describes accurate information related to chronic illness? Most people with chronic conditions do not consider themselves sick or ill. Although some people take on a sick role identity, most people with chronic conditions do not consider themselves sick or ill and try to live as normal a life as is possible. Most people with chronic conditions take on a sick role identity. Research has demonstrated that some people with chronic conditions may take on a sick role identity, but they are not the majority. Chronic conditions do not result from injury. Chronic conditions may be due to illness, genetic factors, or injury Most chronic conditions are easily controlled. Many chronic conditions require therapeutic regimens to keep them under control.

45. In which phase of the trajectory model of chronic illness are the symptoms under control and managed? Stable The stable phase indicates that the symptoms and disability are under control or managed. Acute The acute phase is characterized by sudden onset of severe or unrelieved symptoms or complications that may necessitate hospitalization for their management. Comeback The comeback phase is the period in the trajectory marked by recovery after an acute period. Downward

The downward phase occurs when symptoms worsen or the disability progresses despite attempts to control the course through proper management.

46. Which phase of the trajectory model of chronic illness is characterized by reactivation of the illness? Unstable The unstable phase is characterized by development of complications or reactivation of the illness. Stable The stable phase indicates that the symptoms and disability are under control or managed. Acute The acute phase is characterized by sudden onset of severe or unrelieved symptoms or complications that may necessitate hospitalization for their management. Comeback The comeback phase is the period in the trajectory marked by recovery after an acute period.

47. Which phase of the trajectory model of chronic illness is characterized by the gradual or rapid decline in the trajectory despite efforts to halt the disorder? Dying The dying phase is characterized by stoppage of life-maintaining functions. Unstable The unstable phase is characterized by development of complications or reactivation of the illness. Acute The acute phase is characterized by sudden onset of severe or unrelieved symptoms or complications that may necessitate hospitalization for their management. Downward The downward phase occurs when symptoms worsen or the disability progresses despite attempts to control the course through proper management.

48. In order to help prevent the development of an external rotation deformity of the hip in a patient who must remain in bed for any period of time, the most appropriate nursing action would be to

use a trochanter roll extending from the crest of the ilium to the midthigh. A trochanter roll, properly placed, provides resistance to the external rotation of the hip. pillows under the lower legs. Pillows under the legs will not prevent the hips from rotating externally. a hip-abductor pillow. A hip-abductor pillow is used for the patient after total hip replacement surgery. a footboard. A footboard will not prevent the hips from rotating externally.

49. To prevent footdrop, the patient is positioned in: Order to keep the feet at right angles to the leg When the patient is supine in bed, padded splints or protective boots are used. A semi-sitting position in bed Semi-fowlers positioning is used to decrease the pressure of abdominal contents on the diaphragm. A sitting position with legs hanging off the side of the bed In order to prevent footdrop, the feet must be supported. A side-lying position Side-lying positions do not provide support to prevent footdrop.

50. Through which of the following activities does the patient learn to consciously contract excretory sphincters and control voiding cues? Biofeedback Cognitively intact patients who have stress or urge incontinence may gain bladder control through biofeedback. Kegel exercises Kegel exercises are pelvic floor exercises that strengthen the pubococcygeus muscle. Habit training Habit training is used to try to keep the patient dry by strictly adhering to a toileting schedule and may be successful with stress, urge, or functional incontinence.

Bladder training Habit training is a type of bladder training.

51. During which stage of pressure ulcer development does the ulcer extend into the subcutaneous tissue? Stage III Clinically, a deep crater with or without undermining of adjacent tissues is noted. Stage IV A stage IV pressure ulcer extends into the underlying structure, including the muscle and possibly the bone. Stage II A stage II ulcer exhibits a break in the skin through the epidermis or dermis. Stage I A stage I pressure ulcer is an area of nonblanchable erythema, tissue swelling, and congestion, and the patient complains of discomfort.

52. During which stage of pressure ulcer development does the ulcer extend into the underlying structures, including the muscle and possibly the bone? Stage IV A stage IV pressure ulcer extends into the underlying structure, including the muscle and possibly the bone. Stage III A stage III ulcer extends into the subcutaneous tissue. Stage II A stage II ulcer exhibits a break in the skin through the epidermis or dermis. Stage I A stage I pressure ulcer is an area of nonblanchable erythema, tissue swelling, and congestion, and the patient complains of discomfort.

53. Which type of incontinence is associated with weakened perineal muscles that permit leakage of urine when intra-abdominal pressure is increased?

Stress incontinence Stress incontinence may occur with coughing or sneezing. Urge incontinence Urge incontinence is involuntary elimination of urine associated with a strong perceived need to void. Reflex (neurogenic) incontinence Neurogenic incontinence is associated with a spinal cord lesion. Functional incontinence Functional incontinence refers to incontinence in patients with intact urinary physiology who experience mobility impairment, environmental barriers, or cognitive problems.

54. Ageism refers to Bias against older people based solely on chronological age Individuals demonstrating ageism base their beliefs and attitudes about older people based upon chronological age without consideration of functional capacity. fear of old age. Fear of aging and the inability of many to confront their own aging process may trigger ageist beliefs. loss of memory. Age-related loss of memory occurs more with short-term and recent memory. benign senescent forgetfulness. Benign senescent forgetfulness refers to the age-related loss of memory in the absence of a pathologic process.

55. When assessing the older adult, the nurse anticipates increase in which of the follow components of respiratory status? Residual lung volume As a result, patient experience fatigue and breathlessness with sustained activity. Vital capacity The nurse anticipates decreased vital capacity. Gas exchange and diffusing capacity The nurse anticipates decreased gas exchange and diffusing capacity resulting in impaired healing

of tissues due to decreased oxygenation. Cough efficiency The nurse anticipates difficulty coughing up secretions due to decreased cough efficiency.

56. According to the classification of hypertension diagnosed in the older adult, hypertension that can be attributed to an underlying cause is termed secondary. Secondary hypertension may be caused by a tumor of the adrenal gland (e.g., pheochromacytoma). primary. Primary hypertension has no known underlying cause. essential. Essential hypertension has no known underlying cause. isolated systolic. Isolated systolic hypertension is demonstrated by readings in which the systolic pressure exceeds 140 mm Hg and the diastolic measurement is normal or near normal (less than 90 mm Hg).

57. Which of the following terms refers to the decrease in lens flexibility that occurs with age, resulting in the near point of focus getting farther away? Presbyopia Presbyopia usually begins in the fifth decade of life, when reading glasses are required to magnify objects. Presbycusis Presbycusis refers to age-related hearing loss. Cataract Cataract is the development of opacity of the lens of the eye. Glaucoma Glaucoma is a disease characterized by increased intraocular pressure.

58. Which of the following states is characterized by a decline in intellectual functioning?

Dementia Dementia is an acquired syndrome in which progressive deterioration in global intellectual abilities is of such severity that it interferes with the person's customary occupational and social performance. Depression Depression is a mood disorder that disrupts quality of life. Delirium Delirium is often called acute confusional state. Delusion Delusion is a symptom of psychoses.

59. When a person who has been taking opioids becomes less sensitive to their analgesic properties, that person is said to have developed a (an) tolerance. Tolerance is characterized by the need for increasing dose requirements to maintain the same level of pain relief. addiction. Addiction refers to a behavioral pattern of substance use characterized by a compulsion to take the drug primarily to experience its psychic effects. dependence. Dependence occurs when a patient who has been taking opioids experiences a withdrawal syndrome when the opioids are discontinued. balanced analgesia. Balanced analgesia occurs when the patient is using more than one form of analgesia concurrently to obtain more pain relief with fewer side effects.

60. Prostaglandins are chemical substances thought to increase sensitivity of pain receptors. Prostaglandins are believed to increase sensitivity to pain receptors by enhancing the painprovoking effect of bradykinin. reduce the perception of pain. Endorphins and enkephalins reduce or inhibit transmission or perception of pain.

inhibit the transmission of pain. Endorphins and enkephalins reduce or inhibit transmission or perception of pain. inhibit the transmission of noxious stimuli. Morphine and other opioid medications inhibit the transmission of noxious stimuli by mimicking enkephalin and endorphin.

61. Which of the following principles or guidelines accurately informs the nurse regarding placebos? Placebos should never be used to test the person's truthfulness about pain. Perception of pain is highly individualized. A placebo effect is an indication that the person does not have pain. A placebo effect is a true physiologic response. A placebo should be used as the first line of treatment for the patient. A placebo should never be used as a first line of treatment. A positive response to a placebo indicates that the person's pain is not real. Reduction in pain as a response to placebo should never be interpreted as an indication that the person's pain is not real.

62. Regarding tolerance and addiction, the nurse understands that although patients may need increasing levels of opioids, they are not addicted. Physical tolerance usually occurs in the absence of addiction. tolerance to opioids is uncommon. Tolerance to opioids is common. addiction to opioids commonly develops. Addiction to opioids is rare. the nurse must be primarily concerned about development of addiction by the patient in pain. Addiction is rare and should never be the primary concern for a patient in pain.

63. The preferred route of administration of medication in the most acute care situations is which of the following routes? Intravenous

The IV route is the preferred parenteral route in most acute care situations because it is much more comfortable for the patient, and peak serum levels and pain relief occur more rapidly and reliably. Epidural Epidural administration is used to control postoperative and chronic pain. Subcutaneous Subcutaneous administration results in slow absorption of medication. Intramuscular Intramuscular administration of medication is absorbed more slowly than intravenously administered medication.

64. Mu opioids have which of the following effects on respiratory rate: Stimulation, then depression Mu opioids also cause bradycardia, hypothermia, and constipation. No change Kappa opioids result in no change in respiratory rate. Stimulation, only Delta opioids result in stimulation of respiratory rate. Depression, only Neither mu, nor kappa, nor delta opoids depress respiratory rate as its only effect upon respiratory rate.

65. Which of the following electrolytes is a major cation in body fluid? Potassium Potassium is a major cation that affects cardiac muscle functioning. Chloride Chloride is an anion. Bicarbonate Bicarbonate is an anion. Phosphate Phosphate is an anion.

66. Which of the following electrolytes is a major anion in body fluid? Chloride Chloride is a major anion found in extracellular fluid. Potassium Potassium is a cation. Sodium Sodium is a cation. Calcium Calcium is a cation.

67. Oncotic pressure refers to the osmotic pressure exerted by proteins. Oncotic pressure is a pulling pressure exerted by proteins, such as albumin. the number of dissolved particles contained in a unit of fluid. Osmolality refers to the number of dissolved particles contained in a unit of fluid. the excretion of substances such as glucose through increased urine output. Osmotic diuresis occurs when the urine output increases due to excretion of substances such as glucose. the amount of pressure needed to stop flow of water by osmosis. Osmotic pressure is the amount of pressure needed to stop the flow of water by osmosis.

68. Which of the following solutions is hypotonic? 0.45% NaCl. Half-strength saline is hypotonic Lactated Ringer's solution. Lactated Ringer's is isotonic. 0.9% NaCl. Normal saline (0.9% NaCl) is isotonic.

5% NaCl. A solution that is 5% NaCl is hypertonic.

69. The normal serum value for potassium is 3.5-5.5 mEq/L. Serum potassium must be within normal limits to prevent cardiac dysrhythmias. 135-145 mEq/L. Normal serum sodium is 135-145 mEq/L. 96-106 mEq/L. Normal serum chloride is 96-106 mEq/L. 8.5-10.5 mg/dL. Normal total serum calcium is 8.5-10.5mg/dL.

70. In which type of shock does the patient experiences a mismatch of blood flow to the cells? Distributive Distributive or vasogenic shock results from displacement of blood volume, creating a relative hypovolemia. Cardiogenic Cardiogenic shock results from the failure of a heart as a pump. Hypovolemic In hypovolemic shock, there is a decrease in the intravascular volume. Septic In septic shock, overwhelming infection results in a relative hypovolemia.

71. Which stage of shock is best described as that stage when the mechanisms that regulate blood pressure fail to sustain a systolic pressure above 90 mm Hg? Progressive In the progressive stage of shock, the mechanisms that regulate blood pressure can no longer compensate, and the mean arterial pressure falls below normal limits.

Refractory The refractory or irreversible stage of shock represents the point at which organ damage is so severe that the patient does not respond to treatment and cannot survive. Compensatory In the compensatory state, the patient's blood pressure remains within normal limits due to vasoconstriction, increased heart rate, and increased contractility of the heart. Irreversible The refractory or irreversible stage of shock represents the point at which organ damage is so severe that the patient does not respond to treatment and cannot survive.

72. When the nurse observes that the patient's systolic blood pressure is less than 80--90 mm Hg, respirations are rapid and shallow, heart rate is over 150 beats per minute, and urine output is less than 30 cc per hour, the nurse recognizes that the patient is demonstrating which stage of shock? Compensatory In compensatory shock, the patient's blood pressure is normal, respirations are above 20, and heart rate is above 100 but below 150. Progressive In progressive shock, the patient's skin appears mottled and mentation demonstrates lethargy. Refractory In refractory or irreversible shock, the patient requires complete mechanical and pharmacologic support. Irreversible In refractory or irreversible shock, the patient requires complete mechanical and pharmacologic support.

73. Which of the following vasoactive drugs used in treating shock results in reduced preload and afterload, reducing oxygen demand of the heart? Nitroprusside (Nipride) A disadvantage of nitroprusside is that it causes hypotension. Dopamine (Intropin) Dopamine improves contractility, increases stroke volume, and increases cardiac output. Epinephrine (adrenaline) Epinephrine improves contractility, increases stroke volume, and increases cardiac output.

Methoxamine (Vasoxyl) Methoxamine increases blood pressure by vasoconstriction.

74. The nurse anticipates that the immunosuppressed patient is at greatest risk for which type of shock? Septic Septic shock is associated with immunosuppression, extremes of age, malnourishment, chronic illness, and invasive procedures. Neurogenic Neurogenic shock is associated with spinal cord injury and anesthesia. Cardiogenic Cardiogenic shock is associated with disease of the heart. Anaphylactic Anaphylactic shock is associated with hypersensitivity reactions. 75. Which of the following colloids is expensive but rapidly expands plasma volume? Albumin Albumin is a colloid that requires human donors, is limited in supply, and can cause congestive heart failure. Dextran Dextran is a colloid, synthetic plasma expander that interferes with platelet aggregation and is not recommended for hemorrhagic shock. Lactated Ringers Lactated ringers is a crystalloid, not a colloid. Hypertonic Saline Hypertonic saline is a crystalloid, not a colloid.

76. Which of the following terms refers to cells that lack normal cellular characteristics and differ in shape and organization with respect to their cells of origin? Anaplasia Usually, anaplastic cells are malignant.

Neoplasia Neoplasia refers to uncontrolled cell growth that follows no physiologic demand. Dysplasia Dysplasia refers to bizarre cell growth resulting in cells that differ in size, shape, or arrangement from other cells of the same type of tissue. Hyperplasia Hyperplasia refers to an increase in the number of cells of a tissue, most often associated with a period of rapid body growth.

77. Palliation refers to relief of symptoms associated with cancer. Palliation is the goal for care in terminal cancer patients. hair loss. Alopecia is the term that refers to hair loss. the spread of cancer cells from the primary tumor to distant sites. Metastasis is the term that refers to the spread of cancer cells from the primary tumor to distant sites. the lowest point of white blood cell depression after therapy that has toxic effects on the bone marrow. Nadir is the term that refers to the lowest point of white blood cell depression after therapy that has toxic effects on the bone marrow.

78. During which step of cellular carcinogenesis do cellular changes exhibit increased malignant behavior? Progression During this third step, cells show a propensity to invade adjacent tissues and metastasize. Promotion During promotion, repeated exposure to promoting agents causes the expression of abnormal genetic information even after long latency periods. Initiation During this first step, initiators such as chemicals, physical factors, and biologic agents escape normal enzymatic mechanisms and alter the genetic structure of cellular DNA.

Prolongation No stage of cellular carcinogenesis is termed prolongation.

79. The drug, Interleukin-2, is an example of which type of biologic response modifier? Cytokine Other cytokines include interferon alfa and filgrastim. Monoclonal antibodies Monoclonal antibodies include rituximab, trastuzumab, and gemtuzumab. Retinoids Retinoic acid is an example of a retinoid. Antimetabolites Antimetabolites are cell cycle-specific antineoplastic agents.

80. Of the following terms, which is used to refer to the period of time during which mourning a loss takes place? Bereavement Bereavement is the period of time during which mourning a loss takes place. Grief Grief is the personal feelings that accompany an anticipated or actual loss Mourning Mourning is the individual, family, group and cultural expressions of grief and associated behaviors Hospice Hospice is a coordinated program of interdisciplinary care and services provided primarily in the home to terminally ill patients and their families.

81. Which of the following "awareness contexts" is characterized by the patient, the family, and the health care professionals being aware that the patient is dying but all pretend otherwise? Mutual pretense awareness In mutual pretense awareness, the patient, the family and the health care professionals are aware

that the patient is dying but all pretend otherwise. Closed awareness In closed awareness, the patient is unaware of his terminality in a context where others are aware. Suspected awareness In suspected awareness, the patient suspects what others know and attempts to find it out. Open awareness In open awareness, all are aware that the patient is dying and are able to openly acknowledge that reality.

82. For individuals known to be dying by virtue of age and/or diagnoses, which of the following signs indicate approaching death: Increased restlessness As the oxygen supply to the brain decreases, the patient may become restless. Increased wakefulness As the body weakens, the patient will sleep more and begin to detach from the environment. Increased eating For many patients, refusal of food is an indication that they are ready to die. Increased urinary output Based upon decreased intake, urinary output generally decreases in amount and frequency.

83. Which of the following terms best describes a living will? Medical directive The living will is a type of advance medical directive in which the individual of sound mind documents treatment preferences. Proxy directive A proxy directive is the appointment and authorization of another individual to make medical decisions on behalf of the person who created an advance directive when he/she is no loner able to speak for him/herself. Health care power of attorney Health care power of attorney is a legal document that enables the signer to designate another individual to make health care decisions on his/her behalf when he/she is unable to do so.

Durable power of attorney for health A durable power of attorney for health care is a legal document that enables the signer to designate another individual to make health care decisions on his/her behalf when he/she is unable to do so.

84. A malignant tumor gains access to the blood and lymphatic channels. By this mechanism, the tumor metastasizes to other areas of the body. demonstrates cells that are well-differentiated. Cells of malignant tumors are undifferentiated. is usually slow growing. Malignant tumors demonstrate variable rate of growth; however, the more anaplastic the tumor, the faster its growth. grows by expansion. A malignant tumor grows at the periphery and sends out processes that infiltrate and destroy surrounding tissues.

85. Which of the following classes of antineoplastic agents is cell--cycle-specific? Antimetabolites (5-FU) Antimetabolites are cell--cycle-specific (S phase). Antitumor antibiotics (bleomycin) Antitumor antibiotics are cell-cycle nonspecific. Alkylating agents (cisplatin) Alkylating agents are cell-cycle nonspecific. Nitrosureas (carmustine) Nitrosureas are cell-cycle nonspecific.

86. Regarding the surgical patient, which of the following terms refers to the period of time that constitutes the surgical experience? Perioperative phase Perioperative period includes the preoperative, intraoperative, and postoperative phases.

Preoperative phase Preoperative phase is the period of time from when the decision for surgical intervention is made to when the patient is transferred to the operating room table. Intraoperative phase Intraoperataive phase is the period of time from when the patient is transferred to the operating room table to when he or she is admitted to the postanesthesia care unit. Postoperative phase Postoperative phase is the period of time that begins with the admission of the patient to the postanesthesia care unit and ends after a follow-up evaluation in the clinical setting or home.

87. When the indication for surgery is without delay, the nurse recognizes that the surgery will be classified as emergency. Emergency surgery means that the patient requires immediate attention and the disorder may be life-threatening. urgent. Urgent surgery means that the patient requires prompt attention within 24-30 hours. required. Required surgery means that the patient needs to have surgery, and it should be planned within a few weeks or months. elective. Elective surgery means that there is an indication for surgery, but failure to have surgery will not be catastrophic.

88. When a person with a history of chronic alcoholism is admitted to the hospital for surgery, the nurse anticipates that the patient may show signs of alcohol withdrawal delirium during which time period? Up to 72 hours after alcohol withdrawal Alcohol withdrawal delirium is associated with a significant mortality rate when it occurs postoperatively. Immediately upon admission Onset of symptoms depends upon time of last consumption of alcohol.

Upon awakening in the post-anesthesia care unit Onset of symptoms depends upon time of last consumption of alcohol. Up to 24 hours after alcohol withdrawal Twenty-four hours is too short a time frame to consider alcohol withdrawal delirium no longer a threat to a chronic alcoholic.

89. Which of the following categories of medications may result in seizure activity if withdrawn suddenly? Tranquilizers Abrupt withdrawal of tranquilizers may result in anxiety, tension, and even seizures if withdrawn suddenly. Adrenal corticosteroids Abrupt withdrawal of steroids may precipitate cardiovascular collapse. Antidepressants Monoamine oxidase inhibitors increase the hypotensive effects of anesthetics. Diuretics Thiazide diuretics may cause excessive respiratory depression during anesthesia due to an associated electrolyte imbalance.

90. When the patient is encouraged to concentrate on a pleasant experience or restful scene, the cognitive coping strategy being employed by the nurse is imagery. Imagery has proven effective for oncology patients. optimistic self-recitation. Optimistic self-recitation is practiced when the patient is encouraged to recite optimistic thoughts such as “I know all will go well.” distraction. Distraction is employed when the patient is encouraged to think of an enjoyable story or recite a favorite poem. progressive muscular relaxation. Progressive muscular relaxation requires contracting and relaxing muscle groups and is a physical coping strategy as opposed to cognitive.

91. According to the American Society of Anesthesiology Physical Status Classification System, a patient with severe systemic disease that is not incapacitating is noted to have physical status classification P3 Classification P3 patients are those who have compensated heart failure, cirrhosis, or poorly controlled diabetes, for example. P4 Classification P4 patients have an incapacitating systemic disease that is a constant threat to life. P1 Classification P1 refers to a normal healthy patient P2 Classification P2 reflects a patient with mild systemic disease

92. Which stage of anesthesia is termed surgical anesthesia? III With proper administration of the anesthetic, this stage may be maintained for hours. I Stage I is beginning anesthesia, as the patient breathes in the anesthetic mixture and experiences warmth, dizziness, and a feeling of detachment. II Stage II is the excitement stage, which may be characterized by struggling, singing, laughing, or crying. IV Stage IV is a stage of medullary depression and is reached when too much anesthesia has been administered.

93. Fentanyl (Sublimaze) is categorized as which type of intravenous anesthetic agent? Neuroleptanalgesic Fentanyl is 75-100 times more potent than morphine and has about 25% of the duration of morphine (IV).

Tranquilizer Examples of tranquilizers include midazolam (Versed) and diazepam (Valium). Opioid Opioids include morphine and meperidine hydrochloride (Demerol). Dissociative agent Ketamine is a dissociative agent.

94 Which of the following manifestations is often the earliest sign of malignant hyperthermia? Tachycardia (heart rate above 150 beats per minute) Tachycardia is often the earliest sign of malignant hyperthermia. Hypotension Hypotension is a later sign of malignant hyperthermia. Elevated temperature The rise in temperature is actually a late sign that develops rapidly. Oliguria Scant urinary output is a later sign of malignant hyperthermia.

95. Which of the following terms is used to refer to protrusion of abdominal organs through the surgical incision? Evisceration Evisceration is a surgical emergency. Hernia A hernia is a weakness in the abdominal wall. Dehiscence Dehiscence refers to partial or complete separation of wound edges. Erythema Erythema refers to redness of tissue.

96. When the method of wound healing is one in which wound edges are not surgically approximated and integumentary continuity is restored by granulations, the wound healing is termed

second intention healing. When wounds dehisce, they will be allowed to heal by secondary intention. primary intention healing. Primary or first intention healing is the method of healing in which wound edges are surgically approximated and integumentary continuity is restored without granulating. first intention healing. Primary or first intention healing is the method of healing in which wound edges are surgically approximated and integumentary continuity is restored without granulating. third intention healing. Third intention healing is a method of healing in which surgical approximation of wound edges is delayed and integumentary continuity is restored by bringing apposing granulations together.

97. The nurse recognizes which of the following signs as typical of the patient in shock? Rapid, weak, thready pulse Pulse increases as the body tries to compensate. Flushed face Pallor is an indicator of shock. Warm, dry skin Skin is generally cool and moist in shock. Increased urine output Usually, a low blood pressure and concentrated urine are observed in the patient in shock.

98. When the nurse observes that the postoperative patient demonstrates a constant low level of oxygen saturation, although the patient's breathing appears normal, the nurse identifies that the patient may be suffering which type of hypoxemia? Subacute Supplemental oxygen may be indicated. Hypoxic Hypoxic hypoxemia results from inadequate breathing. Episodic Episodic hypoxemia develops suddenly, and the patient may be at risk for myocardial ischemia,

cerebral dysfunction, and cardiac arrest. Anemic Anemic hypoxemia results from blood loss during surgery.

99. When the surgeon performs an appendectomy, the nurse recognizes that the surgical category will be identified as clean contaminated. Clean-contaminated cases are those with a potential, limited source for infection, the exposure to which, to a large extent, can be controlled. clean. Clean cases are those with no apparent source of potential infection. contaminated. Contaminated cases are those that contain an open and obvious source of potential infection. dirty. A traumatic wound with foreign bodies, fecal contamination, or purulent drainage would be considered a dirty case.

100. Which of the following terms is used to describe inability to breathe easily except in an upright position? Orthopnea Patients with orthopnea are placed in a high Fowler's position to facilitate breathing. Dyspnea Dyspnea refers to labored breathing or shortness of breath. Hemoptysis Hemoptysis refers to expectoration of blood from the respiratory tract. Hypoxemia Hypoxemia refers to low oxygen levels in the blood.

101. In relation to the structure of the larynx, the cricoid cartilage is the only complete cartilaginous ring in the larynx.

The cricoid cartilage is located below the thyroid cartilage. used in vocal cord movement with the thyroid cartilage. The arytenoid cartilages are used in vocal cord movement with the thyroid cartilage. the largest of the cartilage structures. The thyroid cartilage is the largest of the cartilage structures; part of it forms the Adam's apple. the valve flap of cartilage that covers the opening to the larynx during swallowing. The epiglottis is the valve flap of cartilage that covers the opening to the larynx during swallowing.

102. Which respiratory volume is the maximum volume of air that can be inhaled after maximal expiration? Inspiratory reserve volume Inspiratory reserve volume is normally 3000 mL. Tidal volume Tidal volume is the volume of air inhaled and exhaled with each breath. Expiratory reserve volume Expiratory reserve volume is the maximum volume of air that can be exhaled forcibly after a normal exhalation. Residual volume Residual volume is the volume of air remaining in the lungs after a maximum exhalation.

103. The individual who demonstrates displacement of the sternum is described as having a pigeon chest. Pigeon chest may occur with rickets, Marfan's syndrome, or severe kyphoscoliosis. barrel chest. A barrel chest is seen in patients with emphysema as a result of over-inflation of the lungs. funnel chest. A funnel chest occurs when there is a depression in the lower portion of the sternum. kyphoscoliosis. Kyphoscoliosis is characterized by elevation of the scapula and a corresponding S-shaped spine.

104. When the nurse auscultates chest sounds that are harsh and cracking, sounding like two pieces of leather being rubbed together, she records her finding as pleural friction rub. A pleural friction rub is heard secondary to inflammation and loss of lubricating pleural fluid. crackles. Crackles are soft, high-pitched, discontinuous popping sounds that occur during inspiration. sonorous wheezes. Sonorous wheezes are deep, low-pitched rumbling sounds heard primarily during expiration. sibilant wheezes. Sibilant wheezes are continuous, musical, high-pitched, whistle-like sounds heard during inspiration and expiration.

105. Which of the following terms is used to describe hemorrhage from the nose? Epistaxis Epistaxis is due to rupture of tiny, distended vessels in the mucous membrane of any area of the nose. Xerostomia Xerostomia refers to dryness of the mouth. Rhinorrhea Rhinorrhea refers to drainage of a large amount of fluid from the nose. Dysphagia Dysphagia refers to difficulties in swallowing.

106. The herpes simplex virus (HSV-1), which produces a cold sore (fever blister), has an incubation period of 2-12 days. HSV-1 is transmitted primarily by direct contact with infected secretions. 0-3 months. The time period 0-3 months exceeds the incubation period.

20-30 days. The time period 20-30 days exceeds the incubation period. 3-6 months. The time period 3-6 months exceeds the incubation period.

107. Another term for clergyman's sore throat is chronic granular pharyngitis. In clergyman's sore throat, the pharynx is characterized by numerous swollen lymph follicles. aphonia. Aphonia refers to the inability to use one's voice. atrophic pharyngitis. Atrophic pharyngitis is characterized by a membrane that is thin, white, glistening, and at times wrinkled. hypertrophic pharyngitis. Hypertrophic pharyngitis is characterized by general thickening and congestion of the pharyngeal mucous membrane.

108. Which type of sleep apnea is characterized by lack of airflow due to pharyngeal occlusion? Obstructive Obstructive sleep apnea occurs usually in men, especially those who are older and overweight. Simple Types of sleep apnea do not include a simple characterization. Mixed Mixed sleep apnea is a combination of central and obstructive apnea with one apneic episode. Central In central sleep apnea, the patient demonstrates simultaneous cessation of both airflow and respiratory movements.

109. When the patient who has undergone laryngectomy suffers wound breakdown, the nurse monitors him very carefully because he is identified as being at high risk for

carotid artery hemorrhage. The carotid artery lies close to the stoma and may rupture from erosion if the wound does not heal properly. pulmonary embolism. Pulmonary embolism is associated with immobility. dehydration. Dehydration may lead to poor wound healing and breakdown. pneumonia. Pneumonia is a risk for any postoperative patient.

110. Which of the following terms refers to lung tissue that has become more solid in nature due to a collapse of alveoli or infectious process? Consolidation Consolidation occurs during an infectious process such as pneumonia. Atelectasis Atelectasis refers to collapse or airless condition of the alveoli caused by hypoventilation, obstruction to the airways, or compression. Bronchiectasis Bronchiectasis refers to chronic dilation of a bronchi or bronchi in which the dilated airway becomes saccular and a medium for chronic infection. Empyema Empyema refers to accumulation of purulent material in the pleural space.

111. Which of the following community-acquired pneumonias demonstrates the highest occurrence during summer and fall? Legionnaires' disease Legionnaires' disease accounts for 15% of community-acquired pneumon

Posted by StuffedNurse (co-author) at 8:32 PM

Wednesday, January 23, 2008 StuffedNurse : NP1 practice exam NURSING PRACTICE I – Foundation of PROFESSIONAL Nursing Practice SITUATIONAL Situation 1 – Mr. Ibarra is assigned to the triage area and while on duty, he assesses the condition of Mrs. Simon who came in with asthma. She has difficulty breathing and her respiratory rate is 40 per minute. Mr. Ibarra is asked to inject the client epinephrine 0.3 mg subcutaneously. 1. The indication for epinephrine injection for Mrs. Simon is to: A. Reduce anaphylaxis B. Relieve hypersensitivity to allergen C. Relieve respiratory distress due to bronchial spasm D. Restore client’s cardiac rhythm 2. When preparing the epinephrine injection from an ampule, the nurse initially: A. Taps the ampule at the top to allow fluid to flow to the base of the ampule B. Checks expiration date of the medication ampule C. Removes needle cap of syringe and pulls plunger to expel air D. Breaks the neck of the ampule with a gauze wrapped around it

3. Mrs. Simon is obese. When administering a subcutaneous injection to an obese patient, it is best for the nurse to: A. Inject needle at a 15 degree angle over the stretched skin of the client B. Pinch skin at the injection site and use airlock technique C. Pull skin of patient down to administer the drug in a Z track D. Spread skin or pinch at the injection site and inject needle at a 45-90 degree angle 4. When preparing for a subcutaneous injection, the proper size of syringe and needle would be: A. Syringe 3-5 ml and needle gauge 21 to 23 B. Tuberculin syringe 1 ml with needle gauge 26 or 27 C. Syringe 2 ml and needle gauge 22 D. Syringe 1-3 ml and needle gauge 25 to 27 5. The rationale for giving medications through the subcutaneous route is: A. There are many alternative sites for subcutaneous injection B. Absorption time of the medicine is slower C. There are less pain receptors in this area D. The medication can be injected while the client is in any position Situation 2 – The use of massage and meditation to help decrease stress and pain have been strongly recommended based on documented testimonials. 6. Martha wants to do a study on this topic: “Effects of massage and meditation on stress and pain”. The type of research that best suits this topic is: A. Applied research B. Qualitative research C. Basic research D. Quantitative research 7. The type of research design that does not manipulate independent variable is: A. Experimental design B. Quasi-experimental design C. Non-experimental design D. Quantitative design 8. This research topic has the potential to contribute to nursing because it seeks to A. include new modalities of care B. resolve a clinical problem C. clarify an ambiguous modality of care

D. enhance client care 9. Martha does review of related literature for the purpose of A. determine statistical treatment of data research B. gathering data about what is already known or unknown about the problem C. to identify if problem can be replicated D. answering the research question 10. Client’s rights should be protected when doing research using human subjects. Martha identifies these rights as follows EXCEPT: A. right of self-determination B. right to compensation C. right of privacy D. right not to be harmed Situation 3 – Richard has a nursing diagnosis of ineffective airway clearance related to excessive secretions and is at risk for infection because of retained secretions. Part of Nurse Mario’s nursing care plan is to loosen and remove excessive secretions in the airway. 11. Mario listens to Richard’s bilateral sounds and finds that congestion is in the upper lobes of the lungs. The appropriate position to drain the anterior and posterior apical segments of the lungs when Mario does percussion would be: A. Client lying on his back then flat on his abdomen on Trendelenburg position B. Client seated upright in bed or on a chair then leaning forward in sitting position then flat on his back and on his abdomen C. Client lying flat on his back and then flat on his abdomen D. Client lying on his right then left side on Trendelenburg position 12. When documenting outcome of Richard’s treatment Mario should include the following in his recording EXCEPT: A. Color, amount and consistency of sputum B. Character of breath sounds and respiratory rate before and after procedure C. Amount of fluid intake of client before and after the procedure D. Significant changes in vital signs 13. When assessing Richard for chest percussion or chest vibration and postural drainage, Mario would focus on the following EXCEPT: A. Amount of food and fluid taken during the last meal before treatment B. Respiratory rate, breath sounds and location of congestion C. Teaching the client’s relatives to perform the procedure D. Doctor’s order regarding position restrictions and client’s tolerance for lying flat

14. Mario prepares Richard for postural drainage and percussion. Which of the following is a special consideration when doing the procedure? A. Respiratory rate of 16 to 20 per minute B. Client can tolerate sitting and lying positions C. Client has no signs of infection D. Time of last food and fluid intake of the client 15. The purpose of chest percussion and vibration is to loosen secretions in the lungs. The difference between the procedures is: A. Percussion uses only one hand while vibration uses both hands B. Percussion delivers cushioned blows to the chest with cupped palms while vibration gently shakes secretion loose on the exhalation cycle C. In both percussion and vibration the hands are on top of each other and hand action is in tune with client’s breath rhythm D. Percussion slaps the chest to loosen secretions while vibration shakes the secretions along with the inhalation of air Situation 4 – A 61 year old man, Mr. Regalado, is admitted to the private ward for observation after complaints of severe chest pain. You are assigned to take care of the client. 16. When doing an initial assessment, the best way for you to identify the client’s priority problem is to: A. Interview the client for chief complaints and other symptoms B. Talk to the relatives to gather data about history of illness C. Do auscultation to check for chest congestion D. Do a physical examination while asking the client relevant questions 17. Upon establishing Mr. Regalado’s nursing needs, the next nursing approach would be to: A. Introduce the client to the ward staff to put the client and family at ease B. Give client and relatives a brief tour of the physical set up the unit C. Take his vital signs for a baseline assessment D. Establish priority needs and implement appropriate interventions 18. Mr. Regalado says he has “trouble going to sleep”. In order to plan your nursing intervention you will: A. Observe his sleeping patterns in the next few days B. Ask him what he means by this statement C. Check his physical environment to decrease noise level D. Take his blood pressure before sleeping and upon waking up

19. Mr. Regalado’s lower extremities are swollen and shiny. He has pitting pedal edema. When taking care of Mr. Regalado, which of the following interventions would be the most appropriate immediate nursing approach? A. Moisturize lower extremities to prevent skin irritation B. Measure fluid intake and output to decrease edema C. Elevate lower extremities for postural drainage D. Provide the client a list of food low in sodium 20. Mr. Regalado will be discharged from your unit within the hour. Nursing actions when preparing a client for discharge include all EXCEPT: A. Making a final physical assessment before client leaves the hospital B. Giving instructions about his medication regimen C. Walking the client to the hospital exit to ensure his safety D. Proper recording of pertinent data Situation 5 – Nancy, mother of 2 young kids, 36 years old, had a mammogram and was told that she has breast cysts and that she may need surgery. This causes her anxiety as shown by increase in her pulse and respiratory rate, sweating and feelings of tension. 21. Considering her level of anxiety, the nurse can best assist Nancy by: A. Giving her activities to divert her attention B. Giving detailed explanations about the treatments she will undergo C. Preparing her and her family in case surgery is not successful D. Giving her clear but brief information at the level of her understanding 22. Nancy blames God for her situation. She is easily provoked to tears and wants to be left alone, refusing to eat or talk to her family. A religious person before, she now refuses to pray or go to church stating that God has abandoned her. The nurse understands that Nancy is grieving for her self and is in the stage of: A. bargaining B. denial C. anger D. acceptance 23. The nurse visits Nancy and prods her to eat her food. Nancy replies “what’s the use? My time is running out.” The nurse’s best response would be: A. “The doctor ordered full diet for you so that you will be strong for surgery” B. “I understand how you feel but you have to try for your children’s sake” C. “Have you told your doctor how you feel? Are you changing your mind about your surgery?” D. “You sound like you are giving up.”

24. The nurse feels sad about Nancy’s illness and tells her head nurse during the end of shift endorsement that “it’s unfair for Nancy to have cancer when she is still so young and with two kids”. The best response of the head nurse would be: A. Advise the nurse to “be strong and learn to control her feelings” B. Assign the nurse to another client to avoid sympathy for the client C. Reassure the nurse that the client has hope if she goes through all treatments prescribed for her D. Ask the other nurses what they feel about the patient to find out if they share the same feelings 25. Realizing that she feels angry about Nancy’s condition, the nurse learns that being self-aware is a conscious process that she should do in any situation like this because: A. This is a necessary part of the nurse – client relationship process B. The nurse is a role model for the client and should be strong C. How the nurse thinks and feels affect her actions towards her client and her work D. The nurse has to be therapeutic at all times and should not be affected Situation 6 – Mrs. Seva, 52 years old, asks you about possible problems regarding her elimination now that she is in the menopausal stage. 26. Instruction on health promotion regarding urinary elimination is important. Which would you include? A. Hold urine as long as she can before emptying the bladder to strengthen her sphincter muscles B. If burning sensation is experienced while voiding, drink pineapple juice C. After urination, wipe from anal area up towards the pubis D. Tell client to empty the bladder at each voiding 27. Mrs. Seva also tells the nurse that she is often constipated. Because she is aging, what physical changes predispose her to constipation? A. inhibition of the parasympathetic reflex B. weakness of sphincter muscles of anus C. loss of tone of the smooth muscles of the colon D. decreased ability to absorb fluids in the lower intestines 28. The nurse understands that one of these factors contributes to constipation: A. excessive exercise B. high fiber diet C. no regular time for defecation daily D. prolonged use of laxatives 29. Mrs. Seva talks about fear of being incontinent due to a prior experience of dribbling urine when laughing or sneezing and when she has a full bladder. Your most appropriate instruction would be to:

A. tell client to drink less fluids to avoid accidents B. instruct client to start wearing thin adult diapers C. ask the client to bring change of underwear “just in case” D. teach client pelvic exercise to strengthen perineal muscles 30. Mrs. Seva asked for instructions for skin care for her mother who has urinary incontinence and is almost always in bed. Your instruction would focus on prevention of skin irritation and breakdown by: A. Using thick diapers to absorb urine well B. Drying the skin with baby powder to prevent or mask the smell of ammonia C. Thorough washing, rising and drying of skin area that get wet with urine D. Making sure that linen are smooth and dry at all times Situation 7 – Using Maslow’s need theory, Airway, Breathing and Circulation are the physiological needs vital to life. The nurse’s knowledge and ability to identify and immediately intervene to meet these needs is important to save lives. 31. Which of these clients has a problem with the transport of oxygen from the lungs to the tissues: A. Carol with tumor in the brain B. Theresa with anemia C. Sonnyboy with a fracture in the femur D. Brigitte with diarrhea 32. You noted from the lab exams in the chart of M. Santos that he has reduced oxygen in the blood. This condition is called: A. Cyanosis B. Hypoxia C. Hypoxemia D. Anemia 33. You will do nasopharyngeal suctioning on Mr. Abad. Your guide for the length of insertion of the tubing for an adult would be: A. tip of the nose to the base of the neck B. the distance from the tip of the nose to the middle of the neck C. the distance from the tip of the nose to the tip of the ear lobe D. eight to ten inches 34. While doing nasopharyngeal suctioning on Mr. Abad, the nurse can avoid trauma to the area by:

A. Apply suction for at least 20-30 seconds each time to ensure that all secretions are removed B. Using gloves to prevent introduction of pathogens to the respiratory system C. Applying no suction while inserting the catheter D. Rotating catheter as it is inserter with gentle suction 35. Myrna has difficulty breathing when on her back and must sit upright in bed to breath effectively and comfortably. The nurse documents this condition as: A. Apnea B. Orthopnea C. Dyspnea D. Tachypnea Situation 8 – You are assigned to screen for hypertension. Your task is to take blood pressure readings and you are informed about avoiding the common mistakes in BP taking that lead to false or inaccurate blood pressure readings. 36. When taking blood pressure reading the cuff should be: A. deflated fully then immediately start second reading for same client B. deflated quickly after inflating up to 180 mmHg C. large enough to wrap around upper arm of the adult client 1 cm above brachial artery D. inflated to 30 mmHg above the estimated systolic BP based on palpation of radial or bronchial artery 37. Chronic Obstructive Pulmonary Disease (COPD) in one of the leading causes of death world wide and is a preventable disease. The primary cause of COPD is A. tobacco hack B. bronchitis C. asthma D. cigarette smoking 38. In your health education class for clients with diabetes you teach them the areas for control of Diabetes which include all EXCEPT A. regular physical activity B. thorough knowledge of foot care C. prevention nutrition D. proper nutrition 39. You teach your clients the difference between, Type I (IDDM) and Type II (NDDM) diabetes. Which of the following is true? A. both types diabetes mellitus clients are all prone to developing ketosis B. Type II (NIDDM) is more common and is also preventable compared to Type I (IDDM)

diabetes which is genetic in etiology C. Type I (IIDM) is characterized by fasting hyperglycemia D. Type II (NIDDM) is characterized by abnormal immune response 40. Lifestyle-related diseases in general share areas common risk factors. These are the following except: A. physical activity B. smoking C. genetics D. nutrition Situation 9 – Nurse Rivera witnesses a vehicular accident near the hospital where she works. She decides to get involved and help the victims of the accident 41. Her priority nursing action would be to: A. Assess damage to property B. Assist in the police investigation since she is a witness C. Report the incident immediately to the local police authorities D. Assess the extent of injuries incurred by the victims of the accident 42. Priority attention should be given to which of these clients? A. Linda who shows severe anxiety due to trauma of the accident B. Ryan who has chest injury, is pale and with difficulty breathing C. Noel who has lacerations on the arms with mild bleeding D. Andy whose left ankle swelled and has some abrasions 43. In the emergency room, Nurse Rivera is assigned to attend to the client with lacerations on the arms. While assessing the extent of the wound the nurse observes that the wound is now starting to bleed profusely. The most immediate nursing action would be to: A. Apply antiseptic to prevent infection B. Clean the wound vigorously of contaminants C. Control and reduce bleeding of the wound D. Bandage the wound and elevate the arm 44. The nurse applies dressing on the bleeding site. This intervention is done to: A. Reduce the need to change dressing frequently B. Allow the pus to surface faster C. Protect the wound from microorganisms in the air D. Promote hemostasis 45. After the treatment, the client is sent home and asked to come back for follow-up care. Your

responsibilities when the client is to be discharged include the following EXCEPT: A. Encouraging the client to go to the outpatient clinic for follow up care B. Accurate recording of treatment done and instructions given to client C. Instructing the client to see you after discharge for further assistance D. Providing instructions regarding wound care Situation 10 – While working in the clinic, a new client, Geline, 35 years old, arrives for her doctor’s appointment. As the clinic nurse, you are to assist the client fill up forms, gather data and make an assessment. 46. The purpose of your initial nursing interview is to: A. Record pertinent information in the client’s chart for health team to read B. Assist the client find solutions to he her health concerns C. Understand her lifestyle, health needs and possible problems to develop a plan of care D. Make nursing diagnoses for identified health problems 47. While interviewing Geline, she starts to moan and doubles up in pain. She tells you that this pain occurs about an hour after taking black coffee without breakfast for a few weeks now. You will record this as follows: A. Claims to have abdominal pains after intake of coffee unrelieved by analgesics B. After drinking coffee, the client experienced severe abdominal pain C. Client complained of intermittent abdominal pain an hour after drinking coffee D. Client reported abdominal pain an hour after drinking black coffee for three weeks now. 48. Geline tells you that she drinks black coffee frequently within the day to “have energy and be wide awake” and she eats nothing for breakfast and eats strictly vegetable salads for lunch and dinner to lose weight. She has lost weight during the past two weeks. In planning a healthy balanced diet with Geline, you will: A. Start her off with a cleansing diet to free her body of toxins then change to a vegetarian diet and drink plenty of fluids B. Plan a high protein diet, low carbohydrate diet for her considering her favorite food. C. Instruct her to attend classes in nutrition to find food rich in complex carbohydrates to maintain daily high energy level. D. Discuss with her the importance of eating a variety of food from major food groups with plenty of fluids. 49. Geline tells you that she drinks 4-5 cups of black coffee and diet cola drinks. She also smokes up to a pack of cigarettes daily. She confesses that she is in her 2nd month of pregnancy but does not want to become fat that is why she limits her food intake. You warn or caution her about which of the following? A. Caffeine products affect the central nervous system and may cause the mother to have a

“nervous breakdown” B. Malnutrition and its possible effects on growth and development problems in the unborn fetus C. Caffeine causes a stimulant effect on both mother and the baby D. Studies show conclusively that caffeine causes mental retardation 50. Your health education plan for Geline stresses proper diet for a pregnant woman and the prevention of non-communicable diseases that are influenced by her lifestyle. These include the following EXCEPT: A. Cardiovascular diseases B. Cancer C. Diabetes Mellitus D. Osteoporosis answer key np1 note: guys pacensya na dont have time yet to post the rationales, super haba most of them, maybe later this month or maybe i'll post ung mga sources na lang na books so you can read them on your own.(^^,) answers na lang muna for now po.. -diannemaydee NURSING PRACTICE I – Foundation of PROFESSIONAL Nursing Practice SITUATIONAL Situation 1 – Mr. Ibarra is assigned to the triage area and while on duty, he assesses the condition of Mrs. Simon who came in with asthma. She has difficulty breathing and her respiratory rate is 40 per minute. Mr. Ibarra is asked to inject the client epinephrine 0.3 mg subcutaneously. 1. The indication for epinephrine injection for Mrs. Simon is to: Relieve respiratory distress due to bronchial spasm 2. When preparing the epinephrine injection from an ampule, the nurse initially: Checks expiration date of the medication ampule 3. Mrs. Simon is obese. When administering a subcutaneous injection to an obese patient, it is best for the nurse to: Spread skin or pinch at the injection site and inject needle at a 45-90 degree angle 4. When preparing for a subcutaneous injection, the proper size of syringe and needle would be: Syringe 1-3 ml and needle gauge 25 to 27

5. The rationale for giving medications through the subcutaneous route is: Absorption time of the medicine is slower Situation 2 – The use of massage and meditation to help decrease stress and pain have been strongly recommended based on documented testimonials. 6. Martha wants to do a study on this topic: “Effects of massage and meditation on stress and pain”. The type of research that best suits this topic is: Qualitative research 7. The type of research design that does not manipulate independent variable is: Non-experimental design 8. This research topic has the potential to contribute to nursing because it seeks to enhance client care 9. Martha does review of related literature for the purpose of A. athering data about what is already known or unknown about the problem 10. Client’s rights should be protected when doing research using human subjects. Martha identifies these rights as follows EXCEPT: right to compensation Situation 3 – Richard has a nursing diagnosis of ineffective airway clearance related to excessive secretions and is at risk for infection because of retained secretions. Part of Nurse Mario’s nursing care plan is to loosen and remove excessive secretions in the airway. 11. Mario listens to Richard’s bilateral sounds and finds that congestion is in the upper lobes of the lungs. The appropriate position to drain the anterior and posterior apical segments of the lungs when Mario does percussion would be: Client seated upright in bed or on a chair then leaning forward in sitting position then flat on his back and on his abdomen 12. When documenting outcome of Richard’s treatment Mario should include the following in his recording EXCEPT: Amount of fluid intake of client before and after the procedure

13. When assessing Richard for chest percussion or chest vibration and postural drainage, Mario would focus on the following EXCEPT: Teaching the client’s relatives to perform the procedure 14. Mario prepares Richard for postural drainage and percussion. Which of the following is a special consideration when doing the procedure? Client can tolerate sitting and lying positions 15. The purpose of chest percussion and vibration is to loosen secretions in the lungs. The difference between the procedures is: Percussion delivers cushioned blows to the chest with cupped palms while vibration gently shakes secretion loose on the exhalation cycle Situation 4 – A 61 year old man, Mr. Regalado, is admitted to the private ward for observation after complaints of severe chest pain. You are assigned to take care of the client. 16. When doing an initial assessment, the best way for you to identify the client’s priority problem is to: Do a physical examination while asking the client relevant questions 17. Upon establishing Mr. Regalado’s nursing needs, the next nursing approach would be to: Establish priority needs and implement appropriate interventions 18. Mr. Regalado says he has “trouble going to sleep”. In order to plan your nursing intervention you will: Ask him what he means by this statement 19. Mr. Regalado’s lower extremities are swollen and shiny. He has pitting pedal edema. When taking care of Mr. Regalado, which of the following interventions would be the most appropriate immediate nursing approach? Elevate lower extremities for postural drainage 20. Mr. Regalado will be discharged from your unit within the hour. Nursing actions when preparing a client for discharge include all EXCEPT: Walking the client to the hospital exit to ensure his safety Situation 5 – Nancy, mother of 2 young kids, 36 years old, had a mammogram and was told that she has breast cysts and that she may need surgery. This causes her anxiety as shown by increase

in her pulse and respiratory rate, sweating and feelings of tension. 21. Considering her level of anxiety, the nurse can best assist Nancy by: Giving her clear but brief information at the level of her understanding 22. Nancy blames God for her situation. She is easily provoked to tears and wants to be left alone, refusing to eat or talk to her family. A religious person before, she now refuses to pray or go to church stating that God has abandoned her. The nurse understands that Nancy is grieving for her self and is in the stage of: anger 23. The nurse visits Nancy and prods her to eat her food. Nancy replies “what’s the use? My time is running out.” The nurse’s best response would be: “You sound like you are giving up.” 24. The nurse feels sad about Nancy’s illness and tells her head nurse during the end of shift endorsement that “it’s unfair for Nancy to have cancer when she is still so young and with two kids”. The best response of the head nurse would be: Advise the nurse to “be strong and learn to control her feelings” 25. Realizing that she feels angry about Nancy’s condition, the nurse learns that being self-aware is a conscious process that she should do in any situation like this because: How the nurse thinks and feels affect her actions towards her client and her work Situation 6 – Mrs. Seva, 52 years old, asks you about possible problems regarding her elimination now that she is in the menopausal stage. 26. Instruction on health promotion regarding urinary elimination is important. Which would you include? Tell client to empty the bladder at each voiding 27. Mrs. Seva also tells the nurse that she is often constipated. Because she is aging, what physical changes predispose her to constipation? loss of tone of the smooth muscles of the colon 28. The nurse understands that one of these factors contributes to constipation: prolonged use of laxatives

29. Mrs. Seva talks about fear of being incontinent due to a prior experience of dribbling urine when laughing or sneezing and when she has a full bladder. Your most appropriate instruction would be to: teach client pelvic exercise to strengthen perineal muscles 30. Mrs. Seva asked for instructions for skin care for her mother who has urinary incontinence and is almost always in bed. Your instruction would focus on prevention of skin irritation and breakdown by: Thorough washing, rising and drying of skin area that get wet with urine Situation 7 – Using Maslow’s need theory, Airway, Breathing and Circulation are the physiological needs vital to life. The nurse’s knowledge and ability to identify and immediately intervene to meet these needs is important to save lives. 31. Which of these clients has a problem with the transport of oxygen from the lungs to the tissues: Theresa with anemia 32. You noted from the lab exams in the chart of M. Santos that he has reduced oxygen in the blood. This condition is called: Hypoxemia 33. You will do nasopharyngeal suctioning on Mr. Abad. Your guide for the length of insertion of the tubing for an adult would be: the distance from the tip of the nose to the tip of the ear lobe

34. While doing nasopharyngeal suctioning on Mr. Abad, the nurse can avoid trauma to the area by: Applying no suction while inserting the catheter 35. Myrna has difficulty breathing when on her back and must sit upright in bed to breath effectively and comfortably. The nurse documents this condition as: Orthopnea Situation 8 – You are assigned to screen for hypertension. Your task is to take blood pressure readings and you are informed about avoiding the common mistakes in BP taking that lead to false or inaccurate blood pressure readings. 36. When taking blood pressure reading the cuff should be:

deflated fully then immediately start second reading for same client 37. Chronic Obstructive Pulmonary Disease (COPD) in one of the leading causes of death world wide and is a preventable disease. The primary cause of COPD is cigarette smoking 38. In your health education class for clients with diabetes you teach them the areas for control of Diabetes which include all EXCEPT prevention nutrition 39. You teach your clients the difference between, Type I (IDDM) and Type II (NDDM) diabetes. Which of the following is true? Type II (NIDDM) is more common and is also preventable compared to Type I (IDDM) diabetes which is genetic in etiology 40. Lifestyle-related diseases in general share areas common risk factors. These are the following except: genetics Situation 9 – Nurse Rivera witnesses a vehicular accident near the hospital where she works. She decides to get involved and help the victims of the accident 41. Her priority nursing action would be to: Assess the extent of injuries incurred by the victims of the accident 42. Priority attention should be given to which of these clients? Ryan who has chest injury, is pale and with difficulty breathing 43. In the emergency room, Nurse Rivera is assigned to attend to the client with lacerations on the arms. While assessing the extent of the wound the nurse observes that the wound is now starting to bleed profusely. The most immediate nursing action would be to: Bandage the wound and elevate the arm 44. The nurse applies dressing on the bleeding site. This intervention is done to: Promote hemostasis 45. After the treatment, the client is sent home and asked to come back for follow-up care. Your

responsibilities when the client is to be discharged include the following EXCEPT: Instructing the client to see you after discharge for further assistance Situation 10 – While working in the clinic, a new client, Geline, 35 years old, arrives for her doctor’s appointment. As the clinic nurse, you are to assist the client fill up forms, gather data and make an assessment. 46. The purpose of your initial nursing interview is to: Understand her lifestyle, health needs and possible problems to develop a plan of care 47. While interviewing Geline, she starts to moan and doubles up in pain. She tells you that this pain occurs about an hour after taking black coffee without breakfast for a few weeks now. You will record this as follows: Client reported abdominal pain an hour after drinking black coffee for three weeks now. 48. Geline tells you that she drinks black coffee frequently within the day to “have energy and be wide awake” and she eats nothing for breakfast and eats strictly vegetable salads for lunch and dinner to lose weight. She has lost weight during the past two weeks. In planning a healthy balanced diet with Geline, you will: Discuss with her the importance of eating a variety of food from major food groups with plenty of fluids. 49. Geline tells you that she drinks 4-5 cups of black coffee and diet cola drinks. She also smokes up to a pack of cigarettes daily. She confesses that she is in her 2nd month of pregnancy but does not want to become fat that is why she limits her food intake. You warn or caution her about which of the following? Malnutrition and its possible effects on growth and development problems in the unborn fetus

50. Your health education plan for Geline stresses proper diet for a pregnant woman and the prevention of non-communicable diseases that are influenced by her lifestyle. These include the following EXCEPT: Cancer

Wednesday, January 23, 2008 StuffedNurse : NP2 practice exam NURSING PRACTICE II – Community Health Nursing and Care of the Mother and Child SITUATIONAL Situation 1 – Nurse Minette is an Independent Nurse Practitioner following-up referred clients in their respective homes. Here she handles a case of POSTPARTIAL MOTHER AND FAMILY focusing on HOME CARE. 1. Nurse Minette needs to schedule a first home visit to OB client Leah. When is a first home-care visit typically made? A. Within 4 days after discharge B. Within 24 hours after discharge C. Within 1 hour after discharge D. Within 1 week of discharge 2. Leah is developing constipation from being on bed rest. What measures would you suggest she take to help prevent this? A. Eat more frequent small meals instead of three large one daily B. Walk for at least half an hour daily to stimulate peristalsis C. Drink more milk, increased calcium intake prevents constipation D. Drink eight full glasses of fluid such as water daily 3. If you were Minette, which of the following actions would alert you that a new mother is entering a postpartal taking-hold phase?

A. She urges the baby to stay awake so that she can breast-feed him or her B. She tells you she was in a lot of pain all during labor C. She says that she has not selected a name for the baby as yet D. She sleeps as if exhausted from the effort of labor 4. At 6-week postpartum visit what should this postpartal mother’s fundic height be? A. Inverted and palpable at the cervix B. Six fingerbreadths below umbilicus C. No longer palpable on her abdomen D. One centimeter above the symphysis pubis 5. This postpartal mother wants to loose the weight she gained in pregnancy, so she is reluctant to increase her caloric intake for breast-feeding. By how much should a lactating mother increase her caloric intake during the first 6 months after birth? A. 350 kcal/day B. 500 kcal/day C. 200 kcal/day D. 1000 kcal/day Situation 2 – As the CPE is applicable for all professional nurse, the professional growth and development of Nurses with specialties shall be addressed by a Specialty Certification Council. The following questions apply to these special groups of nurses. 6. Which of the following serves as the legal basis and statute authority for the Board of Nursing to promulgate measures to effect the creation of a Specialty Certification Council and promulgate professional development programs for this group of nurseprofessionals?

A. R.A. 7610 B. P.D. 223 C. R.A. 9173 D. R.A. 7164 7. By force of law, therefore, the PRC-Board of Nursing released Resolution No. 14 Series of 1999 entitled: “Adoption of a Nursing Specialty Certification Program and Creation of Nursing Specialty Certification Council.” This rule-making power is called: A. Quasi-Judicial Power B. Regulatory Power C. Quasi-Legislative Power D. Executive/Promulgating Power 8. Under the PRC-Board of Nursing Resolution promulgating the adoption of a Nursing Specialty Certification Program and Council, which two (2) of the following serves as the strongest for its enforcement? (a) Advances made in Science and Technology have provided the climate for specialization in almost all aspects of human endeavor; and (b) As necessary consequence, there has emerged a new concept known as globalization which seeks to remove barriers in trade, industry and services imposed by the national laws of countries all over the world; and (c) Awareness of this development should impel the nursing sector to prepare our people in the services sector to meet the above challenge; and (d) Current trends of specialization in nursing practice recognized by the International Council of Nurses (ICN) of which the Philippines is a member for the benefit of the Filipino in terms of deepening and refining nursing practice and enhancing the quality of nursing care. A. b & c are strong justifications B. a & b are strong justifications

C. a & c are strong justifications D. a & d are strong justifications 9. Which of the following IS NOT a correct statement as regards Specialty Certification? A. The Board of Nursing intended to create the Nursing Specialty Certification Program as a means of perpetuating the creation of an elite force of Filipino Nurse Professionals. B. The Board of Nursing shall oversee the administration of the NSCP through the various Nursing Specialty Boards which will eventually be created C. The Board of Nursing at the time exercised their powers under R.A. 7164 in order to adopt the creation of the Nursing Specialty Certification Council and Program D. The Board of Nursing consulted nursing leaders of national nursing associations and other concerned nursing groups which later decided to ask a special group of nurses of the program for nursing specialty Certification. 10. The NSCC was created for the purpose of implementing the Nursing Specialty policy under the direct supervision and stewardship of the Board of Nursing. Who shall comprise the NSCC? A. A Chairperson who is the current President of the APO; a member from the Academe; and the last member coming from the Regulatory Board B. The chairperson and members of the Regulatory Board ipso facto acts as the CPE Council C. A Chairperson, chosen from among the Regulatory Board members; a Vice Chairperson appointed by the BON at-large; two other members also chosen at-large; and one representing the consumer group; D. A Chairperson who is the President of the Association from the Academe; a member from the Regulatory Board; and the last member coming from the APO

No answer..maybe a bonus question… Situation 3 –Nurse Anna is a new BSN graduate and has just passed her Licensure Examination for Nurses in the Philippines. She has likewise been hired as a new Community Health Nurse in one of the Rural Health Units in their City, which of the following conditions may be acceptable TRUTHS applied to Community Health Nursing Practice. 11. Which of the following is the primary focus of community health nursing practice? A. Cure of illnesses B. Prevention of illnesses C. Rehabilitation back to health D. Promotion of health 12. In community health nursing, which of the following is our unit of service as nurses? A. The community B. The extended members of every family C. The individual members of the Barangay D. The Family 13. A very important part of the Community Health Nursing Assessment Process includes: A. the application of professional judgment in estimating importance of facts to family and community B. evaluation structures and qualifications of health center team C. coordination with other sectors in relation to health concerns D. carrying out nursing procedures as per plan of action 14. In community health nursing it is important to take into account

the family health data coupled with an equally important need to perform ocular inspection of the area as activities which are powerful elements of: A. evaluation B. assessment C. implementation D. planning 15. The initial step in PLANNING process in order to engage in any nursing project or activities at the community level involves A. goal-setting B. monitoring C. evaluation of data D. provision of data Situation 4 – Please continue responding as a professional nurse in these other health situations through the following questions. 16. Transmission of HIV from an infected individual to another person occurs: A. Most frequently in nurses with needlesticks B. Only if there is a large viral load in the blood C. Most commonly as a result of sexual contact D. In all infants born to women with HIV infection 17. The medical record of a client reveals a condition in which the fetus cannot pass through the maternal pelvis. The nurse interprets this as: A. Contracted pelvis B. Maternal disproportion C. Cervical insufficiency D. Fetopelvic disproportion

18. The nurse would anticipate a cesarean birth for a client who has which infection present at the onset of labor? A. Herpes-simplex virus B. Human papilloma virus C. Hepatitis D. Toxoplasmosis 19. After a vaginal examination, the nurse determines that the client’s fetus is in an occiput posterior position. The nurse would anticipate that the client will have: A. A precipitous birth B. Intense back pain C. Frequent leg cramps D. Nausea and vomiting 20. The rationales for using a prostaglandin gel for a client prior to the induction of labor is to: A. Soften and efface the cervix B. Numb cervical pain receptors C. Prevent cervical lacerations D. Stimulate uterine contractions Situation 5 – Nurse Lorena is a Family Planning and Infertility Nurse Specialist and currently attends to FAMILY PLANNING CLIENTS AND INFERTILE COUPLES. The following conditions pertain to meeting the nursing needs of this particular population group. 21. Dina, 17 years old, asks you how a tubal ligation prevents pregnancy. Which would be the best answer? A. Prostaglandins released from the cut fallopian tubes can kill sperm B. Sperm can not enter the uterus because the cervical entrance is

blocked. C. Sperm can no longer reach the ova, because the fallopian tubes are blocked D. The ovary no longer releases ova as there is no where for them to go. 22. The Dators are a couple undergoing testing for infertility. Infertility is said to exist when: A. a woman has no uterus B. a woman has no children C. a couple has been trying to conceive for 1 year D. a couple has wanted a child for 6 months 23. Another client named Lilia is diagnosed as having endometriosis. This condition interferes with fertility because: A. endometrial implants can block the fallopian tubes B. the uterine cervix becomes inflamed and swollen C. the ovaries stop producing adequate estrogen D. pressure on the pituitary leads to decreased FSH levels 24. Lilia is scheduled to have a hysterosalphingogram. Which of the following instructions would you give her regarding this procedure? A. She will not be able to conceive for 3 months after the procedure B. The sonogram of the uterus will reveal any tumors present C. Many women experience mild bleeding as an after effect D. She may feel some cramping when the dye is inserted 25. Lilia’s cousin on the other hand, knowing nurse Lorena’s specialization asks what artificial insemination by donor entails. Which would be your best answer if you were Nurse Lorena? A. Donor sperm are introduced vaginally into the uterus or cervix B. Donor sperm are injected intra-abdominally into each ovary

C. Artificial sperm are injected vaginally to test tubal patency D. The husband’s sperm is administered intravenously weekly Situation 6 – There are other important basic knowledge in the performance of our task as Community Health Nurse in relation to IMMUNIZATION, these include: 26. The correct temperature to store vaccines in a refrigerator is: A. between -4 deg C and +8 deg C B. between 2 deg C and +8 deg C C. between -8 deg C and 0 deg C D. between -8 deg C and +4 deg C 27. Which of the following vaccines is not done by intramuscular (IM) injection? A. Measles vaccine B. DPT C. Hepa-B vaccine D. Tetanus toxoids 28. This vaccine content is derived from RNA recombinants. A. Measles B. Tetanus toxoids C. Hepatitis B vaccines D. DPT 29. This is the vaccine needed before a child reaches one (1) year in order for him/her to qualify as a :fully immunized child”. A. DPT B. Measles C. Hepatitis B D. BCG

30. Which of the following dose of tetanus toxoid is given to the mother to protect her infant from neonatal tetanus and likewise provide 10 years protection for the mother? A. Tetanus toxoid 3 B. Tetanus toxoid 2 C. Tetanus toxoid 1 D. Tetanus toxoid 4 Situation 7 – Records contain those comprehensive descriptions of patient’s health conditions and needs and at the same serve as evidences of every nurse’s accountability in the care giving process. Nursing records normally differ from institution to institution nonetheless they follow similar patterns of meeting needs for specific types of information. The following pertains to documentation/records management. 31. This special form is used when the patient is admitted to the unit. The nurse completes the information in this record particularly his/her basic personal data, current illness, previous health history, health history of the family, emotional profile, environmental history as well as physical assessment together with nursing diagnosis on admission. What do you call this record? A. Nursing Kardex B. Nursing Health History and Assessment Worksheet C. Medicine and Treatment Record D. Discharge Summary 32. These are sheets/forms which provide an efficient and time saving way to record information that must be obtained repeatedly at regular and/or short intervals of time. This does not replace the progress notes; instead this record of information on vital signs, intake and output, treatment, postoperative care, post partum care, and diabetic regimen, etc. This is used whenever specific

measurements or observations are needed to be documented repeatedly. What is this? A. Nursing Kardex B. Graphic Flow Sheets C. Discharge Summary D. Medicine and Treatment Record 33. These records show all medications and treatment provided on a repeated basis. What do you call this record? A. Nursing Health History and Assessment Worksheet B. Discharge Summary C. Nursing Kardex D. Medicine and Treatment Record 34. This flip-over card is usually kept in a portable file at the Nurse’s Station. It has 2-parts: the activity and treatment section and a nursing care plan section. This carries information about basic demographic data, primary medical diagnosis, current orders of the physician to be carried out by the nurse, written nursing care plan, nursing orders, scheduled tests and procedures, safety precautions in patient care and factors related to daily living activities. This record is used in the charge-of-shift reports or during the bedside rounds or walking rounds. What record is this? A. Discharge Summary B. Medicine and Treatment Record C. Nursing Health History and Assessment Worksheet D. Nursing Kardex 35. Most nurses regard this conventional recording of the date, time, and mode by which the patient leaves a healthcare unit but this record includes importantly, directs of planning for discharge that starts soon after the person is admitted to a healthcare institution. It is accepted that collaboration or multidisciplinary involvement (of all

members of the health team) in discharge results in comprehensive care. What do you call this? A. Discharge Summary B. Nursing Kardex C. Medicine and Treatment Record D. Nursing Health History and Assessment Worksheet Situation 8 – As Filipino Professional Nurses we must be knowledgeable about the Code of Ethics for Filipino Nurse and practice these by heart. The next questions pertain to this Code of Ethics. 36. Which of the following is TRUE about the Code of Ethics of Filipino Nurses? A. The Philippine Nurses Association for being the accredited professional organization was given the privilege to formulate a Code of Ethics for Nurses which the Board of Nursing promulgated B. Code for Nurses was first formulated in 1982 published in the Proceedings of the Third Annual Convention of the PNA House of Delegates C. The present code utilized the Code of Good Governance for the Professions in the Philippines D. Certificates of Registration of registered nurses may be revoked or suspended for violations of any provisions of the Code of Ethics. 37. Based on the Code of Ethics for Filipino Nurses, what is regarded as the hallmark of nursing responsibility and accountability? A. Human rights of clients, regardless of creed and gender B. The privilege of being a registered professional nurses C. Health, being a fundamental right of every individual D. Accurate documentation of actions and outcomes 38. Which of the following nurses behavior is regarded as a violation

of the Code of Ethics of Filipino Nurses? A. A nurse withholding harmful information to the family members of a patient B. A nurse declining commission sent by a doctor for her referral C. A nurse endorsing a person running for congress. D. Nurse Reviewers and/or nurse review center managers who pays a considerable amount of cash for reviewees who would memorize items from the licensure exams and submit these to them after the examination. 39. A nurse should be cognizant that professional programs for specialty certification by the Board of Nursing accredited through the: Professional Regulation Commission Nursing Specialty Certification Council Association of Deans of Philippine Colleges of Nursing Philippine Nurse Association 40. Mr. Santos, R.N. works in a nursing home, and he knows that one of his duties is to be an advocate for his patients. Mr. Santos knows a primary duty of an advocate is to; A. act as the patient’s legal representative B. complete all nursing responsibilities on time C. safeguard the well being of every patient D. maintain the patient’s right to privacy Situation 9 – Nurse Joanna works as an OB-Gyne Nurse and attends to several HIGH-RISK PREGNANCIES: Particular women with preexisting or Newly Acquired illness. The following conditions apply 41. Bernadette is a 22-year old woman. Which condition would make her more prone than others to developing a Candida infection during pregnancy?

A. Her husband plays golf 6 days a week B. She was over 35 when she became pregnant C. She usually drinks tomato juice for breakfast D. She has developed gestational diabetes 42. Bernadette develops a deep vein thrombosis following an auto accident and is prescribed heparin sub-Q. What should Joanna educate her about in regard to this? A. Some infants will be born with allergic symptoms to heparin B. Her infant will be born with scattered petechiae on his trunk C. Heparin can cause darkened skin in newborns D. Heparin does not cross placenta and so does not affect a fetus 43. The cousin of Bernadette with sickle-cell anemia alerted Joanna that she may need further instruction on prenatal care. What statement signifies this fact? A. I’ve stopped jogging so I don’t risk becoming dehydrated. B. I take an iron pill every day to help grow new red blood cells C. I am careful to drink at least eight glasses of fluid every day D. I understand why folic acid is important for red cell formation 44. Bernadette routinely takes acetylsalicylic acid (aspirin) for arthritis. Why should she limit or discontinue this toward the end of pregnancy? A. Aspirin can lead to deep vein thrombosis following birth B. Newborns develop a red rash from salicylate toxicity C. Newbors develop withdrawal headaches from salicylates D. Salicylates can lead to increased maternal bleeding at childbirth 45. Bernadette received a laceration on her leg from her automobile accident. Why are lacerations of lower extremities potentially more serious in pregnant women than others?

A. Lacerations can provoke allergic responses because of gonadothropic hormone B. Increased bleeding can occur from uterine pressure on leg veins C. A woman is less able to keep the laceration clean because of her fatigue D. Healing is limited during pregnancy, so these will not heal until after birth. Situation 10 – Still in your self-managed Child Health Nursing Clinic, you encounter these cases pertaining to the CARE OF CHILDREN WITH PULMONARY AFFECTIONS. 46. Josie brought her 3 months old child to your clinic because of cough and colds. Which of the following is your primary action? A. Give cotrimoxazole tablet or syrup B. Assess the patient using the chart on management of children with cough C. Refer to the doctor D. Teach the mother how to count her child’s breathing 47. In responding to the care concerns of children with sever disease, referral to the hospital is of the essence especially if the child manifests which of the following? A. Wheezing B. Stop feeding well C. Fast breathing D. Difficulty to awaken 48. Which of the following is the most important responsibility of a nurse in the prevention of unnecessary deaths from pneumonia and other severe diseases? A. Giving antibiotics

B. Taking of the temperature of the sick child C. Provision of Careful Assessment D. Weighing of the sick child 49. You were able to identify factors that lead to respiratory problems in the community where your health facility serve. Your primary role therefore in order to reduce morbidity due to pneumonia is to: A. Teach mothers how to recognize early signs and symptoms of pneumonia B. Make home visits to sick children C. Refer cases to hospitals D. Seek assistance and mobilize the BHWs to have a meeting with mothers 50. Which of the following is the principal focus of the CARI program of the Department of Health? A. Enhancement of health team capabilities B. Teach mothers how to detect signs and where to refer C. Mortality reduction through early detection D. Teach other community health workers how to assess patients. answer key np note: guys, i'll post the rationales later this month na lang ha, super hectic lang talaga my sched (^^,)

NURSING PRACTICE II – Community Health Nursing and Care of the Mother and Child SITUATIONAL Situation 1 – Nurse Minette is an Independent Nurse Practitioner following-up referred clients in their respective homes. Here she

handles a case of POSTPARTIAL MOTHER AND FAMILY focusing on HOME CARE. 1. Nurse Minette needs to schedule a first home visit to OB client Leah. When is a first home-care visit typically made? Within 24 hours after discharge 2. Leah is developing constipation from being on bed rest. What measures would you suggest she take to help prevent this? Drink eight full glasses of fluid such as water daily 3. If you were Minette, which of the following actions would alert you that a new mother is entering a postpartal taking-hold phase? She urges the baby to stay awake so that she can breast-feed him or her 4. At 6-week postpartum visit what should this postpartal mother’s fundic height be? No longer palpable on her abdomen 5. This postpartal mother wants to loose the weight she gained in pregnancy, so she is reluctant to increase her caloric intake for breast-feeding. By how much should a lactating mother increase her caloric intake during the first 6 months after birth? 500 kcal/day Situation 2 – As the CPE is applicable for all professional nurse, the professional growth and development of Nurses with specialties shall be addressed by a Specialty Certification Council. The following questions apply to these special groups of nurses.

6. Which of the following serves as the legal basis and statute authority for the Board of Nursing to promulgate measures to effect the creation of a Specialty Certification Council and promulgate professional development programs for this group of nurseprofessionals? R.A. 7164 7. By force of law, therefore, the PRC-Board of Nursing released Resolution No. 14 Series of 1999 entitled: “Adoption of a Nursing Specialty Certification Program and Creation of Nursing Specialty Certification Council.” This rule-making power is called: Quasi-Legislative Power 8. Under the PRC-Board of Nursing Resolution promulgating the adoption of a Nursing Specialty Certification Program and Council, which two (2) of the following serves as the strongest for its enforcement? (a) Advances made in Science and Technology have provided the climate for specialization in almost all aspects of human endeavor; and (b) As necessary consequence, there has emerged a new concept known as globalization which seeks to remove barriers in trade, industry and services imposed by the national laws of countries all over the world; and (c) Awareness of this development should impel the nursing sector to prepare our people in the services sector to meet the above challenge; and (d) Current trends of specialization in nursing practice recognized by the International Council of Nurses (ICN) of which the Philippines is a member for the benefit of the Filipino in terms of deepening and refining nursing practice and enhancing the quality of nursing care. a & b are strong justifications

9. Which of the following IS NOT a correct statement as regards Specialty Certification? The Board of Nursing intended to create the Nursing Specialty Certification Program as a means of perpetuating the creation of an elite force of Filipino Nurse Professionals. 10. The NSCC was created for the purpose of implementing the Nursing Specialty policy under the direct supervision and stewardship of the Board of Nursing. Who shall comprise the NSCC? A. A Chairperson who is the current President of the APO; a member from the Academe; and the last member coming from the Regulatory Board B. The chairperson and members of the Regulatory Board ipso facto acts as the CPE Council C. A Chairperson, chosen from among the Regulatory Board members; a Vice Chairperson appointed by the BON at-large; two other members also chosen at-large; and one representing the consumer group; D. A Chairperson who is the President of the Association from the Academe; a member from the Regulatory Board; and the last member coming from the APO No answer..maybe a bonus question… Situation 3 –Nurse Anna is a new BSN graduate and has just passed her Licensure Examination for Nurses in the Philippines. She has likewise been hired as a new Community Health Nurse in one of the Rural Health Units in their City, which of the following conditions may be acceptable TRUTHS applied to Community Health Nursing Practice. 11. Which of the following is the primary focus of community health nursing practice?

Promotion of health 12. In community health nursing, which of the following is our unit of service as nurses? The Family 13. A very important part of the Community Health Nursing Assessment Process includes: the application of professional judgment in estimating importance of facts to family and community 14. In community health nursing it is important to take into account the family health data coupled with an equally important need to perform ocular inspection of the area as activities which are powerful elements of: assessment 15. The initial step in PLANNING process in order to engage in any nursing project or activities at the community level involves goal-setting Situation 4 – Please continue responding as a professional nurse in these other health situations through the following questions. 16. Transmission of HIV from an infected individual to another person occurs: Most commonly as a result of sexual contact 17. The medical record of a client reveals a condition in which the fetus cannot pass through the maternal pelvis. The nurse interprets this as:

A. Fetopelvic disproportion 18. The nurse would anticipate a cesarean birth for a client who has which infection present at the onset of labor? Herpes-simplex virus 19. After a vaginal examination, the nurse determines that the client’s fetus is in an occiput posterior position. The nurse would anticipate that the client will have: Intense back pain 20. The rationales for using a prostaglandin gel for a client prior to the induction of labor is to: Soften and efface the cervix Situation 5 – Nurse Lorena is a Family Planning and Infertility Nurse Specialist and currently attends to FAMILY PLANNING CLIENTS AND INFERTILE COUPLES. The following conditions pertain to meeting the nursing needs of this particular population group. 21. Dina, 17 years old, asks you how a tubal ligation prevents pregnancy. Which would be the best answer? Sperm can no longer reach the ova, because the fallopian tubes are blocked 22. The Dators are a couple undergoing testing for infertility. Infertility is said to exist when: a couple has been trying to conceive for 1 year 23. Another client named Lilia is diagnosed as having endometriosis.

This condition interferes with fertility because: endometrial implants can block the fallopian tubes 24. Lilia is scheduled to have a hysterosalphingogram. Which of the following instructions would you give her regarding this procedure? She may feel some cramping when the dye is inserted 25. Lilia’s cousin on the other hand, knowing nurse Lorena’s specialization asks what artificial insemination by donor entails. Which would be your best answer if you were Nurse Lorena? Donor sperm are introduced vaginally into the uterus or cervix Situation 6 – There are other important basic knowledge in the performance of our task as Community Health Nurse in relation to IMMUNIZATION, these include: 26. The correct temperature to store vaccines in a refrigerator is: between 2 deg C and +8 deg C 27. Which of the following vaccines is not done by intramuscular (IM) injection? Measles vaccine 28. This vaccine content is derived from RNA recombinants. Hepatitis B vaccines 29. This is the vaccine needed before a child reaches one (1) year in order for him/her to qualify as a :fully immunized child”. Measles

30. Which of the following dose of tetanus toxoid is given to the mother to protect her infant from neonatal tetanus and likewise provide 10 years protection for the mother? Tetanus toxoid 4 Situation 7 – Records contain those comprehensive descriptions of patient’s health conditions and needs and at the same serve as evidences of every nurse’s accountability in the care giving process. Nursing records normally differ from institution to institution nonetheless they follow similar patterns of meeting needs for specific types of information. The following pertains to documentation/records management. 31. This special form is used when the patient is admitted to the unit. The nurse completes the information in this record particularly his/her basic personal data, current illness, previous health history, health history of the family, emotional profile, environmental history as well as physical assessment together with nursing diagnosis on admission. What do you call this record? Nursing Health History and Assessment Worksheet 32. These are sheets/forms which provide an efficient and time saving way to record information that must be obtained repeatedly at regular and/or short intervals of time. This does not replace the progress notes; instead this record of information on vital signs, intake and output, treatment, postoperative care, post partum care, and diabetic regimen, etc. This is used whenever specific measurements or observations are needed to be documented repeatedly. What is this? Graphic Flow Sheets 33. These records show all medications and treatment provided on a

repeated basis. What do you call this record? Medicine and Treatment Record 34. This flip-over card is usually kept in a portable file at the Nurse’s Station. It has 2-parts: the activity and treatment section and a nursing care plan section. This carries information about basic demographic data, primary medical diagnosis, current orders of the physician to be carried out by the nurse, written nursing care plan, nursing orders, scheduled tests and procedures, safety precautions in patient care and factors related to daily living activities. This record is used in the charge-of-shift reports or during the bedside rounds or walking rounds. What record is this? Nursing Kardex 35. Most nurses regard this conventional recording of the date, time, and mode by which the patient leaves a healthcare unit but this record includes importantly, directs of planning for discharge that starts soon after the person is admitted to a healthcare institution. It is accepted that collaboration or multidisciplinary involvement (of all members of the health team) in discharge results in comprehensive care. What do you call this? Discharge Summary Situation 8 – As Filipino Professional Nurses we must be knowledgeable about the Code of Ethics for Filipino Nurse and practice these by heart. The next questions pertain to this Code of Ethics. 36. Which of the following is TRUE about the Code of Ethics of Filipino Nurses? The present code utilized the Code of Good Governance for the Professions in the Philippines

37. Based on the Code of Ethics for Filipino Nurses, what is regarded as the hallmark of nursing responsibility and accountability? Accurate documentation of actions and outcomes 38. Which of the following nurses behavior is regarded as a violation of the Code of Ethics of Filipino Nurses? Nurse Reviewers and/or nurse review center managers who pays a considerable amount of cash for reviewees who would memorize items from the licensure exams and submit these to them after the examination. 39. A nurse should be cognizant that professional programs for specialty certification by the Board of Nursing accredited through the: Nursing Specialty Certification Council 40. Mr. Santos, R.N. works in a nursing home, and he knows that one of his duties is to be an advocate for his patients. Mr. Santos knows a primary duty of an advocate is to; maintain the patient’s right to privacy Situation 9 – Nurse Joanna works as an OB-Gyne Nurse and attends to several HIGH-RISK PREGNANCIES: Particular women with preexisting or Newly Acquired illness. The following conditions apply 41. Bernadette is a 22-year old woman. Which condition would make her more prone than others to developing a Candida infection during pregnancy? She has developed gestational diabetes

42. Bernadette develops a deep vein thrombosis following an auto accident and is prescribed heparin sub-Q. What should Joanna educate her about in regard to this? Heparin does not cross placenta and so does not affect a fetus 43. The cousin of Bernadette with sickle-cell anemia alerted Joanna that she may need further instruction on prenatal care. What statement signifies this fact? I take an iron pill every day to help grow new red blood cells 44. Bernadette routinely takes acetylsalicylic acid (aspirin) for arthritis. Why should she limit or discontinue this toward the end of pregnancy? Salicylates can lead to increased maternal bleeding at childbirth 45. Bernadette received a laceration on her leg from her automobile accident. Why are lacerations of lower extremities potentially more serious in pregnant women than others? Increased bleeding can occur from uterine pressure on leg veins Situation 10 – Still in your self-managed Child Health Nursing Clinic, you encounter these cases pertaining to the CARE OF CHILDREN WITH PULMONARY AFFECTIONS. 46. Josie brought her 3 months old child to your clinic because of cough and colds. Which of the following is your primary action? Assess the patient using the chart on management of children with cough A. Refer to the doctor B. Teach the mother how to count her child’s breathing

47. In responding to the care concerns of children with sever disease, referral to the hospital is of the essence especially if the child manifests which of the following? Difficulty to awaken 48. Which of the following is the most important responsibility of a nurse in the prevention of unnecessary deaths from pneumonia and other severe diseases? Provision of Careful Assessment 49. You were able to identify factors that lead to respiratory problems in the community where your health facility serve. Your primary role therefore in order to reduce morbidity due to pneumonia is to: Teach mothers how to recognize early signs and symptoms of pneumonia 50. Which of the following is the principal focus of the CARI program of the Department of Health? Mortality reduction through early detection

Wednesday, January 23, 2008 StuffedNurse: NP3 practice exam NURSING PRACTICE III – Care of Clients with Physiologic and Psychosocial Alterations (Part A)

SITUATIONAL Situation 1 – Concerted work efforts among members of the surgical team is essential to the success of the surgical procedure. 1. The sterile nurse or sterile personnel touch only sterile supplies and instruments. When there is a need for sterile supply which is not in the sterile field, who hands out these items by opening its outer cover? A. Circulating Nurse B. Anaesthesiologist C. Surgeon D. Nursing Aide 2. The OR team performs distinct roles for one surgical procedure to be accomplished within a prescribed time frame and deliver a standard patient outcome. While the surgeon performs the surgical procedure, who monitors the status of the client like urine output, blood loss? A. Scrub Nurse B. Surgeon C. Anaesthesiologist D. Circulating Nurse 3. Surgery schedules are communicated to the OR usually a day prior to the procedure by the nurse of the floor or ward where the patient is confined. For orthopedic cases, what department is usually informed to be present in the OR? A. Rehabilitation department B. Laboratory department C. Maintenance department D. Radiology department 4. Minimally invasive surgery is very much into technology. Aside from the usual surgical team, who else has to be present when a client undergoes laparoscopic surgery? A. Information technician B. Biomedical technician C. Electrician D. Laboratory technician 5. In massive blood loss, prompt replacement of compatible blood is crucial. What department needs to be alerted to coordinate closely with the patient’s family for immediate blood component therapy? A. Security Division B. Chaiplaincy

C. Social Service Section D. Pathology department Situation 2 – You are assigned in the Orthopedic Ward where clients are complaining of pain in varying degrees upon movement of body parts. 6. Troy is a one day post open reduction and internal fixation (ORIF) of the left hip and is in pain. Which of the following observation would prompt you to call the doctor? A. Dressing is intact but partially soiled B. Left foot is cold to touch and pedal pulse is absent C. Left leg in limited functional anatomic position D. BP 114/78, pulse of 82 beats/minute 7. There is an order of Demerol 50 mg I.M. now and every 6 hours p r n. You injected Demerol at 5 pm. The next dose of Demerol 50 mg I.M. is given: A. When the client asks for the next dose B. When the patient is in severe pain C. At 11 pm D. At 12 pm 8. You continuously evaluate the client’s adaptation to pain. Which of the following behaviors indicate appropriate adaptation? A. The client reports pain reduction and decreased activity B. The client denies existence of pain C. The client can distract himself during pain episodes D. The client reports independence from watchers 9. Pain in ortho cases may not be mainly due to the surgery. There might be other factors such as cultural or psychological that influence pain. How can you alter these factors as the nurse? A. Explain all the possible interventions that may cause the client to worry B. Establish trusting relationship by giving his medication on time C. Stay with the client during pain episodes D. Promote client’s sense of control and participation in control by listening to his concerns 10. In some hip surgeries, an epidural catheter for Fentanyl epidural analgesia is given. What is your nursing priority care in such a case? A. Instruct client to observe strict bed rest B. Check for epidural catheter drainage C. Administer analgesia through epidural catheter as prescribed D. Assess respiratory rate carefully

Situation 3 – Records are vital tools in any institution and should be properly maintained for specific use and time. 11. The patient’s medical record can work as a double edged sword. When can the medical record become the doctor’s/nurse’s worst enemy? A. When the record is voluminous B. When a medical record is subpoenaed in court C. When it is missing D. When the medical record is inaccurate, incomplete, and inadequate 12. Disposal of medical records in government hospitals/institutions must be done in close coordination with what agency? A. Department of Interior and Local Government (DILG) B. Metro Manila Development Authority (MMDA) C. Records Management Archives Office (RMAO) D. Department of Health (DOH) 13. In the hospital, when you need the medical record of a discharged patient for research you will request permission through: A. Doctor in charge B. The hospital director C. The nursing service D. Medical records section 14. You readmitted a client who was in another department a month ago. Since you will need the previous chart, from whom do you request the old chart? A. Central supply section B. Previous doctor’s clinic C. Department where the patient was previously admitted D. Medical records section 15. Records Management and Archives Office of the DOH is responsible for implementing its policies on record disposal. You know that your institution is covered by this policy if: A. Your hospital is considered tertiary B. Your hospital is in Metro Manila C. It obtained permit to operate from DOH D. Your hospital is PhilHealth accredited Situation 4 – In the OR, there are safety protocols that should be followed. The OR nurse should be well versed with all these to safeguard the safety and quality of patient delivery outcome.

16. Which of the following should be given highest priority when receiving patient in the OR? A. Assess level of consciousness B. Verify patient identification and informed consent C. Assess vital signs D. Check for jewelry, gown, manicure, and dentures 17. Surgeries like I and D (incision and drainage) and debridement are relatively short procedures but considered ‘dirty cases’. When are these procedures best scheduled? A. Last case B. In between cases C. According to availability of anaesthesiologist D. According to the surgeon’s preference 18. OR nurses should be aware that maintaining the client’s safety is the overall goal of nursing care during the intraoperative phase. As the circulating nurse, you make certain that throughout the procedure… A. the surgeon greets his client before induction of anesthesia B. the surgeon and anesthesiologist are in tandem C. strap made of strong non-abrasive materials are fastened securely around the joints of the knees and ankles and around the 2 hands around an arm board. D. Client is monitored throughout the surgery by the assistant anesthesiologist 19. Another nursing check that should not be missed before the induction of general anesthesia is: A. check for presence underwear B. check for presence dentures C. check patient’s ID D. check baseline vital signs 20. Some lifetime habits and hobbies affect postoperative respiratory function. If your client smokes 3 packs of cigarettes a day for the past 10 years, you will anticipate increased risk for: A. perioperative anxiety and stress B. delayed coagulation time C. delayed wound healing D. postoperative respiratory function Situation 5 – Nurses hold a variety of roles when providing care to a perioperative patient. 21. Which of the following role would be the responsibility of the scrub nurse? A. Assess the readiness of the client prior to surgery B. Ensure that the airway is adequate

C. Account for the number of sponges, needles, supplies, used during the surgical procedure. D. Evaluate the type of anesthesia appropriate for the surgical client 22. As a perioperative nurse, how can you best meet the safety need of the client after administering preoperative narcotic? A. Put side rails up and ask the client not to get out of bed B. Send the client to OR with the family C. Allow client to get up to go to the comfort room D. Obtain consent form 23. It is the responsibility of the pre-op nurse to do skin prep for patients undergoing surgery. If hair at the operative site is not shaved, what should be done to make suturing easy and lessen chance of incision infection? A. Draped B. Pulled C. Clipped D. Shampooed 24. It is also the nurse’s function to determine when infection is developing in the surgical incision. The perioperative nurse should observe for what signs of impending infection? A. Localized heat and redness B. Serosanguinous exudates and skin blanching C. Separation of the incision D. Blood clots and scar tissue are visible 25. Which of the following nursing interventions is done when examining the incision wound and changing the dressing? A. Observe the dressing and type and odor of drainage if any B. Get patient’s consent C. Wash hands D. Request the client to expose the incision wound Situation 6 – Carlo, 16 years old, comes to the ER with acute asthmatic attack. RR is 46/min and he appears to be in acute respiratory distress. 26. Which of the following nursing actions should be initiated first? A. Promote emotional support B. Administer oxygen at 6L/min C. Suction the client every 30 min D. Administer bronchodilator by nebulizer

27. Aminophylline was ordered for acute asthmatic attack. The mother asked the nurse, what is its indication, the nurse will say: A. Relax smooth muscles of the bronchial airway B. Promote expectoration C. Prevent thickening of secretions D. Suppress cough 28. You will give health instructions to Carlo, a case of bronchial asthma. The health instruction will include the following, EXCEPT: A. Avoid emotional stress and extreme temperature B. Avoid pollution like smoking C. Avoid pollens, dust, seafood D. Practice respiratory isolation 29. The asthmatic client asked you what breathing techniques he can best practice when asthmatic attack starts. What will be the best position? A. Sit in high-Fowler’s position with extended legs B. Sit-up with shoulders back C. Push on abdomen during exhalation D. Lean forward 30-40 degrees with each exhalation 30. As a nurse, you are always alerted to monitor status asthmaticus who will likely and initially manifest symptoms of: A. metabolic alkalosis B. respiratory acidosis C. respiratory alkalosis D. metabolic acidosis Situation 7 – Joint Commission on Accreditation of Hospital Organization (JCAHO) patient safety goals and requirements include the care and efficient use of technology in the OR and elsewhere in the healthcare facility. 31. As the head nurse in the OR, how can you improve the effectiveness of clinical alarm systems? A. Limit suppliers to a few so that quality is maintained B. Implement a regular inventory of supplies and equipment C. Adherence to manufacturer’s recommendation D. Implement a regular maintenance and testing of alarm systems 32. Overdosage of medication or anesthetic can happen even with the aid of technology like infusion pumps, sphygmomanometer and similar devices/machines. As a staff, how can you

improve the safety of using infusion pumps? A. Check the functionality of the pump before use B. Select your brand of infusion pump like you do with your cellphone C. Allow the technician to set the infusion pump before use D. Verify the flow rate against your computation 33. JCAHOs universal protocol for surgical and invasive procedures to prevent wrong site, wrong person, and wrong procedure/surgery includes the following, EXCEPT: A. Mark the operative site if possible B. Conduct pre-procedure verification process C. Take a video of the entire intra-operative procedure D. Conduct ‘time out’ immediately before starting the procedure 34. You identified a potential risk of pre-and postoperative clients. To reduce the risk of patient harm resulting from fall, you can implement the following, EXCEPT: A. Assess potential risk of fall associated with the patient’s medication regimen B. Take action to address any identified risks through Incident Report (IR) C. Allow client to walk with relative to the OR D. Assess and periodically reassess individual client’s risk for falling 35. As a nurse, you know you can improve on accuracy of patient’s identification by 2 patient identifiers, EXCEPT: A. identify the client by his/her wrist tag and verify with family members B. identify client by his/her wrist tag and call his/her by name C. call the client by his/her case and bed number D. call the patient by his/her name and bed number Situation 8 – Team efforts is best demonstrated in the OR. 36. If you are the nurse in charge for scheduling surgical cases, what important information do you need to ask the surgeon? A. Who is your internist B. Who is your assistant and anesthesiologist, and what is your preferred time and type of surgery? C. Who are your anesthesiologist, internist, and assistant D. Who is your anesthesiologist 37. In the OR, the nursing tandem for every surgery is: A. Instrument technician and circulating nurse B. Nurse anesthetist, nurse assistant, and instrument technician

C. Scrub nurse and nurse anesthetist D. Scrub and circulating nurses 38. While team effort is needed in the OR for efficient and quality patient care delivery, we should limit the number of people in the room for infection control. Who comprise this team? A. Surgeon, anesthesiologist, scrub nurse, radiologist, orderly B. Surgeon, assistants, scrub nurse, circulating nurse, anesthesiologist C. Surgeon, assistant surgeon, anesthesiologist, scrub nurse, pathologist D. Surgeon, assistant surgeon, anesthesiologist, intern, scrub nurse 39. When surgery is on-going, who coordinates the activities outside, including the family? A. Orderly/clerk B. Nurse Supervisor C. Circulating Nurse D. Anesthesiologist 40. The breakdown in teamwork is often times a failure in: A. Electricity B. Inadequate supply C. Leg work D. Communication Situation 9 – Colostomy is a surgically created anus. It can be temporary or permanent, depending on the disease condition. 41. Skin care around the stoma is critical. Which of the following is not indicated as a skin care barriers? A. Apply liberal amount of mineral oil to the area B. Use karaya paste and rings around the stoma C. Clean the area daily with soap and water before applying bag D. Apply talcum powder twice a day 42. What health instruction will enhance regulation of a colostomy (defecation) of clients? A. Irrigate after lunch everyday B. Eat fruits and vegetables in all three meals C. Eat balanced meals at regular intervals D. Restrict exercise to walking only 43. After ileostomy, which of the following condition is NOT expected? A. Increased weight

B. Irritation of skin around the stoma C. Liquid stool D. Establishment of regular bowel movement 44. The following are appropriate nursing interventions during colostomy irrigation, EXCEPT: A. Increase the irrigating solution flow rate when abdominal cramps is felt B. Insert 2-4 inches of an adequately lubricated catheter to the stoma C. Position client in semi-Fowler D. Hang the solution 18 inches above the stoma 45. What sensation is used as a gauge so that patients with ileostomy can determine how often their pouch should be drained? A. Sensation of taste B. Sensation of pressure C. Sensation of smell D. Urge to defecate Situation 10 – As a beginner in research, you are aware that sampling is an essential elements of the research process. 46. What does a sample group represent? A. Control group B. Study subjects C. General population D. Universe 47. What is the most important characteristic of a sample? A. Randomization B. Appropriate location C. Appropriate number D. Representativeness 48. Random sampling ensures that each subject has: A. Been selected systematically B. An equal chance of selection C. Been selected based on set criteria D. Characteristics that match other samples 49. Which of the following methods allows the use of any group of research subject? A. Purposive

B. Convenience C. Snow-ball D. Quota 50. You decided to include 5 barangays in your municipality and chose a sampling method that would get representative samples from each barangay. What should be the appropriate method ofor you to use in this care? A. Cluster sampling B. Random sampling C. Startified ampling D. Systematic sampling

answer key np3 note: later na po ung rationales! (^^,) NURSING PRACTICE III – Care of Clients with Physiologic and Psychosocial Alterations (Part A) SITUATIONAL Situation 1 – Concerted work efforts among members of the surgical team is essential to the success of the surgical procedure. 1. The sterile nurse or sterile personnel touch only sterile supplies and instruments. When there is a need for sterile supply which is not in the sterile field, who hands out these items by opening its outer cover? Circulating Nurse 2. The OR team performs distinct roles for one surgical procedure to be accomplished within a prescribed time frame and deliver a standard patient outcome. While the surgeon performs the surgical procedure, who monitors the status of the client like urine output, blood loss? Anaesthesiologist 3. Surgery schedules are communicated to the OR usually a day prior to the procedure by the nurse of the floor or ward where the patient is confined. For orthopedic cases, what department is usually informed to be present in the OR? Radiology department 4. Minimally invasive surgery is very much into technology. Aside from the usual surgical team, who else has to be present when a client undergoes laparoscopic surgery?

Biomedical technician 5. In massive blood loss, prompt replacement of compatible blood is crucial. What department needs to be alerted to coordinate closely with the patient’s family for immediate blood component therapy? Social Service Section Situation 2 – You are assigned in the Orthopedic Ward where clients are complaining of pain in varying degrees upon movement of body parts. 6. Troy is a one day post open reduction and internal fixation (ORIF) of the left hip and is in pain. Which of the following observation would prompt you to call the doctor? Left foot is cold to touch and pedal pulse is absent

7. There is an order of Demerol 50 mg I.M. now and every 6 hours p r n. You injected Demerol at 5 pm. The next dose of Demerol 50 mg I.M. is given: At 11 pm 8. You continuously evaluate the client’s adaptation to pain. Which of the following behaviors indicate appropriate adaptation? The client can distract himself during pain episodes 9. Pain in ortho cases may not be mainly due to the surgery. There might be other factors such as cultural or psychological that influence pain. How can you alter these factors as the nurse? Promote client’s sense of control and participation in control by listening to his concerns 10. In some hip surgeries, an epidural catheter for Fentanyl epidural analgesia is given. What is your nursing priority care in such a case? Assess respiratory rate carefully Situation 3 – Records are vital tools in any institution and should be properly maintained for specific use and time. 11. The patient’s medical record can work as a double edged sword. When can the medical record become the doctor’s/nurse’s worst enemy? When the medical record is inaccurate, incomplete, and inadequate

12. Disposal of medical records in government hospitals/institutions must be done in close coordination with what agency? Department of Health (DOH) 13. In the hospital, when you need the medical record of a discharged patient for research you will request permission through: Medical records section 14. You readmitted a client who was in another department a month ago. Since you will need the previous chart, from whom do you request the old chart? Medical records section 15. Records Management and Archives Office of the DOH is responsible for implementing its policies on record disposal. You know that your institution is covered by this policy if: It obtained permit to operate from DOH Situation 4 – In the OR, there are safety protocols that should be followed. The OR nurse should be well versed with all these to safeguard the safety and quality of patient delivery outcome. 16. Which of the following should be given highest priority when receiving patient in the OR? Verify patient identification and informed consent 17. Surgeries like I and D (incision and drainage) and debridement are relatively short procedures but considered ‘dirty cases’. When are these procedures best scheduled? Last case 18. OR nurses should be aware that maintaining the client’s safety is the overall goal of nursing care during the intraoperative phase. As the circulating nurse, you make certain that throughout the procedure… strap made of strong non-abrasive materials are fastened securely around the joints of the knees and ankles and around the 2 hands around an arm board. 19. Another nursing check that should not be missed before the induction of general anesthesia is: check baseline vital signs 20. Some lifetime habits and hobbies affect postoperative respiratory function. If your client smokes 3 packs of cigarettes a day for the past 10 years, you will anticipate increased risk for:

delayed wound healing Situation 5 – Nurses hold a variety of roles when providing care to a perioperative patient. 21. Which of the following role would be the responsibility of the scrub nurse? Account for the number of sponges, needles, supplies, used during the surgical procedure. 22. As a perioperative nurse, how can you best meet the safety need of the client after administering preoperative narcotic? Put side rails up and ask the client not to get out of bed

23. It is the responsibility of the pre-op nurse to do skin prep for patients undergoing surgery. If hair at the operative site is not shaved, what should be done to make suturing easy and lessen chance of incision infection? Clipped 24. It is also the nurse’s function to determine when infection is developing in the surgical incision. The perioperative nurse should observe for what signs of impending infection? Localized heat and redness 25. Which of the following nursing interventions is done when examining the incision wound and changing the dressing? Observe the dressing and type and odor of drainage if any Situation 6 – Carlo, 16 years old, comes to the ER with acute asthmatic attack. RR is 46/min and he appears to be in acute respiratory distress. 26. Which of the following nursing actions should be initiated first? Administer bronchodilator by nebulizer 27. Aminophylline was ordered for acute asthmatic attack. The mother asked the nurse, what is its indication, the nurse will say: Relax smooth muscles of the bronchial airway 28. You will give health instructions to Carlo, a case of bronchial asthma. The health instruction will include the following, EXCEPT: Practice respiratory isolation

29. The asthmatic client asked you what breathing techniques he can best practice when asthmatic attack starts. What will be the best position? Lean forward 30-40 degrees with each exhalation 30. As a nurse, you are always alerted to monitor status asthmaticus who will likely and initially manifest symptoms of: respiratory acidosis Situation 7 – Joint Commission on Accreditation of Hospital Organization (JCAHO) patient safety goals and requirements include the care and efficient use of technology in the OR and elsewhere in the healthcare facility. 31. As the head nurse in the OR, how can you improve the effectiveness of clinical alarm systems? Implement a regular maintenance and testing of alarm systems 32. Overdosage of medication or anesthetic can happen even with the aid of technology like infusion pumps, sphygmomanometer and similar devices/machines. As a staff, how can you improve the safety of using infusion pumps? Check the functionality of the pump before use 33. JCAHOs universal protocol for surgical and invasive procedures to prevent wrong site, wrong person, and wrong procedure/surgery includes the following, EXCEPT: Take a video of the entire intra-operative procedure 34. You identified a potential risk of pre-and postoperative clients. To reduce the risk of patient harm resulting from fall, you can implement the following, EXCEPT: Allow client to walk with relative to the OR 35. As a nurse, you know you can improve on accuracy of patient’s identification by 2 patient identifiers, EXCEPT: call the client by his/her case and bed number Situation 8 – Team efforts is best demonstrated in the OR. 36. If you are the nurse in charge for scheduling surgical cases, what important information do you need to ask the surgeon?

Who is your assistant and anesthesiologist, and what is your preferred time and type of surgery? 37. In the OR, the nursing tandem for every surgery is: Scrub and circulating nurses 38. While team effort is needed in the OR for efficient and quality patient care delivery, we should limit the number of people in the room for infection control. Who comprise this team?

Surgeon, assistants, scrub nurse, circulating nurse, anesthesiologist 39. When surgery is on-going, who coordinates the activities outside, including the family? Circulating Nurse 40. The breakdown in teamwork is often times a failure in: Communication Situation 9 – Colostomy is a surgically created anus. It can be temporary or permanent, depending on the disease condition. 41. Skin care around the stoma is critical. Which of the following is not indicated as a skin care barriers? Apply liberal amount of mineral oil to the area 42. What health instruction will enhance regulation of a colostomy (defecation) of clients? Eat balanced meals at regular intervals 43. After ileostomy, which of the following condition is NOT expected? Increased weight 44. The following are appropriate nursing interventions during colostomy irrigation, EXCEPT: Increase the irrigating solution flow rate when abdominal cramps is felt 45. What sensation is used as a gauge so that patients with ileostomy can determine how often their pouch should be drained? Sensation of pressure Situation 10 – As a beginner in research, you are aware that sampling is an essential elements of

the research process. 46. What does a sample group represent? General population 47. What is the most important characteristic of a sample? Representativeness 48. Random sampling ensures that each subject has: An equal chance of selection 49. Which of the following methods allows the use of any group of research subject? Convenience 50. You decided to include 5 barangays in your municipality and chose a sampling method that would get representative samples from each barangay. What should be the appropriate method ofor you to use in this care? Cluster sampling

Wednesday, January 23, 2008 StuffedNurse: NP4 practice exam NURSING PRACTICE IV – Care of Clients with Physiologic and Psychosocial Alterations (Part B)

MULTIPLE CHOICE

Situation 1 – Because of the serious consequences of severe burns, management requires a multidisciplinary approach. You have important responsibilities as a nurse.

1. While Sergio was lighting a barbecue grill with a lighter fluid, his shirt burns into flames. The most effective way to extinguish the flames with as little further damage as possible is to: A. log roll on the grass/ground B. slap the flames with his hands C. remove the burning clothes D. pour cold liquid over the flames 2. Once the flames are extinguished, it is most important to: A. cover Sergio with a warm blanket B. give him sips of water C. calculate the extent of his burns D. assess the Sergio’s breathing

3. Sergio is brought to Emergency Room after the barbecue grill accident. Based on the assessment of the physician, Sergio sustained superficial partial thickness burns on his trunk, right upper extremities and right lower extremities. His wife asks what that means? Your most accurate response would be: A. Structures beneath the skin are damage B. Dermis is partially damaged C. Epidermis and dermis are both damaged D. Epidermis is damaged 4. During the first 24 hours after the thermal injury, you should asses Sergio for: A. hypokalemia and hypernatremia B. hypokalemia and hyponatremia C. hyperkalemia and hyponatremia D. hyperkalemia and hypernatremia 5. Teddy, who sustained deep partial thickness and full thickness burns of the face, whole anterior chest and both upper extremities two days ago begins to exhibit extreme restlessness. You recognize that this most likely indicates that Teddy is developing: A. Cerebral hypoxia C. metabolic acidosis B. Hypervolemia D. Renal failure

Situation 2 – You are now working as a staff nurse in a general hospital. You have to be prepared to handle situations with ethico-legal and moral implifications.

6. You are in night duty in surgical ward. One of your patients Martin is a prisoner who sustained an abdominal gunshot wound. He is being guarded by policeman from the local police unit. During your rounds you heard a commotion. You saw the policeman trying to hit Martin. You asked why he was trying to hit Martin. He denied the matter. Which among the following activities will you do first? A. Write an accident report B. Call security officer and report the incident C. Call your nurse supervisor and report the incident D. Call the physician on duty

7. You are on morning duty in the medical ward. You have 10 patients assigned to you. During your endorsement rounds, you found out that one of your patients was not in bed. The patient next to him informed you that he went home without notifying the nurses. Which among the following will you do first? A. Make an incident report B. Call security to report the incident C. Wait for 2 hours before reporting D. Report the incident to your supervisor 8. You are on duty in the medical ward. You were asked to check the narcotics cabinet. You found out that what is on record does not tally with the drugs used. Which among the following will you do first? A. Write an incident report and refer the matter to the nursing director B. Keep your findings to yourself C. Report the matter to your supervisor D. Find out from the endorsement any patient who might have been given narcotics 9. You are on duty in the medical ward. The mother of your patient who is also a nurse, came running to the nurses station and informed you that Fiolo went into cardiopulmonary arrest. A. Start basic life support measures B. Call for the Code C. Bring the crash cart to the room D. Go to see Fiolo and assess for airway patency and breathing problems

10. You are admitting Jorge to the ward and you found out that he is positive for HIV. Which among the following will you do first? A. Take note of it and plan to endorse this to next shift B. Keep this matter to yourself C. Write an incident report D. Report the matter to your head nurse

Situation 3 - Colorectal cancer can affect old and younger people. Surgical procedures and other modes of treatment are done to ensure quality of life. You are assigned in the cancer institute to care of patients with this type of cancer.

11. Larry, 55 years old, who is suspected of having colorectal cancer, is admitted to the CI. After taking the history and vital signs the physician does which test as a screening test for colorectal cancer. A. Barium enema B. Carcinoembryonic antigen C. Annual digital rectal examination D. Proctosigmoidoscopy 12. To confirm his impression of colorectal cancer, Larry will require which diagnostic study? A. Carcinoembryonic antigen B. Proctosigmoidoscopy C. Stool hematologic test D. Abdominal computed tomography (CT) test 13. The following are risk factors for colorectal cancer, EXCEPT: A. Inflammatory bowels B. High fat, high fiber diet C. Smoking D. Genetic factors-familial adenomatous polyposis 14. Symptoms associated with cancer of the colon include: A. constipation, ascites and mucus in the stool B. diarrhea, heart burn and eructation C. blood in the stools, anemia, and “pencil shaped” stools D. anorexia, hematemesis, and increased peristalasis 15. Several days prior to bowel surgery, Larry may be given sulfasuxidine and neomycin primarily to:

A. promote rest of the bowel by minimizing peristalsis B. reduce the bacterial content of the colon C. empty the bowel of solid waste D. soften the stool by retaining water in the colon

Situation 4 – ENTEROSTOMAL THERAPY is now considered a specialty in nursing. You are participating in the OSTOMY CARE CLASS.

16. You plan to teach Fermin how to irrigate the colostomy when: A. The perineal wound heals And Fermin can sit comfortably on the commode B. Fermin can lie on the side comfortably, about the 3rd postoperative day C. The abdominal incision is closed and contamination is no longer a danger D. The stools starts to become formed, around the 7th postoperative day 17. When preparing to teach Fermin how to irrigate colostomy, you should plan to do the procedure: A. When Fermin would have normal bowel movement B. At least 2 hours before visiting hours C. Prior to breakfast and morning care D. After Fermin accepts alteration in body image 18. When observing a return demonstration of a colostomy irrigation, you know that more teaching is required if Fermin: A. Lubricates the tip of the catheter prior to inserting into the stoma B. Hangs the irrigating bag on the bathroom door cloth hook during fluid insertion C. Discontinues the insertion of fluid after only 500 ml of fluid has been instilled D. Clamps of the flow of fluid when felling uncomfortable 19. You are aware that teaching about colostomy care is understood when Fermin states, “I will contact my physician and report: A. If I have any difficulty inserting the irrigating tub into the stoma.” B. If I noticed a loss of sensation to touch in the stoma tissue.” C. The expulsion of flatus while the irrigating fluid is running out.” D. When mucus is passed from the stoma between the irrigations.” 20. You would know after teaching Fermin that dietary instruction for him is effective when he states, “It is important that I eat: A. Soft food that are easily digested and absorbed by my large intestines.”

B. Bland food so that my intestines do not become irritated.” C. Food low in fiber so that there is less stool.” D. Everything that I ate before the operation, while avoiding foods that cause gas.”

Situation 5 – Ensuring safety is one of your most important responsibilities. You will need to provide instructions and information to your clients to prevent complications.

21. Randy has chest tubes attached to a pleural drainage system. When caring for him you should: A. empty the drainage system at the end of the shift B. clamp the chest tube when suctioning C. palpate the surrounding areas for crepitus D. change the dressing daily using aseptic techniques 22. Fanny, came in from PACU after pelvic surgery. As Fanny’s nurse you know that the sign that would be indicative of a developing thrombophlebitis would be: A. a tender, painful area on the leg B. a pitting edema of the ankle C. a reddened area at the ankle D. pruritus on the calf and ankle 23. To prevent recurrent attacks on Terry who has acute glumerulonephritis, you should instruct her to: A. seek early treatment for respiratory infections B. take showers instead of tub bath C. continue to take the same restrictions on fluid intake D. avoid situations that involve physical activity 24. Herbert had a laryngectomy and he is now for discharge. He verbalized his concern regarding his laryngectomy tube being dislodged. What should you teach him first? A. Recognize that prompt closure of the tracheal opening may occur B. Keep calm because there is no immediate emergency C. Reinsert another tubing immediately D. Notify the physician at once 25. When caring for Larry after an exploratory chest surgery and pneumonectomy, your priority would be to maintain: A. supplementary oxygen B. ventilation exchange C. chest tube drainage

D. blood replacement

Situation 6 – Infection can cause debilitating consequences when host resistance is compromised and virulence of microorganisms and environmental factors are favorable. Infection control is one important responsibility of the nurse to ensure quality of care. 26. Honrad, who has been complaining of anorexia and feeling tired, develops jaundice, after a workup he is diagnosed of having Hepatitis A. his wife asks you about gamma globulin for herself and her household help. Your most appropriate response would be: A. “Don’t worry your husband’s type of hepatitis is no longer communicable” B. “Gamma globulin provides passive immunity for hepatitis B” C. “You should contact your physician immediately about getting gammaglobulin.” D. “A vaccine has been developed for this type of hepatitis” 27. Voltaire develops a nosocomial respiratory tract infection. He ask you what that means? Your best response would be: A. “You acquired the infection after you have been admitted to the hospital.” B. “This is a highly contagious infection requiring complete isolation.” C. “The infection you had prior to hospitalization flared up.” D. “As a result of medical treatment, you have acquired a secondary infection.’ 28. As a nurse you know that one of the complications that you have to watch out for when caring for Omar who is receiving total parenteral nutrition is: A. stomatitis B. hepatitis C. dysrhythmia D. infection 29. A solution used to treat Pseudomonas wound infection is: A. Dakin’s solution B. Half-strength hydrogen peroxide C. Acetic acid D. Betadine 30. Which of the following is the most reliable in diagnosing a wound infection? A. Culture and sensitivity B. Purulent drainage from a wound C. WBC count of 20,000/μL D. Gram stain testing

Situation 7 – As a nurse you need to anticipate the occurrence of complications of stroke so that life threatening situations can be prevented.

31. Wendy is admitted to the hospital with signs and symptoms of stroke. Her Glasgow Coma Scale is 6 on admission. A central venous catheter was inserted an I.V. infusion was started. As a nurse assigned to Wendy what will be your priority goal? A. Prevent skin breakdown B. Preserve muscle function C. Promote urinary elimination D. Maintain a patent airway 32. Knowing that for a comatose patient hearing is the last sense to be lost, as Judy’s nurse, what should you do? A. Tell her family that probably she can’t hear them B. Talk loudly so that Wendy can hear you C. Tell her family who are in the room not to talk D. Speak softly then hold her hands gently 33. Which among the following interventions should you consider as the highest priority when caring for June who has hemiparesis secondary to stroke? A. Place June on an upright lateral position B. Perform range of motion exercises C. Apply antiembolic stockings D. Use hand rolls or pillows for support 34. Ivy, age 40, was admitted to the hospital with a severe headache, stiff neck and photophobia. She was diagnosed with a subarachnoid hemorrhage secondary to ruptured aneurysm. While waiting for surgery, you can provide a therapeutic environment by doing which of the following? A. honoring her request for a television B. placing her bed near the window C. dimming the light in her room D. allowing the family unrestricted visiting privileges

35. When performing a neurologic assessment on Walter, you find that his pupils are fixed and dilated. This indicated that he: A. probably has meningitis B. is going to be blind because of trauma C. is permanently paralyzed

D. has received a significant brain injury

Situation 8 – With the improvement in life expectancies and the emphasis in the quality of life it is important to provide quality care to our older patients. There are frequently encountered situations and issues relevant to the older patients.

36. Hypoxia may occur in the older patients because of which of the following physiologic changes associated with aging. A. Ineffective airway clearance B. Decreased alveolar surfaced area C. Decreased anterior-posterior chest diameter D. Hyperventilation 37. The older patient is at higher risk for incontinence because of: A. dilated urethra B. increased glomerular filtration rate C. diuretic use D. decreased bladder capacity 38. Merle, age 86, is complaining of dizziness when she stands up. This may indicate: A. dementia B. a visual problem C. functional decline D. drug toxicity 39. Cardiac ischemia in an older patient usually produces: A. ST-T wave changes B. Very high creatinine kinase level C. Chest pain radiating to the left arm D. Acute confusion

40. The most dependable sign of infection in the older patient is: A. change in mental status B. fever C. pain D. decreased breath sounds with crackles

Situation 9 – A “disaster” is a large-scale emergency—even a small emergency left unmanaged may turn into a disaster. Disaster preparedness is crucial and is everybody’s business. There are agencies that are in charge of ensuring prompt response. Comprehensive Emergency Management (CEM) is an integrated approach to the management of emergency programs and activities for all four emergency phases (mitigation, preparedness, response, and recovery), for all types of emergencies and disasters (natural, man-made, and attack) and for all levels of government and the private sector. 41. Which of the four phases of emergency management is defined as “sustained action that reduces or eliminates long-term risk to people and property from natural hazards and their effects.”? A. Recovery B. Mitigation C. Response D. Preparedness 42. You are a community health nurse collaborating with the Red Cross and working with disaster relief following a typhoon which flooded and devastated the whole province. Finding safe housing for survivors, organizing support for the family, organizing counseling debriefing sessions and securing physical care are the services you are involved with. To which type of prevention are these activities included: A. Tertiary prevention B. Primary prevention C. Aggregate care prevention D. Secondary prevention 43. During the disaster you see a victim with a green tag, you know that the person: A. has injuries that are significant and require medical care but can wait hours with threat to life or limb B. has injuries that are life threatening but survival is good with minimal intervention C. indicates injuries that are extensive and chances of survival are unlikely even with definitive care D. has injuries that are minor and treatment can be delayed from hours to days

44. The term given to a category of triage that refers to life threatening or potentially life threatening injury or illness requiring immediate treatment: A. Immediate B. Emergent C. Non-acute D. Urgent

45. Which of the following terms refer to a process by which the individual receives education about recognition of stress reaction and management strategies for handling stress which may be instituted after a disaster? A. Clinical incident stress management B. Follow-up C. Defriefing D. Defusion Situation 10 – As a member of the health and nursing team you have a crucial role to play in ensuring that all the members participate actively is the various tasks agreed upon. 46. While eating his meal, Matthew accidentally dislodges his IV lines and bleeds. Blood oozes on the surface of the over-bed table. It is most appropriate that you instruct the housekeeper to clean the table with: A. Acetone B. Alcohol C. Ammonia D. Bleach 47. You are a member of the infection control team of the hospital. Based on a feedback during the meeting of the committee there is an increased incidence of pseudomonas infection in the Burn Unit (3 out of 10 patients had positive blood and wound culture). What is your priority activity? A. Establish policies for surveillance and monitoring B. Do data gathering about the possible sources of infection (observation, chart review, interview). C. Assign point persons who can implement policies. D. Meet with the nursing group working in the burn unit and discuss problem with them.

48. Part of your responsibility as a member of the diabetes core group is to get referrals from the various wards regarding diabetic patients needing diabetes education. Prior to discharge today, 4 patients are referred to you. How would you start prioritizing your activities? A. Bring your diabetes teaching kit and start your session taking into consideration their distance from your office B. Contact the nurse in-charge and find out from her the reason for the referral C. Determine their learning needs then prioritize D. Involve the whole family in the teaching class 49. You have been designated as a member of the task force to plan activities for the Cancer Consciousness Week. Your committee has 4 months to plan and implement the plan. You are assigned to contact the various cancer support groups in your hospital. What will be your priority

activity? A. Find out if there is a budget for this activity B. Clarify objectives of the activity with the task force before contacting the support groups C. Determine the VIPs and Celebrities who will be invited D. Find out how many support groups there are in the hospital and get the contact number of their president 50. You are invited to participate in the medical mission activity of your alumni association. In the planning stage everybody is expected to identify what they can do during the medical mission and what resources are needed. You thought it is also your chance to share what you can do for others. What will be your most important role where you can demonstrate the impact of nursing in health? A. Conduct health education on healthy life style B. Be a triage nurse C. Take the initial history and document findings D. Act as a coordinator np4 practice exam answer key note: i may not be able to post the rationales, its start of the review season and i am super busy, plus there are people na nangpipirate ng materials...so so so bad NURSING PRACTICE IV – Care of Clients with Physiologic and Psychosocial Alterations (Part B)

MULTIPLE CHOICE Situation 1 – Because of the serious consequences of severe burns, management requires a multidisciplinary approach. You have important responsibilities as a nurse.

1. While Sergio was lighting a barbecue grill with a lighter fluid, his shirt burns into flames. The most effective way to extinguish the flames with as little further damage as possible is to: log roll on the grass/ground 2. Once the flames are extinguished, it is most important to: assess the Sergio’s breathing 3. Sergio is brought to Emergency Room after the barbecue grill accident. Based on the assessment of the physician, Sergio sustained superficial partial thickness burns on his trunk, right upper extremities and right lower extremities. His wife asks what that means? Your most

accurate response would be: Epidermis and dermis are both damaged 4. During the first 24 hours after the thermal injury, you should asses Sergio for: hyperkalemia and hyponatremia 5. Teddy, who sustained deep partial thickness and full thickness burns of the face, whole anterior chest and both upper extremities two days ago begins to exhibit extreme restlessness. You recognize that this most likely indicates that Teddy is developing: metabolic acidosis

Situation 2 – You are now working as a staff nurse in a general hospital. You have to be prepared to handle situations with ethico-legal and moral implifications.

6. You are in night duty in surgical ward. One of your patients Martin is a prisoner who sustained an abdominal gunshot wound. He is being guarded by policeman from the local police unit. During your rounds you heard a commotion. You saw the policeman trying to hit Martin. You asked why he was trying to hit Martin. He denied the matter. Which among the following activities will you do first? Call your nurse supervisor and report the incident

7. You are on morning duty in the medical ward. You have 10 patients assigned to you. During your endorsement rounds, you found out that one of your patients was not in bed. The patient next to him informed you that he went home without notifying the nurses. Which among the following will you do first? Report the incident to your supervisor 8. You are on duty in the medical ward. You were asked to check the narcotics cabinet. You found out that what is on record does not tally with the drugs used. Which among the following will you do first? Report the matter to your supervisor 9. You are on duty in the medical ward. The mother of your patient who is also a nurse, came running to the nurses station and informed you that Fiolo went into cardiopulmonary arrest.

Bring the crash cart to the room 10. You are admitting Jorge to the ward and you found out that he is positive for HIV. Which among the following will you do first? Report the matter to your head nurse

Situation 3 - Colorectal cancer can affect old and younger people. Surgical procedures and other modes of treatment are done to ensure quality of life. You are assigned in the cancer institute to care of patients with this type of cancer.

11. Larry, 55 years old, who is suspected of having colorectal cancer, is admitted to the CI. After taking the history and vital signs the physician does which test as a screening test for colorectal cancer. Annual digital rectal examination 12. To confirm his impression of colorectal cancer, Larry will require which diagnostic study? Proctosigmoidoscopy 13. The following are risk factors for colorectal cancer, EXCEPT: Smoking 14. Symptoms associated with cancer of the colon include: blood in the stools, anemia, and “pencil shaped” stools 15. Several days prior to bowel surgery, Larry may be given sulfasuxidine and neomycin primarily to: reduce the bacterial content of the colon

Situation 4 – ENTEROSTOMAL THERAPY is now considered a specialty in nursing. You are participating in the OSTOMY CARE CLASS.

16. You plan to teach Fermin how to irrigate the colostomy when: The stools starts to become formed, around the 7th postoperative day 17. When preparing to teach Fermin how to irrigate colostomy, you should plan to do the

procedure: When Fermin would have normal bowel movement 18. When observing a return demonstration of a colostomy irrigation, you know that more teaching is required if Fermin: Hangs the irrigating bag on the bathroom door cloth hook during fluid insertion 19. You are aware that teaching about colostomy care is understood when Fermin states, “I will contact my physician and report: If I have any difficulty inserting the irrigating tub into the stoma.” 20. You would know after teaching Fermin that dietary instruction for him is effective when he states, “It is important that I eat: A. Everything that I ate before the operation, while avoiding foods that cause gas.”

Situation 5 – Ensuring safety is one of your most important responsibilities. You will need to provide instructions and information to your clients to prevent complications.

21. Randy has chest tubes attached to a pleural drainage system. When caring for him you should: palpate the surrounding areas for crepitus 22. Fanny, came in from PACU after pelvic surgery. As Fanny’s nurse you know that the sign that would be indicative of a developing thrombophlebitis would be: a tender, painful area on the leg 23. To prevent recurrent attacks on Terry who has acute glumerulonephritis, you should instruct her to: seek early treatment for respiratory infections

24. Herbert had a laryngectomy and he is now for discharge. He verbalized his concern regarding his laryngectomy tube being dislodged. What should you teach him first? Notify the physician at once 25. When caring for Larry after an exploratory chest surgery and pneumonectomy, your priority would be to maintain:

ventilation exchange Situation 6 – Infection can cause debilitating consequences when host resistance is compromised and virulence of microorganisms and environmental factors are favorable. Infection control is one important responsibility of the nurse to ensure quality of care. 26. Honrad, who has been complaining of anorexia and feeling tired, develops jaundice, after a workup he is diagnosed of having Hepatitis A. his wife asks you about gamma globulin for herself and her household help. Your most appropriate response would be: “You should contact your physician immediately about getting gammaglobulin.” 27. Voltaire develops a nosocomial respiratory tract infection. He ask you what that means? Your best response would be: “You acquired the infection after you have been admitted to the hospital.” 28. As a nurse you know that one of the complications that you have to watch out for when caring for Omar who is receiving total parenteral nutrition is: infection 29. A solution used to treat Pseudomonas wound infection is: Dakin’s solution 30. Which of the following is the most reliable in diagnosing a wound infection? WBC count of 20,000/μL

Situation 7 – As a nurse you need to anticipate the occurrence of complications of stroke so that life threatening situations can be prevented.

31. Wendy is admitted to the hospital with signs and symptoms of stroke. Her Glasgow Coma Scale is 6 on admission. A central venous catheter was inserted an I.V. infusion was started. As a nurse assigned to Wendy what will be your priority goal? Maintain a patent airway 32. Knowing that for a comatose patient hearing is the last sense to be lost, as Judy’s nurse, what should you do? Speak softly then hold her hands gently

33. Which among the following interventions should you consider as the highest priority when caring for June who has hemiparesis secondary to stroke? Perform range of motion exercises 34. Ivy, age 40, was admitted to the hospital with a severe headache, stiff neck and photophobia. She was diagnosed with a subarachnoid hemorrhage secondary to ruptured aneurysm. While waiting for surgery, you can provide a therapeutic environment by doing which of the following? dimming the light in her room

35. When performing a neurologic assessment on Walter, you find that his pupils are fixed and dilated. This indicated that he: has received a significant brain injury

Situation 8 – With the improvement in life expectancies and the emphasis in the quality of life it is important to provide quality care to our older patients. There are frequently encountered situations and issues relevant to the older patients.

36. Hypoxia may occur in the older patients because of which of the following physiologic changes associated with aging. Decreased alveolar surfaced area 37. The older patient is at higher risk for incontinence because of: decreased bladder capacity 38. Merle, age 86, is complaining of dizziness when she stands up. This may indicate: functional decline 39. Cardiac ischemia in an older patient usually produces: Acute confusion

40. The most dependable sign of infection in the older patient is: change in mental status

Situation 9 – A “disaster” is a large-scale emergency—even a small emergency left unmanaged may turn into a disaster. Disaster preparedness is crucial and is everybody’s business. There are agencies that are in charge of ensuring prompt response. Comprehensive Emergency Management (CEM) is an integrated approach to the management of emergency programs and activities for all four emergency phases (mitigation, preparedness, response, and recovery), for all types of emergencies and disasters (natural, man-made, and attack) and for all levels of government and the private sector. 41. Which of the four phases of emergency management is defined as “sustained action that reduces or eliminates long-term risk to people and property from natural hazards and their effects.”? Mitigation 42. You are a community health nurse collaborating with the Red Cross and working with disaster relief following a typhoon which flooded and devastated the whole province. Finding safe housing for survivors, organizing support for the family, organizing counseling debriefing sessions and securing physical care are the services you are involved with. To which type of prevention are these activities included: Tertiary prevention 43. During the disaster you see a victim with a green tag, you know that the person: has injuries that are minor and treatment can be delayed from hours to days

44. The term given to a category of triage that refers to life threatening or potentially life threatening injury or illness requiring immediate treatment: Emergent 45. Which of the following terms refer to a process by which the individual receives education about recognition of stress reaction and management strategies for handling stress which may be instituted after a disaster? Defusion Situation 10 – As a member of the health and nursing team you have a crucial role to play in ensuring that all the members participate actively is the various tasks agreed upon. 46. While eating his meal, Matthew accidentally dislodges his IV lines and bleeds. Blood oozes on the surface of the over-bed table. It is most appropriate that you instruct the housekeeper to clean the table with:

Bleach 47. You are a member of the infection control team of the hospital. Based on a feedback during the meeting of the committee there is an increased incidence of pseudomonas infection in the Burn Unit (3 out of 10 patients had positive blood and wound culture). What is your priority activity? Do data gathering about the possible sources of infection (observation, chart review, interview). 48. Part of your responsibility as a member of the diabetes core group is to get referrals from the various wards regarding diabetic patients needing diabetes education. Prior to discharge today, 4 patients are referred to you. How would you start prioritizing your activities? Determine their learning needs then prioritize 49. You have been designated as a member of the task force to plan activities for the Cancer Consciousness Week. Your committee has 4 months to plan and implement the plan. You are assigned to contact the various cancer support groups in your hospital. What will be your priority activity? Clarify objectives of the activity with the task force before contacting the support groups 50. You are invited to participate in the medical mission activity of your alumni association. In the planning stage everybody is expected to identify what they can do during the medical mission and what resources are needed. You thought it is also your chance to share what you can do for others. What will be your most important role where you can demonstrate the impact of nursing in health? Conduct health education on healthy life style

Wednesday, January 23, 2008

StuffedNurse: NP5 practice exam NURSING PRACTICE V- Care of Clients with physiologic and Psychosocial Alterations (Part C). SITUATIONAL Situation 1 – Jimmy developed this goal for hospitalization. “To get a handle on my nervousness.” The nurse is going to collaborate with him to reach his goal. Jimmy was admitted to the hospital because he called his therapist that he planned to asphyxiate himself with exhaust from his car but frightened instead. He realized he needed help. 1. The nurse recognized that Jimmy had conceptualized his problem and the next priority goal in the care plan is: A. help the client find meaning in his experience B. help the client to plan alternatives C. help the client cope with the present problem D. help the client to communicate

2. The nurse is guided that Jimmy is aware of his concerns of the “here and now” when he crossed out which item from this “list of what to know”.

A. anxiety laden unconscious conflicts B. subjective idea of the range of mild to severe anxiety C. early signs of anxiety D. physiologic indices of anxiety 3. While Jimmy was discussing the signs and symptoms of anxiety with his nurse, he recognized that complete disruption of the ability

to perceive occurs in: A. panic state of anxiety B. severe anxiety C. moderate anxiety D. mild anxiety 4. Jimmy initiates independence and takes an active part in his self care with the following EXCEPT: A. agreeing to contact the staff when he is anxious B. becoming aware of the conscious feeling C. assessing need for medication and medicating himself D. writing out a list of behaviors that he identified as anxious 5. The nurse notes effectiveness of interventions in using subjective and objective data in the: A. initial plans or orders B. database C. problem list D. progress notes Situation 2 – A research study was undertaken in order to identify and analyze a disabled boy’s coping reaction pattern during stress. 6. This study which is an in depth study of one boy is a: A. case study B. longitudinal study C. cross-sectional study D. evaluative study 7. The process recording was the principal tool for data collection. Which of the following is NOT a part of a process recording?

A. Non verbal narrative account B. Analysis and interpretation C. Audio-visual recording D. Verbal narrative account 8. Which of these does NOT happen in a descriptive study? A. Exploration of relationship between two or more phenomena. B. Explanation of relationship between two or more phenomena. C. Investigation of a phenomenon in real life context. D. Manipulation of variable 9. The investigator also provided the nursing care of the subject. The investigator is referred to as a/an: A. Participant-observer B. Observer researcher C. Caregiver D. Advocate 10. To ensure reliability of the study, the investigator’s analysis and interpretations were: A. subjected to statistical treatment B. correlated with a list of coping behaviors C. subjected to an inter-observer agreement D. scored and compared standard criteria

Situation 3 – During the morning endorsement, the outgoing nurse informed the nursing staff that Regina, 35 years old, was given Flurazepam (Dalmane) 15mg at 10:00pm because she had trouble going to sleep. Before approaching Regina, the nurse read the observation of the night nurse. 11. Which of the following approaches of the nurse validates the data

gathered? A. “I learned that you were up till ten last night, tell me what happened before you were finally able to sleep and how was your sleep?” B. “Hmm.. You look like you had a very sound sleep. That pill you were given last night is effective isn’t it?” C. “Regina, did you sleep well?” D. “Regina, how are you?” 12. Regina is a high school teacher. Which of these information LEAST communicate attention and care for her needs for information about her medicine? A. Guided by a medication teaching plan, go over with her the purpose, indications and special instructions, about the medication and provide her a checklist B. Provide a drug literature. C. Have an informal conversation about the medication and its effects D. Ask her what time she would like to watch the informative video about the medication. 13. The nurse engages Regina in the process of mutual inquiry to provide an opportunity for Regina to: A. face emerging problems realistically B. conceptualize her problem C. cope with her present problem D. perceive her participation in an experience !4. Which of these responses indicate that Regina needs further discussion regarding special instructions? A. “I have to take this medicine judiciously” B. “I know I will stop taking the medicine when there is advice from

the doctor for me to discontinue.” C. “I will inform you and the doctor any untoward reactions I have.” D. “I like taking this sleeping pill. It solves my problem of insomnia. I wish I can take it for life.” 15. Regina commits to herself that she understood and will observe all the medicine precautions by: A. affixing her signature to the teaching plan that she has understood the nurse B. committing what she learned to her memory C. verbally agreeing with the nurse D. relying on her husband to remember the precautions Situation 4 – The nurse-patient relationship is a modality through which the nurse meets the client’s needs. 16. The nurse’s most unique tool in working with the emotionally ill client is his/her A. theoretical knowledge B. personality make up C. emotional reactions D. communication skills 17. The psychiatric nurse who is alert to both the physical and emotional needs of clients is working from the philosophical framework that states: A. All behavior is meaningful, communicating a message or a need. B. Human beings are systems of interdependent and interrelated parts. C. Each individual has the potential for growth and change in the direction of positive mental health. D. There is a basic similarity among all human beings.

18. One way to increase objectivity in dealing with one’s fears and anxieties is through the process of: A. observation B. intervention C. validation D. collaboration 19. All of the following responses are non therapeutic. Which is the MOST direct violation of the concept, congruence of behavior? A. Responding in a punitive manner to the client. B. Rejecting the client as a unique human being C. Tolerating all behavior in the client. D. Communicating ambivalent messages to the client. 20. The mentally ill person responds positively to the nurse who is warm and caring. This is a demonstration of the nurse’s role as: A. counselor B. mother surrogate C. therapist D. socializing agent Situation 5 – The nurse engages the client in a nurse-patient interaction. 21. The best time to inform the client about terminating the nursepatient relationship is: A. when the client asks how long the relationship would be B. during the working phase C. towards the end of the relationship D. at the start of the relationship 22. The client says, “I want to tell you something but can you

promise that you will keep this a secret?” A therapeutic response of the nurse is: A. “Yes, our interaction is confidential provided the information you tell me is not detrimental to your safety.” B. “Of course yes, this is just between you and me. Promise!” C. “Yes, it is my principle to uphold my client’s rights.” D. “Yes, you have the right to invoke confidentiality of our interaction.” 23. When the nurse respects the client’s self-disclosure, this is a gauge for the nurse’s: A. trustworthiness B. loyalty C. integrity D. professionalism 24. Rapport has been established in the nurse-client relationship. The client asks to visit the nurse after his discharge. The appropriate response of the nurse would be: A. “The best time to talk is during the nurse-client interaction time. I am committed to have this time available for us while you are at the hospital and ends after your discharge.” B. “Yes, If you keep it confidential, this is part of privileged communication.” C. “I am committed for your care.” D. “I am sorry, though I would want to, it is against hospital policy.” 25. The client has not been visited by relatives for months. He gives a telephone number and requests the nurse to call. An appropriate action of the nurse would be: A. Inform the attending physician about the request of the client. B. Assist the client to bring his concern to the attention of the social

worker. C. “Here (gives her mobile phone). You may call this number now”. D. Ask the client what is the purpose of contacting his relatives. Situation 6 – Camila, 25 years old, was reported to be gradually withdrawing and isolating herself from friends and family members. She became neglectful of her personal hygiene. She was observed to be talking irrelevantly and incoherently. She was diagnosed as schizophrenia disorder. 26. The past history of Camila would most probably reveal that her premorbid personality is: A. schizoid B. extrovert C. ambivert D. cycloid 27. Camila refuses to relate with to others because she: A. is irritable B. feels superior of others C. anticipates rejection D. is depressed 28. Which of the following disturbances in interpersonal relationships MOST often predispose to the development of schizophrenia? A. Lack of participation in peer groups B. Faulty family atmosphere and interaction C. Extreme rebellion towards authority figures D. Solo parenting 29. Camila’s indifference toward the environment is a compensatory behavior to overcome:

A. Guilt feelings B. Ambivalence C. Narcissistic behavior D. Insecurity feelings 30. Schizophrenia is a/an: A. anxiety disorder B. neurosis C. psychosis D. personality disorder Situation 7 – Salome, 80 year old widow, has been observed to be irritable, demanding and speaking louder than usual. She would prefer to be alone and take her meals by herself, minimize receiving visitors at home and no longer bothers to answer telephone calls because of deterioration of hearing. She was brought by her daughter to the Geriatric clinic for assessment and treatment. 31. The nurse counsels Salome’s daughter that Salome’s becoming very loud and tendency to become aggressive is a/an: A. beginning indifference to the world around her B. attempt to maintain authoritative role C. overcompensation for hearing loss D. behavior indicative of unresolved repressed conflict of the past 32. A nursing diagnosis for Salome is: A. sensory deprivation B. social isolation C. cognitive impairment D. ego despair 33. The nurse will assist Salome and her daughter to plan a goal which is for Salome to:

A. adjust to the loss of sensory and perceptual function B. participate in conversation and other social situations C. accept the steady loss of hearing that occurs with aging D. increase her self-esteem to maintain her authoritative role 34. The daughter understood, the following ways to assist Salome meet her needs and avoiding which of the following: A. Using short simple sentences B. Speaking distinctly and slowly C. Speaking at eye level and having the client’s attention D. Allowing her to take her meals alone 35. Salome was fitted a hearing aid. She understood the proper use and wear of this device when she says that the battery should be functional, the device is turned on and adjusted to a: A. therapeutic level B. comfortable level C. prescribed level D. audible level Situation 8 – For more than a month now, Cecilia is persistently feeling restless, worried and feeling as if something dreadful is going to happen. She fears being alone in places and situations where she thinks that no one might come to rescue her just in case something happens to her. 36. Cecilia is demonstrating: A. acrophobia B. claustrophobia C. agoraphobia D. xenophobia

37. Cecilia’s problem is that she always sees and thinks negative things hence she is always fearful. Phobia is a symptom described as: A. organic B. psychosomatic C. psychotic D. neurotic 38. Cecilia has a lot of irrational thoughts: The goal of therapy is to modify her: A. communication B. cognition C. observation D. perception 39. Cognitive therapy is indicated for Cecilia when she is already able to handle anxiety reactions. Which of the following should the nurse implement? A. assist her in recognizing irrational beliefs and thoughts B. help find meaning in her behavior C. provide positive reinforcement for acceptable behavior D. Administer anxiolytic drug 40. After discharge, which of these behaviors indicate a positive result of being able to overcome her phobia? A. She reads a book in the public library B. She drives alone along the long expressway. C. She watches television with the family in the recreation room D. She joins an art therapy group Situation 9 – It is the first day of clinical experience of nursing students at the Psychiatry Ward. During the orientation, the nurse emphasizes that the team members including nursing students are

legally responsible to safeguard patient’s records from loss or destruction or from people not authorized to read it. 41. It is unethical to tell one’s friends and family members data about patients because doing so is a violation of patients’ rights to: A. Informed consent B. Confidentiality C. Least restrictive environment D. Civil liberty 42. The nurse must see to it that the written consent of mentally ill patients must be taken from: A. Doctor B. Social worker C. Parents or legal guardian D. Law enforcement authorities 43. In an extreme situation and when no other resident or intern is available, should a nurse receive telephone orders, the order has to be correctly written and signed by the physician within: A. 24 hours B. 36 hours C. 48 hours D. 12 hours 44. The following are SOAP (Subjective – Objective – Analysis – Plan) statements on a problem: Anxiety about diagnosis. What is the objective data? A. Relate patient’s feelings to physician; initiate and encourage her to verbalize her fears; give emotional support by spending more time with patient; continue to make necessary explanations regarding diagnostic tests.

B. Has periods of crying; frequently verbalizes fear of what diagnostic tests will reveal C. Anxiety due to unknown D. “I’m so worried about what else they’ll find wrong with me.” 45. Nursing care plans provide very meaningful data for the patient profile and initial plan because the focus is on the: A. Summary of chronological notations made by individual health team members B. Identification of patient’s responses to medical diagnoses and treatment C. Patient’s responses to health and illness as a total person in interaction with the environment D. Step by step procedures for the management of common problems Situation 10 – Marie is 5½ years old and described by the mother as bedwetting at night. 46. Which of the following is the MOST common physiological cause of night bed wetting? A. deep sleep factors B. abnormal bladder development C. infections D. familial and genetic factors 47. All of the following, EXCEPT one compromise the concepts of behavior therapy program. A. reward and punishment B. extinction C. learning D. placebo as a form of treatment 48. To help Marie who bed wets at night practice acceptable and

appropriate behavior, it is important for the parents to be consistent with the following approaches EXCEPT: A. discipline with a kind attitude B. matter of fact in handling the behavior C. sympathize for the child D. be loving yet firm 49. A therapeutic verbal approach that communicates strong disapproval is: A. “You are supposed to get up and go in the toilet when you feel you have to go and did not. The next time you bed wet, I’ll tell your friends and hang your sheets out the window for them to see.” B. “You are supposed to get up and go in the toilet when you feel you have to go and did not. I expect you to from now on without fail.” C. “If you bed wet, you will change your bed linen and wash the sheets.” D. “If you don’t make an effort to control your bedwetting, I’d be upset and disappointed.” 50. During your conference, the parent inquires how to motivate Marie to be dry in the morning. Your response which is an immediate intervention would be: A. Give a star each time she wakes up dry and every set of five stars, give a prize. B. Tokens make her materialistic at an early age. Give praise and hugs occasionally. C. What does your child want that you can give every time he/she wakes up dry in the morning? D. Promise him/her a long awaited vacation after school is over. NP5 practice exam answer key note: rationales will be for classroom discussion to avoid piracy

NURSING PRACTICE V- Care of Clients with physiologic and Psychosocial Alterations (Part C). SITUATIONAL Situation 1 – Jimmy developed this goal for hospitalization. “To get a handle on my nervousness.” The nurse is going to collaborate with him to reach his goal. Jimmy was admitted to the hospital because he called his therapist that he planned to asphyxiate himself with exhaust from his car but frightened instead. He realized he needed help. 1. The nurse recognized that Jimmy had conceptualized his problem and the next priority goal in the care plan is: help the client to plan alternatives

2. The nurse is guided that Jimmy is aware of his concerns of the “here and now” when he crossed out which item from this “list of what to know”.

subjective idea of the range of mild to severe anxiety 3. While Jimmy was discussing the signs and symptoms of anxiety with his nurse, he recognized that complete disruption of the ability to perceive occurs in: panic state of anxiety 4. Jimmy initiates independence and takes an active part in his self care with the following EXCEPT:

agreeing to contact the staff when he is anxious 5. The nurse notes effectiveness of interventions in using subjective and objective data in the: progress notes Situation 2 – A research study was undertaken in order to identify and analyze a disabled boy’s coping reaction pattern during stress. 6. This study which is an in depth study of one boy is a: case study 7. The process recording was the principal tool for data collection. Which of the following is NOT a part of a process recording? Audio-visual recording

8. Which of these does NOT happen in a descriptive study? Manipulation of variable 9. The investigator also provided the nursing care of the subject. The investigator is referred to as a/an: Observer researcher 10. To ensure reliability of the study, the investigator’s analysis and interpretations were: subjected to an inter-observer agreement

Situation 3 – During the morning endorsement, the outgoing nurse

informed the nursing staff that Regina, 35 years old, was given Flurazepam (Dalmane) 15mg at 10:00pm because she had trouble going to sleep. Before approaching Regina, the nurse read the observation of the night nurse. 11. Which of the following approaches of the nurse validates the data gathered? “I learned that you were up till ten last night, tell me what happened before you were finally able to sleep and how was your sleep?”

12. Regina is a high school teacher. Which of these information LEAST communicate attention and care for her needs for information about her medicine? Provide a drug literature. 13. The nurse engages Regina in the process of mutual inquiry to provide an opportunity for Regina to: perceive her participation in an experience !4. Which of these responses indicate that Regina needs further discussion regarding special instructions? “I like taking this sleeping pill. It solves my problem of insomnia. I wish I can take it for life.” 15. Regina commits to herself that she understood and will observe all the medicine precautions by: committing what she learned to her memory Situation 4 – The nurse-patient relationship is a modality through which the nurse meets the client’s needs.

16. The nurse’s most unique tool in working with the emotionally ill client is his/her communication skills 17. The psychiatric nurse who is alert to both the physical and emotional needs of clients is working from the philosophical framework that states: All behavior is meaningful, communicating a message or a need. 18. One way to increase objectivity in dealing with one’s fears and anxieties is through the process of: validation

19. All of the following responses are non therapeutic. Which is the MOST direct violation of the concept, congruence of behavior? Communicating ambivalent messages to the client. 20. The mentally ill person responds positively to the nurse who is warm and caring. This is a demonstration of the nurse’s role as: mother surrogate Situation 5 – The nurse engages the client in a nurse-patient interaction. 21. The best time to inform the client about terminating the nursepatient relationship is: at the start of the relationship

22. The client says, “I want to tell you something but can you promise that you will keep this a secret?” A therapeutic response of the nurse is: “Yes, our interaction is confidential provided the information you tell me is not detrimental to your safety.”

23. When the nurse respects the client’s self-disclosure, this is a gauge for the nurse’s: trustworthiness 24. Rapport has been established in the nurse-client relationship. The client asks to visit the nurse after his discharge. The appropriate response of the nurse would be: “The best time to talk is during the nurse-client interaction time. I am committed to have this time available for us while you are at the hospital and ends after your discharge.” 25. The client has not been visited by relatives for months. He gives a telephone number and requests the nurse to call. An appropriate action of the nurse would be: Ask the client what is the purpose of contacting his relatives. Situation 6 – Camila, 25 years old, was reported to be gradually withdrawing and isolating herself from friends and family members. She became neglectful of her personal hygiene. She was observed to be talking irrelevantly and incoherently. She was diagnosed as schizophrenia disorder. 26. The past history of Camila would most probably reveal that her premorbid personality is:

schizoid 27. Camila refuses to relate with to others because she: anticipates rejection 28. Which of the following disturbances in interpersonal relationships MOST often predispose to the development of schizophrenia? Faulty family atmosphere and interaction 29. Camila’s indifference toward the environment is a compensatory behavior to overcome: Insecurity feelings 30. Schizophrenia is a/an: psychosis Situation 7 – Salome, 80 year old widow, has been observed to be irritable, demanding and speaking louder than usual. She would prefer to be alone and take her meals by herself, minimize receiving visitors at home and no longer bothers to answer telephone calls because of deterioration of hearing. She was brought by her daughter to the Geriatric clinic for assessment and treatment. 31. The nurse counsels Salome’s daughter that Salome’s becoming very loud and tendency to become aggressive is a/an: overcompensation for hearing loss 32. A nursing diagnosis for Salome is: social isolation

33. The nurse will assist Salome and her daughter to plan a goal which is for Salome to: adjust to the loss of sensory and perceptual function 34. The daughter understood, the following ways to assist Salome meet her needs and avoiding which of the following: Allowing her to take her meals alone 35. Salome was fitted a hearing aid. She understood the proper use and wear of this device when she says that the battery should be functional, the device is turned on and adjusted to a: prescribed level Situation 8 – For more than a month now, Cecilia is persistently feeling restless, worried and feeling as if something dreadful is going to happen. She fears being alone in places and situations where she thinks that no one might come to rescue her just in case something happens to her. 36. Cecilia is demonstrating: agoraphobia 37. Cecilia’s problem is that she always sees and thinks negative things hence she is always fearful. Phobia is a symptom described as: neurotic 38. Cecilia has a lot of irrational thoughts: The goal of therapy is to modify her: cognition

39. Cognitive therapy is indicated for Cecilia when she is already able to handle anxiety reactions. Which of the following should the nurse implement? provide positive reinforcement for acceptable behavior 40. After discharge, which of these behaviors indicate a positive result of being able to overcome her phobia? She drives alone along the long expressway. Situation 9 – It is the first day of clinical experience of nursing students at the Psychiatry Ward. During the orientation, the nurse emphasizes that the team members including nursing students are legally responsible to safeguard patient’s records from loss or destruction or from people not authorized to read it. 41. It is unethical to tell one’s friends and family members data about patients because doing so is a violation of patients’ rights to: Confidentiality 42. The nurse must see to it that the written consent of mentally ill patients must be taken from: Parents or legal guardian 43. In an extreme situation and when no other resident or intern is available, should a nurse receive telephone orders, the order has to be correctly written and signed by the physician within: 24 hours 44. The following are SOAP (Subjective – Objective – Analysis – Plan) statements on a problem: Anxiety about diagnosis. What is the objective data?

Has periods of crying; frequently verbalizes fear of what diagnostic tests will reveal 45. Nursing care plans provide very meaningful data for the patient profile and initial plan because the focus is on the: Identification of patient’s responses to medical diagnoses and treatment Situation 10 – Marie is 5½ years old and described by the mother as bedwetting at night. 46. Which of the following is the MOST common physiological cause of night bed wetting? familial and genetic factors 47. All of the following, EXCEPT one compromise the concepts of behavior therapy program. placebo as a form of treatment 48. To help Marie who bed wets at night practice acceptable and appropriate behavior, it is important for the parents to be consistent with the following approaches EXCEPT:

49. A therapeutic verbal approach that communicates strong disapproval is:

“If you don’t make an effort to control your bedwetting, I’d be upset and disappointed.” 50. During your conference, the parent inquires how to motivate

Marie to be dry in the morning. Your response which is an immediate intervention would be: What does your child want that you can give every time he/she wakes up dry in the morning?

Related Documents